Acca f7 Course Notes

You might also like

Download as docx, pdf, or txt
Download as docx, pdf, or txt
You are on page 1of 202

lOMoARcPSD|3181314

FAR Course notes


lOMoARcPSD|3181314

CHAPTER 1:
A REGULATORY FRAMEWORK FOR FINANCIAL
REPORTING
1. ACCA SYLLABUS GUIDE OUTCOME 1:-
Explain why a regulatory framework is needed.
A regulatory framework is needed to ensure relevant and reliable information is given to users.
Financial statements are used by a wide range of users. Thus, these statements need to be useful
to these users.
A regulatory framework regulates the behaviour of companies towards their investors. They
increase users’ understanding of, and their confidence, in financial statements.
2. ACCA SYLLABUS GUIDE OUTCOME 2:-
Explain why accounting standards on their own are not a complete regulatory
framework.
Accounting standards on their own are not a complete regulatory framework. Legal and market
regulations are also required to regulate the preparation and presentation of financial statements.
3. ACCA SYLLABUS GUIDE OUTCOME 3:-
Distinguish between a principles-based and a rules-based framework and discuss
whether they can be complementary.
Principles-based Framework
Principles which reflect the initial objectives of financial statements are set. All accounting
standards then follow these principles.
Therefore, a principles-based framework is based upon a conceptual framework. In the next
chapter, we will be discussing the IASB’s Conceptual Framework for Financial Reporting, which
is a conceptual framework.
Rules-based Framework
Rules are laid out as events arise, designed to cover all eventualities. Therefore, accounting
standards are a set of rules which companies must follow.
1.4 ACCA SYLLABUS GUIDE OUTCOME 4:-
Describe the IASB’s Standard setting process including revisions to and interpretations
of standards.
International Financial Reporting Standards (IFRSs) are developed through an international
consultation process, the "due process".
The due process comprises six stages :- 1
1. Setting the agenda – the IASB identifies a subject (mainly by reference to the needs of
the investors)
2. Planning the project - after considering the nature of the issues and the level of interest
among constituents, the IASB may establish a working group at this stage.
3. Developing and publishing the discussion paper - Typically, a discussion paper includes:
a. a comprehensive overview of the issue;

1 www.iasb.org
lOMoARcPSD|3181314

b. possible approaches in addressing the issue


c. the preliminary views of its authors or the IASB;
d. an invitation to comment.
4. Developing and publishing the exposure draft for public comment , which is a draft
version of the intended standard.
5. Developing and publishing the standard following the consideration of comments
received on the exposure draft.
6. After the standard is issued, the staff and the IASB members hold regular meetings with
interested parties, including other standard-setting bodies, to help understand
unanticipated issues related to the practical implementation and potential impact of its
proposals.
1.5 ACCA SYLLABUS GUIDE OUTCOME 5:-
Explain the relationship of national standard setters to the IASB in respect of the
standard setting process.
In order to achieve its objectives, the IASB works in partnership with the major national
standard-setting bodies:-
1. They coordinate each other’s work plans.
2. They review each other’s standards and give priority where differences are greatest.
3. National standard setters can issue IASB discussion papers and exposure draft for
comments in their own countries.
4. National standard setters may include more guidance in their exposure drafts on relevant
issues to them.
1.6 ACCA SYLLABUS GUIDE OUTCOME 6:-
The advantages and disadvantages of IFRS over a national regulatory
framework. 2
1.6.1 What are the benefits of adopting IFRS to national standards?
IFRS have the following advantages for those companies that adopt them:
• IFRS are widely accepted as a set of high-quality and transparent global standards that
are intended to achieve consistency and comparability across the world. They have been
produced in cooperation with other internationally renowned standard setters, with the
aspiration of achieving consensus and global convergence.
• Companies that use IFRS and have their financial statements audited in accordance with
International Standards on Auditing (ISA) will have an enhanced status and reputation
(e.g. improved credit ratings).
• The International Organisation of Securities Commissions (IOSCO) recognise IFRS for
listing purposes – thus companies that use IFRS need produce only one set of financial
statements for any securities listing for countries that are members of IOSCO. This
makes it easier and cheaper to raise finance in international markets.

2 ACCA F7 Exam June 2012 Qs 5b


lOMoARcPSD|3181314

•Companies that own foreign subsidiaries will find the process of consolidation simplified
if all their subsidiaries use IFRS.
• Companies that use IFRS will find their results are more easily compared with those of
other companies that use IFRS. This would help the company to better assess and rank
prospective investments in its foreign trading partners.
1.6.2 What are the challenges of adopting IFRS to national standards?
1. Changes to regulatory environment, i.e. laws and regulations
2. IFRS training to finance staff and regulators
3. Greater complexity in the financial reporting process
In addition to significant use of judgment, companies would need to increasingly use fair
value measures. For small and medium companies, the use of IFRSs and the extensive
disclosures they require, may significantly increase their costs in the preparation of
financial statements.
1.7 ACCA SYLLABUS GUIDE OUTCOME 7:-
Distinguish between the primary aims of not-for-profit and public sector entities and
those of profit oriented entities.
Not-for-profit organisations are distinguished from profit maximising organisations by three
characteristics.
1. Most not-for-profit organisations do not have external shareholders providing risk capital
for the business. Finance is mainly limited to donations and/or government subsidies.
2. They do not distribute dividends, so any profit (or surplus) that is generated is retained by
the business as a further source of capital.
3. Their objective is to achieve value for money. Thus, it usually includes some social,
cultural, philanthropic, welfare or environmental dimension, which in their absence,
would not be readily provided efficiently through the workings of the market system. In
the case of a profit oriented sector, the main aim is to make a profit and increase
shareholder wealth.

1.8 ACCA SYLLABUS GUIDE OUTCOME 8:-


Discuss the extent to which International Financial Reporting Standards (IFRSs) are
relevant to specialised, not-for-profit and public sector entities.
Accounting standards are required to measure the financial position and performance of
organisations.
Not-for-profit and public sector entities do not have to produce financial statements for the public
but they will have to account for their income and costs. They rely on measures that estimate the
performance of the organisation in relation to:
1. effectiveness – the extent to which the organisation achieves its objectives
2. economy – the ability of the organisation to optimise the use of its productive resources
(keeping the cost of input resources as low as possible)
3. efficiency – the ‘output’ of the organisation per unit of resource consumed
lOMoARcPSD|3181314

3
3
F7 Dec 14 MCQ 15
CHAPTER 2:
A CONCEPTUAL FRAMEWORK FOR FINANCIAL REPORTING
2.1 ACCA SYLLABUS GUIDE OUTCOME 1:-
Describe what is meant by a conceptual framework of accounting.
In a broad sense, a conceptual framework can be seen as an attempt to define the nature and
purpose of accounting. A conceptual framework must consider the theoretical and conceptual
issues surrounding financial reporting and form a coherent and consistent foundation that will
underpin the development of accounting standards.
A conceptual framework can be seen as a statement of generally accepted accounting principles
(GAAP) that form a frame of reference for the evaluation of existing practices and the
development of new ones. As the purpose of financial reporting is to provide useful information
as a basis for economic decision making, a conceptual framework will form a theoretical basis
for determining how transactions should be measured (historical value or current value) and
reported – i.e. how they are presented or communicated to users. 3
The IASB’s Conceptual Framework for Financial Reporting describes the basic concepts by
which financial statements are prepared. The main purpose of the Framework is to:
• assist in the development of future IFRS and the review of existing standards by setting
out the underlying concepts
• promote harmonisation of accounting regulation and standards by reducing the number of
permitted alternative accounting treatments
• assist the preparers of financial statements in the application of IFRS, which would
include dealing with accounting transactions for which there is not (yet) an accounting
standard.
The Framework is also of value to auditors, and the users of financial statements, and more
generally help interested parties to understand the IASB’s approach to the formulation of an
accounting standard. 4
The Framework is not an accounting standard, and where there is perceived to be a conflict
between the Framework and the specific provisions of an accounting standard then the
accounting standard prevails. 5
The content of the Framework can be summarised as follows:
• Identifying the objective of financial statements
• The reporting entity (to be issued)
• Identifying the parties that use financial statements

3 “The need for and an understanding of a conceptual framework” by Steve Scott, October 2011
http://www.accaglobal.com/content/dam/acca/global/PDF-students/2012/sa_oct11_framework.pdf
4 “The need for and an understanding of a conceptual framework” by Steve Scott, October 2011
http://www.accaglobal.com/content/dam/acca/global/PDF-students/2012/sa_oct11_framework.pdf
5 “The IASB’s conceptual framework for financial reporting” by Tom Clendon, March 2011
http://www.accaglobal.com/content/dam/acca/global/PDF-students/acca/tech/sa_mar11_f7p2.pdf
lOMoARcPSD|3181314

• The qualitative characteristics that make financial statements useful


• The remaining text of the old Framework dealing with elements of financial statements:
assets, liabilities equity income and expenses and when they should be recognised and a
discussion of measurement issues (for example, historic cost, current cost) and the related
concept of capital maintenance.
2.2 ACCA SYLLABUS GUIDE OUTCOME 2:-
Discuss whether a conceptual framework is necessary and what an alternative system
might be6.
Some accountants have questioned whether a conceptual framework is necessary in order to
produce reliable financial statements. Past history of standard setting bodies throughout the
world tells us it is. In the absence of a conceptual framework, accounting standards were often
produced that had serious defects – that is:
• they were not consistent with each other particularly in the role of prudence versus
accruals/matching
• they were also internally inconsistent and often the effect of the transaction on the
statement of financial position was considered more important than its effect on income
the statement
• standards were produced on a ‘fire fighting’ approach, often reacting to a corporate
scandal or failure, rather than being proactive in determining best policy
• some standard setting bodies were biased in their composition (i.e. not fairly
representative of all user groups) and this influenced the quality and direction of
standards
• the same theoretical issues were revisited many times in successive standards – for
example, does a transaction give rise to an asset (research and development expenditure)
or liability (environmental provisions)?
It could be argued that the lack of a conceptual framework led to a proliferation of ‘rulesbased’
accounting systems whose main objective is that the treatment of all accounting transactions
should be dealt with by detailed specific rules or requirements. Such a system is very prescriptive
and inflexible, but has the attraction of financial statements being more comparable and
consistent.
By contrast, the availability of a conceptual framework could lead to ‘principles-based’ system
whereby accounting standards are developed from an agreed conceptual basis with specific
objectives. 7
What is the alternative to a conceptual framework?
The IASB’s Conceptual Framework for Financial Reporting is based on principles. IFRSs do
contain many rules and requirements but these are based on underlying concepts.

6 ACCA F7 Exam June 2012 Qs 5


7 “The need for and an understanding of a conceptual framework” by Steve Scott, October 2011
http://www.accaglobal.com/content/dam/acca/global/PDF-students/2012/sa_oct11_framework.pdf
lOMoARcPSD|3181314

In reality, most accounting systems have an element of both rules and principles. Their
designation whether they are rules-based or principles-based depends on the relative importance
of the principles compared to the volume and the manner in which the rules are derived.
2.3 The IASB’s Conceptual Framework for Financial Reporting
2.3.1 Why are financial statements prepared?
The objective of financial reporting is to provide financial information about the reporting entity
that is useful to existing and potential investors, lenders and other creditors in making decisions
about providing resources to the entity.
Some observations about the objective: -
1. Users
This revised Framework lists as primary users of financial statements, existing or potential
investors, lenders and other creditors. Hence, it is aimed primarily at capital providers. This
does not mean that other users will not find financial reports useful; for e.g. employees,
suppliers, customers, governments and the general public. It is just
lOMoARcPSD|3181314

that, in deciding on the principles for recognition, measurement, presentation and disclosure, the
information needs of capital providers are paramount. 2. Financial information
The revised Framework introduces a broader reference to financial information, i.e. reporting of
an entity’s economic resources, claims and changes therein.
Information about the nature and amounts of a reporting entity’s economic resources and claims
assists users to assess that entity’s financial strengths and weaknesses; to assess liquidity and
solvency, and its need and ability to obtain financing. A reporting entity’s economic resources
and claims are reported in the statement of financial position.
Changes in a reporting entity’s economic resources and claims result from that entity’s
performance and from other events or transactions such as issuing debt or equity instruments.
Users need to be able to distinguish between both of these changes. This is useful in assessing
the entity’s past and future ability to generate net cash inflows. Such information may also
indicate the extent to which general economic events have changed the entity’s ability to
generate future cash inflows. These changes are presented in the statement of comprehensive
income – Statement of profit or loss and other comprehensive income.
Information about a reporting entity’s cash flows during the reporting period also assists users to
assess the entity’s ability to generate future net cash inflows. This information indicates how the
entity obtains and spends cash, including information about its borrowing and repayment of debt,
cash dividends to shareholders, etc. The changes in the entity’s cash flows are presented in the
statement of cash flows.
Information about changes in an entity’s economic resources and claims resulting from events
and transactions other than financial performance, such as the issue of equity instruments or
distributions of cash or other assets to shareholders is necessary to complete the picture of the
total change in the entity’s economic resources and claims. This is presented in the statement of
changes in equity.
3. Decision usefulness and stewardship
Decision usefulness to capital providers is the overriding purpose of financial reporting, as well
as assessing the stewardship of resources already committed to the entity.
The ability of management to discharge their stewardship responsibilities effectively has an
effect on the entity’s ability to generate net cash inflows in the future, implying that potential
investors are also assessing management performance as they make their investment decision.
11!
2.3.2 Underlying Assumption
The main underlying assumption is:

• Going Concern
The financial statements presume that an enterprise will continue in operation in the
foreseeable future or, if that presumption is not valid, disclosure and a different basis of
reporting are required.

acowtancy.com

Downloaded by sajedul Islam (sajedulnsu15@gmail.com)


lOMoARcPSD|3181314

What is ‘accrual basis’?


• Accrual Basis
The effects of transactions and other events are recognised when they occur, rather than
when cash or its equivalent is received or paid, and they are reported in the financial
statements of the periods to which they relate.

2.3.3 ACCA SYLLABUS GUIDE OUTCOME 3:-


Discuss what is meant by relevance and faithful representation and describe the qualities
that enhance these characteristics.
Discuss whether faithful representation constitutes more than compliance with
accounting standards.
Discuss what is meant by understandability and verifiability in relation to the provision
of financial information.
Discuss the importance of comparability and timeliness to users of financial statements.
Qualitative Characteristics of Financial Information8

The revised Framework distinguishes between two types of qualitative characteristics that are
necessary to provide useful financial information: -
1. Fundamental qualitative characteristics (relevance and faithful representation) and
2. enhancing qualitative characteristics (comparability (including consistency), timeliness,
verifiability and understandability).
2.3.3.1 Fundamental Qualitative Characteristics
For information to be useful, it must be both relevant and faithfully represented.
1. Relevance
Influences economic decisions of user
Relevant financial information is capable of making a difference in the decisions made by users.
Has predictive value and/or confirmatory value or both
Relevant information assists in the predictive ability of financial statements.
That is not to say the financial statements should be predictive in the sense of forecasts, but that
(past) information should be presented in a manner that assists users to assess an entity’s ability
to take advantage of opportunities and react to adverse situations.
For example, discontinued operations are presented separately in the statement of profit or loss.
From this information, users will be better able to assess the parts of the entity that will produce
future profits (continuing operations).
Users can also judge the merits of the discontinuation:- why was that part of the business sold?
Was it to curtail the adverse effect of a loss making operation?
Materiality

8 examined December 2013 Qs 4


!9 acowtancy.com

Downloaded by sajedul Islam (sajedulnsu15@gmail.com)


lOMoARcPSD|3181314

Materiality is a threshold or cut-off point for information whose omission or misstatement could
influence the economic decisions of users taken on the basis of the financial statements.
This depends on the size of the item or error judged in the particular circumstances of its
omission or misstatement.
Hence, materiality is not a matter to be considered by standard-setters but by preparers and their
auditors.
2. Faithful Representation
General purpose financial reports represent economic phenomena in words and numbers.
To be useful, financial information must not only be relevant, it must also represent faithfully the
phenomena it purports to represent. Financial statements will generally show a fair presentation
when:
• They conform with accounting standards
• They conform with the any relevant legal requirements
• They have applied the qualitative characteristics from the Framework.
Financial information that faithfully represents economic phenomena has three
characteristics: -
❖ it is complete
❖ it is neutral ❖ it is free from error
.
The revised Framework acknowledges limitations in achieving a faithful representation, e.g. due
to inherent uncertainties, estimates and assumptions. Accordingly, financial information might
not always be entirely free from error. Faithful representation, however, is achieved if no errors
or omissions affect the description of economic phenomena and the process applied to produce
reported information has been selected and applied without errors.
Disclosure
The entity should disclose:
• that management has concluded that the financial statements do give a fair presentation
• that it has complied with IFRSs and Interpretations except where it has departed in order
to achieve a fair presentation
• the standard or interpretation from which the entity has departed, and an explanation of
the circumstances
• the financial impact of the departure. If the relevant regulatory framework prohibits such
departure, the circumstances should be explained
2.3.3.2 Enhancing Qualitative Characteristics9
Comparability, verifiability, timeliness and understandability are directed to enhance both
relevant and faithfully represented financial information. Those characteristics should be
maximised both individually and in combination.

9 Examinable Dec 2012 Qs 4a


!10 acowtancy.com

Downloaded by sajedul Islam (sajedulnsu15@gmail.com)


lOMoARcPSD|3181314

1. Comparability
Users can identify similarities and differences
Comparability is fundamental to assessing the performance of an entity by using its financial
statements. Assessing the performance of an entity over time (trend analysis) requires that the
financial statements used have been prepared on a comparable (consistent) basis. Generally this
can be interpreted as using consistent accounting policies (unless a change is required to show a
fairer presentation). A similar principle is relevant to comparing one entity with another;
however it is more difficult to achieve consistent accounting policies across entities10.
Consistent application of methods
Comparability is enhanced by the use and disclosure of consistent accounting policies. Users can
confirm that comparative information for calculating trends is comparable. The disclosure of
accounting policies at least informs users if different entities use different policies.
Comparability should be distinguished from consistency (the consistent use of accounting
methods).
It is recognised that there are situations where it is necessary to adopt new accounting policies
(usually through new Standards) if they enhance relevance and reliability. Consistency and
comparability require the existence and disclosure of accounting policies.
IAS 8 “Accounting Policies, changes in accounting estimates and errors” will be discussed in
more detail in Chapter 3.
2. Verifiability
Financial information is verifiable when it enables knowledgeable and independent observers to
reach a consensus on whether a particular depiction of an event or transaction is a faithful
representation. 3. Timeliness
Timeliness means that information is available to decision-makers in time to be capable of
influencing their decisions.
4. Understandability
Understandability is enhanced when the information is:
1. classified
2. characterised
3. presented clearly and concisely
However, relevant information should not be excluded solely because it may be too complex and
cannot be made easy to understand. To exclude such information would make financial reports
incomplete and potentially misleading. Financial reports are prepared for users who have a
reasonable knowledge of business and economic activities and who review and analyse the
information with diligence. The Cost Constraint on Useful Financial Reporting
Cost is a pervasive constraint to financial reporting. Reporting such information imposes costs
and those costs should be justified by the benefits of reporting that information. The IASB
assesses costs and benefits in relation to financial reporting generally, and not solely in relation

10 ACCA F7 Exam June 2008 Question 4


!11 acowtancy.com

Downloaded by sajedul Islam (sajedulnsu15@gmail.com)


lOMoARcPSD|3181314

to individual reporting entities. The IASB will consider whether different sizes of entities and
other factors justify different reporting requirements in certain situations.
2.3.4 ACCA SYLLABUS GUIDE OUTCOME 4:-
Define what is meant by ‘recognition’ in financial statements and discuss the recognition
criteria.
For an item to be recognised in the accounts it must pass three tests:
a) Meet the definition of an asset/liability or income/expense or equity
b) It is probable that any future economic benefit associated with the item will flow to or
from the entity
c) The item has a cost or value than can be reliably measured
2.3.5 ACCA SYLLABUS GUIDE OUTCOME 5:-
Apply the recognition criteria to:
a. assets and liabilities
b. income and expenses
Asset
An asset is a resource controlled by the enterprise as a result of past events and from which
future economic benefits (normally net cash inflows) are expected to flow to the entity.
Assets can only be recognised in the statement of financial position when those expected benefits
are probable and can be measure reliably11.
While most assets will be both controlled and legally owned by the entity, it should be noted that
legal ownership is not a prerequisite for recognition, rather it is control that is the key issue.
Liability
A liability is a present obligation of the enterprise arising from past events, the settlement of
which is expected to result in an outflow from the enterprise of resources embodying economic
benefits (normally cash)12.
Liabilities are also presented on the statement of financial position as being non-current or
current. Examples of liabilities include trade payables, tax creditors and loans.
It should be noted that in order to recognise a liability there does not have to be an obligation that
is due on demand but rather there has to be a present obligation. Thus for example IAS 37,
Provisions, Contingent Liabilities and Contingent Assets is consistent with the Framework's
approach when considering whether there is a liability for the future costs to decommission oil
rigs. As soon as a company has erected an oil rig that it is required to dismantle at the end of the
oil rig's life, it will have a present obligation in respect of the decommissioning costs. This
liability will be recognised in full as a non-current liability and measured at present value to
reflect the time value of money. The past event that creates the present obligation is the original

11 ACCA F7 Exam Dec 07 Q5a; PP Q4bii

12 ACCA F7 Exam Dec 08 Q4


!12 acowtancy.com

Downloaded by sajedul Islam (sajedulnsu15@gmail.com)


lOMoARcPSD|3181314

erection of the oil rig as once it is erected the company is responsible to incur the costs of
decommissioning. 13
Equity
Equity is the residual interest in the assets of the enterprise after deducting all its liabilities.
This definition acknowledges the supreme conceptual importance of identifying, recognising and
measuring assets and liabilities, as equity is conceptually regarded as a function of assets and
liabilities, i.e. a balancing figure.
Equity includes the original capital introduced by the owners, i.e. share capital and share
premium, retained earnings, unrealised asset gains in the form of revaluation reserves and, in
group accounts, the equity interest in the subsidiaries not enjoyed by the parent company, i.e. the
non-controlling interest (NCI) (will be discussed in Chapter 5). Equity can also include the
equity element of convertible loan stock (will be discussed in Chapter 15), equity settled share
based payments, differences arising when there are increases or decreases in the NCI, group
foreign exchange differences and contingently issuable shares. These would probably all be
included in equity under the umbrella term of Other Components of Equity. 15
Income
Income is increases in assets (or decreases of liabilities) that result in increases in equity, other
than contributions from equity participants.
Most income is revenue generated from the normal activities of the business in selling goods and
services, and as such is recognised in the Income section of the Statement of Profit or Loss and
other Comprehensive Income. However certain types of income are required by specific
standards to be recognised directly to equity, i.e. reserves, for example certain revaluation gains
on assets. In these circumstances the income (gain) is then also reported in the Other
Comprehensive Income section of the Statement of Profit or Loss and other Comprehensive
Income.
The reference to ‘other than those relating to contributions from equity participants’ means that
when the entity issues shares to equity shareholders, while this clearly increases the asset of cash,
it is a transaction with equity participants and so does not represent income for the entity. 14
Expense
Expenses are decreases in assets or (incurrences of liabilities) that result in decreases in equity,
other than distributions to equity participants.

13 “The IASB’s Conceptual Framework for Financial Reporting”, T. Clendon, March 2011

http://www.accaglobal.com/content/dam/acca/global/PDF-students/acca/tech/sa_mar11_f7p2.pdf
15
“The IASB’s Conceptual Framework for Financial Reporting”, T. Clendon, March 2011
http://www.accaglobal.com/content/dam/acca/global/PDF-students/acca/tech/sa_mar11_f7p2.pdf
14 “The IASB’s Conceptual Framework for Financial Reporting”, T. Clendon, March 2011

http://www.accaglobal.com/content/dam/acca/global/PDF-students/acca/tech/sa_mar11_f7p2.pdf
17
“The IASB’s Conceptual Framework for Financial Reporting”, T. Clendon, March 2011
http://www.accaglobal.com/content/dam/acca/global/PDF-students/acca/tech/sa_mar11_f7p2.pdf
!13 acowtancy.com

Downloaded by sajedul Islam (sajedulnsu15@gmail.com)


lOMoARcPSD|3181314

The reference to ‘other than those relating to distributions to equity participants’ refers to the
payment of dividends to equity shareholders. Such dividends are not an expense and so are not
recognised anywhere in the Statement of Profit or Loss. Rather they represent an appropriation
of profit that is as reported as a deduction from Retained Earnings in the Statement of Changes in
Equity.
Examples of expenses include depreciation, impairment of assets and purchases. As with income
most expenses are recognised in the Statement of Profit or Loss section of the Statement of Profit
or Loss and Comprehensive Income, but in certain circumstances expenses (losses) are required
by specific standards to be recognised directly in equity and reported in the Other
Comprehensive Income Section of the Statement of Profit or Loss and other Comprehensive
Income. An example of this is an impairment loss, on a previously revalued asset, that does not
exceed the balance of its Revaluation Reserve (will be discussed in detail in Chapter 19). 17
2.4 ACCA SYLLABUS GUIDE OUTCOME 6:-
Discuss revenue recognition issues; indicate when income and expense recognition
should occur.
Looking into the definitions provided in the Framework, you can see that the Framework takes a
financial position approach. It focuses on assets and liabilities and that income and expense
definitions are derived from them rather than vice-versa.
Income results from changes that increase the entity’s wealth, and losses result from changes that
decrease its wealth. Thus, revenues arise from increases in assets or decreases in liabilities, while
expenses result from decreases in assets or increases in liabilities.
Further Questions
Question 1 15
Financial statements represent transactions in words and numbers. To be useful, financial
information must represent faithfully these transactions in terms of how they are reported.
Which of the following accounting treatments would be an example of faithful representation?
A. Charging the rental payments for an item of plant to the statement of profit or loss
where the rental agreement meets the criteria for a finance lease
B. Including a convertible loan note in equity on the basis that the holders are likely to
choose the equity option on conversion
C. Derecognising factored trade receivables sold without recourse
D. Treating redeemable preference shares as part of equity in the statement of financial
position
Question 2 16
Which of the following is NOT a purpose of the IASB’s Conceptual Framework?
A. To assist the IASB in the preparation and review of IFRS
B. To assist auditors in forming an opinion on whether financial statements comply with
IFRS

15 Specimen Exam Applicable from December 2014


!14 acowtancy.com

Downloaded by sajedul Islam (sajedulnsu15@gmail.com)


lOMoARcPSD|3181314

C. To assist in determining the treatment of items not covered by an existing


IFRS
D. To be authoritative where a specific IFRS conflicts with the Conceptual Framework
Question 3 17
Although the objectives and purposes of not-for-profit entities are different from those of
commercial entities, the accounting requirements of not-for-profit entities are moving closer to
those entities to which IFRSs apply.
Which of the following IFRS requirements would NOT be relevant to a not-for-profit entity?
A. Preparation of a statement of cash flows
B. Requirement to capitalise a finance lease
C. Disclosure of earnings per share
D. Disclosure of non-adjusting events after the reporting date
Question 4
The IASB’s Conceptual Framework for Financial Reporting is
A. A set of financial reporting standards
B. A set of items which make up an entity’s financial statements
C. A set of principles which underpin financial reporting
D. A set of regulations which govern financial reporting
Question 5
The IASB conceptual framework is being developed jointly with:
A. The UK Accounting Standards Board
B. National Accounting Standard Setters
C. The European Union
D. The US Financial Accounting Standards Board
Question 6
The objective of preparing financial statements is to provide information about the reporting
entity which is useful to:
A. Investors and employees
B. Investors and lenders
C. Employees and lenders
D. Investors and customers
Question 7
The two fundamental qualitative characteristics of financial information are:
A. Relevance and faithful representation
B. Relevance and comparability
C. Faithful representation and substance over form
D. Verifiability and understandability

16 Specimen Exam Applicable from December 2014


17 Specimen Exam Applicable from December 2014
!15 acowtancy.com

Downloaded by sajedul Islam (sajedulnsu15@gmail.com)


lOMoARcPSD|3181314

Question 8
The enhancing qualitative characteristics of financial information include:
A. Relevance and faithful representation
B. Comparability and understandability
C. Relevance and timeliness

Which of the following isnot a contributory factor towards faithful representation?

Which of the following statements about the Framework are true?


(1) The Framework is an accounting standard.
(2) It assists in harmonising accounting practice.
(3) It assists national standard setters in developing national standards.
(4) It assists users of the accounts to interpret financial statements.

D. Understandability and faithful representation


Question 9
A. Completeness
B. Freedom from error
C. Neutrality
D. Predictive value
Question 10
A. 1 and 2
B. 2, 3 and 4
C. All 4
D. 1 and 3
CHAPTER 3:
ACCOUNTING POLICIES AND CONCEPTS
IAS 8: ACCOUNTING POLICIES, CHANGES IN ACCOUNTING ESTIMATES AND
ERRORS
3.1 ACCA SYLLABUS GUIDE OUTCOME 1:-

!16 acowtancy.com

Downloaded by sajedul Islam (sajedulnsu15@gmail.com)


lOMoARcPSD|3181314

Distinguish between changes in accounting policies and changes in accounting


estimates and describe how accounting standards apply the principle of comparability
where an entity changes its accounting policies. Recognise and account for changes in
accounting policies.
3.1.1 Accounting Policies
Definition
Accounting policies are:
the specific principles, bases, conventions, rules and practices applied by an entity in preparing
and presenting the financial statements
An entity should follow accounting standards when deciding its accounting policies. If there is
no guidance in the standards, management should use the policy which gives the most relevant
and reliable information as outlined in the Framework for the Preparation and Presentation of
Financial Statements.
Changes in accounting policies are only
made if:
• It is required by a Standard or Interpretation; or
• It would give more relevant and reliable information
Accounting treatment
• Adjust the opening balance of the item affected in equity (e.g. retained earnings) – put
this in the statement of changes in equity also (second line)
• Adjust the comparative amounts for the affected item as if the policy had always been
applied
Disclosures Required
• The nature of the change in accounting policy
• The reasons for the change
• The amount of the adjustment
3.1.2 Changes in Accounting Estimates
Definition

!17 acowtancy.com

Downloaded by sajedul Islam (sajedulnsu15@gmail.com)


lOMoARcPSD|3181314

A change in an accounting estimate is an adjustment of the carrying amount of an asset


or liability, or related expense, resulting from reassessing the expected future benefits
and obligations associated with that asset or liability.

Allowances for doubtful debts;

Inventory obsolescence;

Useful economic life of property, plant and equipment

Simply change the current period. No change to comparatives

Examples



Accounting Treatment
3.2 ACCA SYLLABUS GUIDE OUTCOME 2:-
Recognise and account for the correction of prior period errors.
3.2.1 Prior Period Errors
Definition
Prior period errors are omissions from, and misstatements in, an entity's financial statements for
one or more prior periods arising from a failure to use, or misuse of, reliable information that
was available and could reasonably be expected to have been obtained and taken into account in
preparing those statements.
Examples
• mathematical mistakes,
• mistakes in applying accounting policies, • oversights or misinterpretations of facts, and
• fraud.
Accounting treatment
• adjust the opening balance of the item affected in equity (eg retained earnings) – put this
in the statement of changes in equity also (second line)
• restate the comparative amounts for the prior period(s) presented in which the error
occurred; or
• if the error occurred before the earliest prior period presented, restating the opening
balances of assets, liabilities and equity for the earliest prior period presented.
Disclosures Required
• The nature of the prior period error
• for each prior period presented, to the extent practicable, the amount of the correction
!18 acowtancy.com

Downloaded by sajedul Islam (sajedulnsu15@gmail.com)


lOMoARcPSD|3181314

• the amount of the correction at the beginning of the earliest prior period presented
• if retrospective restatement is impracticable, an explanation and description of how the
error has been corrected
Lecture Example 1
Emerald has had a policy of writing off development expenditure to the income statement as it
was incurred. 18
In preparing its financial statements for the year ended 30 September 2007, it has become aware
that, under IFRS rules, qualifying development expenditure should be treated as an intangible
asset. Below is the qualifying development expenditure for Emerald:
$’000
Year ended 30 September 2004 300
Year ended 30 September 2005 240

All capitalised development expenditure is deemed to have a four year life. Assume
amortisation commences at the beginning of the accounting period following
capitalisation. Emerald had no development expenditure before that for the year ended
30 September 2004.

Calculate the amounts which should appear in the Statement of Profit or Loss
and SFP (including comparative figures), and SOCIE for the year ended 30

ACCA F7 Exam December 2007 Question 5(b)

Year ended 30 September 2006 800


Year ended 30 September 2007 400
Required:- September
2007.
3.3 ACCA SYLLABUS GUIDE OUTCOME 3:-
Explain the following measures and compute amounts using: o
historic cost o fair value o current cost o net realisable value o
present value of future cash flows

18 now referred to as statement of profit or loss and other comprehensive income


!19 acowtancy.com

Downloaded by sajedul Islam (sajedulnsu15@gmail.com)


lOMoARcPSD|3181314

3.3.1 Historic Cost


The amount paid or fair value of the consideration given.
3.3.2 Fair Value
IFRS 13 defines fair value as “the price that would be received to sell an asset or paid 19to
transfer a liability in an orderly transaction between market participants at the measurement
date”. IFRS 13 is explained in more detail at the end of this section.
The amount that would have to be paid if the same or an equivalent asset was acquired

The amount that could currently be obtained by selling the asset, net of the estimated

3.3.5Present value of future cash flows

The present discounted value of the future net cash inflows that the item is expected to

A company owns a machine which was purchased last year for $280,000. Depreciation

3.3.3 Current Cost


currently.
3.3.4 Net realisable value
selling and completion costs.
generate. Illustration 1
is provided at 25% straight line.
It is estimated that this machine could be sold second hand for $88,000 although the company
would have to spend about $500 in advertising costs to do so.
If replaced, the machine would cost $360,000, although this current model is 20% more efficient.
The machine is expected to generate net cash inflows of $40,000 for the next 5 years after which
time it will be scrapped.
The company’s cost of borrowing is 6%.
The discount factors at 6% at the end of:
Year 1 0.943
Year 2 0.890

19 IFRS 13, Fair Value Measurement


!20 acowtancy.com

Downloaded by sajedul Islam (sajedulnsu15@gmail.com)


lOMoARcPSD|3181314

Year 3 0.840
Year 4 0.792
Year 5 0.747
What is the value of the following asset using:-
a. Historical Cost
b. Fair Value
c.Current Cost
d. Net Realisable Value
e. Present Value of Future Cashflows

Historical Cost

Cost – Accumulated Depreciation

280,000 – 70,000 = $210,000

(Accumulated depn = 0.25 x 280,000)

Fair Value

$88,000
Solution
Current cost
360,000 x 100/120 = 300,000 – 75,000(depn) = $225,000
Net realisable Value
88,000 – 500 = $87,500
Present Value of Future Cashflows
40,000 x 4.212 = $168,480
(0.943 + 0.890 + 0.840 + 0.792 + 0.747 = 4.212)
Lecture Example 2
Update has been considering the effect of alternative methods of preparing their financial
statements. As an example, they picked an item of plant that they acquired from Suppliers on 1
April 2000 at a cost of $250,000.
The following details have been obtained:
▪ the company policy is to depreciate plant at 20% per annum on the reducing balance basis.

!21 acowtancy.com

Downloaded by sajedul Islam (sajedulnsu15@gmail.com)


lOMoARcPSD|3181314

▪ suppliers’ price catalogue at 31 March 2003 shows an item of similar plant at a cost of
$320,000. On reading the specification, it appears that the new model can produce 480
units per hour whereas the model owned by Update can only produce 420 units per hour.
Required:-
Calculate for Update the depreciation charge for the plant for the year to 31 March 2003
(based on year end values) and its carrying value on that date using:
i.the historical cost basis; and ii.a
current cost basis.
(ACCA Exam Paper 2.5 June 2003 Qs 5 a(ii) revised)
Lecture Example 320
Which of the following criticisms does NOT apply to historical cost accounts during a period of
rising prices?
A. They contain mixed values; some items are at current values, some at out of date values
B. They are difficult to verify as transactions could have happened many years ago
C. They understate assets and overstate profit
D. They overstate gearing in the statement of financial position
Fair Value Measurement
IFRS 13 provides guidance on how to measure the fair value of financial and nonfinancial assets
and liabilities. Its aim is to reduce complexity and improve consistency when measuring fair
value.
When measuring fair value, an entity considers the characteristics of the asset or liability, for
example:
• the condition and location of an asset
• restrictions, if any, on the sale or use of an asset (that would transfer with the asset).
This means that when revaluing its property, plant and equipment, an entity should consider the
highest and best use of the assets.
A fair value measurement assumes that the transaction to sell the asset or transfer the liability
takes place either in the: -
1. principal market – the market with greatest volume and level of activity for the asset or
liability
2. most advantageous market (in the absence of a principal market) – the market that
maximises the amount that would be received to sell the asset or minimises the amount
that would be paid to transfer the liability.
IFRS 13 describes three different valuation techniques that may be used to measure fair value: -
1. Market approach – uses prices from market transactions involving identical or similar
assets or liabilities, e.g. quoted prices of listed equity, debt securities or futures, or market
interest rates

20 Specimen Exam Applicable from December 2014


!22 acowtancy.com

Downloaded by sajedul Islam (sajedulnsu15@gmail.com)


lOMoARcPSD|3181314

2. Income approach - this converts future cash flows to a single discounted amount; e.g.
discounted cash flow models and option pricing models
3. Cost approach – this reflects the amount required currently to replace the service
capacity of an asset, i.e. the current replacement cost.
When measuring fair value, an entity is required to maximise the use of relevant observable
inputs and minimise the use of unobservable inputs.
Level 1 Level 2 Level 3

Definition Quoted prices in active Observable inputs Unobservable inputs


markets for
identical assets or
liabilities
Example Quoted prices for an Current market rents Projected cash flows
equity security that for similar properties used to value a
trades on the Stock and market interest business or
Exchange rates or yields taken to noncontrolling interest
measure the fair value in an entity that is not
of an investment publicly listed
property
How does all this work in practice?
E.g. an entity owns 10,000 ordinary shares in M & S. Since there is an active market for these
shares through the London stock exchange, the entity must use a market approach (level 1 input).
The same principle applies to a listed debt security that is quoted in an active market. However,
the measurement of the fair value of an unlisted debt security may require the use of an income
approach, e.g. a discounted cash flow model using market interest rate for similar debt securities
(level 2 input) and market credit spreads adjusted for entity-specific credit risk (level 2 or 3
inputs).
3.4 ACCA SYLLABUS GUIDE OUTCOME 4:-
Describe the advantages and disadvantages of the use of historical cost accounting.
Historical cost has been defined as the amount paid or fair value of the consideration given.
Advantages of Historical Cost Accounting
1. the cost is known and can be proved (e.g. against an invoice). It is therefore objective
2. it enhances comparability
3. it leads to stable pricing – using current market values would lead to volatility in asset
values
Disadvantages of Historical Cost Accounting
1. non-current asset values are unrealistic

!23 acowtancy.com

Downloaded by sajedul Islam (sajedulnsu15@gmail.com)


lOMoARcPSD|3181314

2. since non-current asset values are low, depreciation is low and does not fully reflect the
value of the asset consumed during the accounting year
3. lower costs, e.g. depreciation expense, would lead to higher profits. There is a possibility
that this may lead to higher taxation, wage demands and dividend expectation (based on
overstated earnings per share). The combination of these effects is that a company may
overspend or over distribute its profits and not maintain its capital base.
4. comparisons over time are unrealistic
5. Understatement of asset values tends to overstate gearing, and leads to a low asset per
share value and can make the company vulnerable to a take over
6. Where assets, particularly land and buildings, are being used as security to raise finance,
it is current value that lenders are interested in, not historical values
These disadvantages usually arise in times of rising prices. In fact, in times of rising prices,
historical cost accounting tends to understate asset values and overstate profits.
3.5 ACCA SYLLABUS GUIDE OUTCOME 5:-
Discuss whether the use of current value accounting overcomes the problems of
historical cost accounting.
Current cost accounting attempts to provide more realistic book values by valuing assets at
current replacement cost, rather than the amount actually paid for them. This contrasts with the
usual historical cost approach. In fact, the current cost is usually calculated by adjusting the
historical cost for inflation.
The current operating profit is considered to be more relevant to many decisions such as dividend
distribution, employee wage claims and even as a basis for taxation.
The problems that current cost accounting (and other approaches to accounting for inflation)
attempt to solve are obviously linked to inflation. In practical terms, it can be very difficult to
determine the current value of assets. It is often subjective and complex.
3.6 ACCA SYLLABUS GUIDE OUTCOME 6:-
Describe the concept of financial and physical capital maintenance and how this affects
the determination of profits.
The concept of capital maintenance is concerned with how an entity defines the capital that it
seeks to maintain. It provides the linkage between the concepts of capital and the concepts of
profit. Profit is the residual amount that remains after expenses (including capital maintenance
adjustments, where appropriate) have been deducted from income. If expenses exceed income
the residual amount is a loss.
Financial capital maintenance
Under this concept a profit is earned only if the financial (or money) amount of the net assets at
the end of the period exceeds the financial (or money) amount of net assets at the beginning of
the period, after excluding any distributions to, and contributions from, owners during the period.
Financial capital maintenance can be measured in either nominal monetary units or units of
constant purchasing power.

!24 acowtancy.com

Downloaded by sajedul Islam (sajedulnsu15@gmail.com)


lOMoARcPSD|3181314

Physical capital maintenance


Under this concept, a profit is earned only if the physical productive capacity (or operating
capability) of the entity (or the resources or funds needed to achieve that capacity) at the end of
the period exceeds the physical productive capacity at the beginning of the period, after
excluding any distributions to, and contributions from, owners during the period.
Further Notes
Not-For-Profit and Public Sector Entities
Not-for-profit and public sector entities have different goals and objectives to profitmaking
organisations. They operate in different environments and are responsible to different
stakeholders. They have to account for funds received and show how they have been spent. They
need to focus on generating cash flows but are not expected to show a profit. E.g. include
government departments and agencies, higher education institutions, charitable bodies.
Phase G of the Conceptual Framework for Financial Reporting deals with the
Application to not-for-profit entities in the private and public sector. A set of International Public
Sector Accounting Standards (IPSASs) are being developed which closely match the IASs and
IFRSs. They are all based on the accrual concept although many not-forprofit organisations still
use cash accounting. However, some issues are debatable, e.g. the definition of a liability in
public entities: does the commitment to provide public benefits meets the definition of a
liability? Can the obligation be reliably measured when we are aware that some projects cost
much more that the amount actually budgeted?
Who are the users for not-for-profit entities?
The main users are the providers of funds, very often the taxpayers for government departments.
In the case of charities, it will be the different individuals/entities who are financially supporting
the charities.
How is performance measured?
The principle used is referred to a Value for Money (VFM) approach. It is made up of the 3 Es: -
1. effectiveness – the extent to which the organisation achieves its objectives
2. economy – the ability of the organisation to optimise the use of its productive resources
(keeping the cost of input resources as low as possible)
3. efficiency – the ‘output’ of the organisation per unit of resource consumed
Performance measurement is based on Key Performance Indicators (KPIs). E.g. % of roads
resurfaced, average spend on each student’s education, average waiting time for an operation.
Best Value is the term used by the United Kingdom's (UK) national government for its
performance management initiative
1. Challenging why, how and by whom a service is provided
2. Comparing performance against other local authorities.
3. Consulting the local community, users, etc
4. Using fair competition to secure efficient and effective services.
June 2010 Qs 3 (part)

!25 acowtancy.com

Downloaded by sajedul Islam (sajedulnsu15@gmail.com)


lOMoARcPSD|3181314

On a separate matter, you have been asked to advise on an application for a loan to build an
extension to a sports club which is a not-for-profit organisation. You have been provided with
the audited financial statements of the sports club for the last four years.
Required:
Identify and explain the ratios that you would calculate to assist in determining whether you
would advise that the loan should be granted. (5 marks)
Answer
1. To assess the gearing of the sports club: e.g. long-term loans compared to capital
employed (total assets less current liabilities)
2. To assess the repayment of loan + interest: e.g. excess of income over expenditure is
compared to interest payments – i.e. find the number of times this excess covers interest;
the higher the number of times, the lower the risk of defaulting
3. Other factors to be considered: the value of assets to be used as security; any ‘one-off’
donations; future prospects of the sports club.
Please also refer to page 6 of the notes
Current Value Accounting (CVA) (pls refer first to notes pgs 25 to 31)
As described in the notes, the problems of historical cost accounting arise in periods of inflation.
In order to offset some of these disadvantages, entities are now permitted to revalue non-current
assets (e.g. land and buildings). Financial assets and liabilities are carried at fair value (IFRS
13), i.e. “the price that would be received to sell an asset or paid to transfer a liability in an
orderly transaction between market participants at the measurement date”.
Under current value accounting, the original cost of an asset, e.g. receivables and payables,
would be replaced with its discounted present value (the present value of its future cash flows).
For non-current assets such as motor vehicles and machinery, the current cost can be its
replacement cost; for inventories, it will be either the current replacement cost or net realisable
value.
Two techniques which address the disadvantages of historical cost accounting are: - 1. Current
purchasing power (CPP)
2. Current cost accounting (CCA)
But before we look at these two techniques, how is profit measured? Profit can be measured as
the difference between how wealthy a company is at the beginning and end of an accounting
period.
Financial capital maintenance – profit is the increase in nominal money capital over the period.
This is used in CPP and historical cost.
Physical capital maintenance – profit is the increase in the physical productive capacity over the
period. This is used in CCA.
Current Purchasing Power (CPP)
CPP accounts for general inflation. The profit for the year in CPP terms is found by converting
sales, inventory, purchases and other expenses into year-end units of $CPP. In addition, a profit

!26 acowtancy.com

Downloaded by sajedul Islam (sajedulnsu15@gmail.com)


lOMoARcPSD|3181314

on holding net monetary liabilities, e.g. payables, or a loss on holding monetary assets, e.g. cash
and receivables, is calculated. E.g.
1.1.2011 – purchased 1000 units for $5 each
30.6.2011 – sold these 1000 units for $7 each
30.11.2011 – purchased 500 units for $6.50 each
The general price index is: -
1.1.2011 – 100
30.6.2011 – 120
30.11.2011 - 140
31.12.2011 – 150
Under historical cost accounting, gross profit is
Sales 7000
COS 5000 Profit 2000
Under CPP,
Sales would be (7000/120 x 150) = 8750
COS would be (5000/100 x 150) = 7500
There is also a loss of holding the cash for 6 months between July and December – 8750 – 7000
= 1750
There is a gain for owing creditors for one month
500 x 6.50 = 3250/140 x 150 = 3482 – 3250 = 232
Advantages of CPP
1. CPP method adopts the same unit of measurement by taking into account the price
changes.
2. Under CPP method, historical accounts continue to be maintained. CPP statements are
prepared on supplementary basis.
3. CPP method facilitates the calculation of gain or loss in purchasing power due to the
holding of monetary items (cash, receivables and payables).
4. CPP method uses common purchasing power as measuring unit. So, the comparative
study is easy.
5. CPP method provides reliable financial information for taking management decision to
formulate plans and policies.
6. CPP method ensures keeping intact the purchasing power of capital contributed by
shareholders. So, this method is of great importance from the point of view of the shareholders.
Disadvantages of CPP
1. CPP method considers only the changes in general purchasing power. It does not consider the
changes in the value of individual items.
2. CPP method is based on statistical index number which cannot be used in an individual firm.
3. It is very difficult to choose a suitable price index.
4. CPP method fails to remove all the defects of historical cost accounting system.

!27 acowtancy.com

Downloaded by sajedul Islam (sajedulnsu15@gmail.com)


lOMoARcPSD|3181314

Current Cost Accounting


CCA is based on the physical concept of capital maintenance. It makes adjustments to allow for
specific price movements. The value of assets consumed or sold and the value of assets in the
SFP should be stated at their deprival value, i.e. their value to the business. Deprival value can be
either the replacement cost, net realisable value or value in use (economic value). Example: -
Item A
Cost $10
Selling Price $15
Replacement cost $12
The current cost profit, when the item is sold, is: - Sales 15
RC 12 Profit 3
The difference of $2 ($12 - $10) is known as a holding gain. This holding gain is excluded from
profit.
Advantages of CCA
1. By excluding the holding gain from profit, CCA indicates whether the physical capital
maintenance has been maintained. Did the dividends paid to shareholders reduce the
operating capability of the entity?
2. It is more relevant to users and provides up-to-date information
3. The value is arrived at after considering the opportunity cost of holding the assets and the
expected benefits from their future use
Disadvantages of CCA
1. It is not objective – subjective judgement has to be used
2. It may be difficult to estimate replacement cost
3. Using the deprival value concept may not be meaningful. Some assets will be valued at
replacement cost, but others will be valued at NRV or value in use.
Further Questions
Question 1 21
Which of the following would be a change in accounting policy in accordance with IAS 8
Accounting Policies, Changes in Accounting Estimates and Errors?
A. Adjusting the financial statements of a subsidiary prior to consolidation as its
accounting policies differ from those of its parent
B. A change in reporting depreciation charges as cost of sales rather than as
administrative expenses

21 Specimen Exam Applicable from December 2014


!28 acowtancy.com

Downloaded by sajedul Islam (sajedulnsu15@gmail.com)


lOMoARcPSD|3181314

C. Depreciation charged on reducing balance method rather than straight line


D. Reducing the value of inventory from cost to net realisable value due to a valid
adjusting event after the reporting period

If the current cost measurement basis is used, assets are measured at:

A. Replacement cost
B. The amount paid to acquire them
C. The amount which could be obtained by selling them

In times of falling prices, the historical cost convention


Question 2
D. Present value
Question 3
A. Understates asset values and profits
B. Understates asset values and overstates profits
C. Overstates asset values and profits
D. Overstates asset values and understates profits
Question 4
Which of the following are advantages of historical cost accounting?
(1) It maintains financial and physical capital
(2) The statement of financial position shows the value of the business.
(3) Reported amounts are objectively verifiable.
(4) The profit concept is well understood.
A. 3 and 4
B. 1 and 2
C. 1 and 3
D. 2 and 4
Question 5
Gironimo Plc makes two changes to accounting practice at the end of 20X7:
(1) It changes the way in which it depreciates motor vehicles from 20% straight line to 25%
reducing balance.
(2) It starts to capitalise interest costs where allowed in accordance with the relevant
standard. Previously it had adopted a policy of writing off all interest costs to the statement of
comprehensive income.
What is the correct way to account for these two changes?
1 2
A. Do not adjust opening reserves Do not adjust opening reserves
B. Do not adjust opening reserves Adjust opening reserves
!29 acowtancy.com

Downloaded by sajedul Islam (sajedulnsu15@gmail.com)


lOMoARcPSD|3181314

C. Adjust opening reserves Adjust opening reserves


D. Adjust opening reserves Do not adjust opening reserves
Question 6
ABC Co was set up on 1 January 2001. In the first 3 years of operation, it recognised
development expenditure as an intangible asset in its accounts. During 2003, the managers of
the company decided that all development expenditure should be written off as incurred.
The following information is available: -
Development Exp Development Exp
Incurred and Capitalised Amortised to P&L
$’000 $’000
2001 500 - 2002
800 200 2003 1000
400
The 2003 accounts showed the following movements in retained earnings: -
Opening balance 1.1.2003 $9,000,000
Profits for the year $4,000,000
Closing balance 31.12.2003 $13,000,000
Profits for the year 2004 were $6,000,000 after charging the actual development expenditure to
the income statement.
Fill in the missing information in the following SOCIE extract for retained earnings:
Retained Earnings
2004 2003
’000 ’000
Balance b/fwd 13,000 9,000
Retrospective change
in accounting policy (b) (a)
Restated balance (d) (c)
Profit for the year (f) (e)
Balance c/fwd h g

CHAPTER 4:
IAS 16 – Property, Plant and Equipment (PPE)
4.1 ACCA SYLLABUS GUIDE OUTCOME 1:-
Define and compute the initial measurement of a non-current (including a
selfconstructed) asset.
IAS 16 defines property, plant and equipment (PPE) as tangible items that are:
• held for use in the production or supply of goods or services, for rental to others, or for
administrative purposes and
• expected to be used during more than one accounting period.

!30 acowtancy.com

Downloaded by sajedul Islam (sajedulnsu15@gmail.com)


lOMoARcPSD|3181314

An item of PPE should be recognized as an asset if and only if it is probable that future economic
benefits associated with the asset will flow to the entity and the cost of the item can be measured
reliably.
IAS 16 requires that PPE should initially be measured at cost.
Cost includes all costs necessary to bring the asset to working condition for its intended use.
Examples:
• Original purchase price
• Costs of site preparation
• Delivery and handling
• Installation
• Related professional fees for architects and engineers
• Estimated cost of dismantling and removing the asset and restoring the site
Where these costs are incurred over a period of time (such as employee benefits), the period for
which the costs can be included in the cost of PPE ends when the asset is ready for use, even if
the asset is not brought into use until a later date.
4.2 ACCA SYLLABUS GUIDE OUTCOME 2:-
Identify subsequent expenditure that may be capitalised (including borrowing costs),
distinguishing between capital and revenue items .22
4.2.1 IAS 23: Borrowing Costs23
IAS 23 requires that, where borrowing costs are directly incurred on a ‘qualifying asset’, they
must be capitalised as part of the cost of that asset.
A qualifying asset may be a tangible or an intangible asset that takes a substantial period of time
to get ready for its intended use or eventual sale. Property construction would be a typical
example, but it can also be applied to intangible assets during their development period.
Borrowing costs include interest based on its effective rate (which incorporates the amortisation
of discounts, premiums and certain expenses) on overdrafts, loans and (some) other financial
instruments and finance charges on finance leased assets.
If the entity has borrowed funds specifically to finance the construction of an asset, then the
amount to be capitalised is the actual finance costs incurred. Where the borrowings form part of
the general borrowing of the entity, then a capitalisation rate that represents the weighted average
borrowing rate of the entity should be used.
Capitalisation should commence when expenditure is being incurred on the asset, which is not
necessarily from the date funds are borrowed. Capitalisation should cease when the asset is ready
for its intended use, even though the funds may still be incurring borrowing costs. Also
capitalisation should be suspended if there is a suspension of active development of the asset.
Any borrowing costs that are not eligible for capitalisation must be expensed. Borrowing costs
cannot be capitalised for assets measured at fair value.

22 ACCA F7 Exam Dec 08 Q5


23 ACCA F7 Exam June 2010
!31 acowtancy.com

Downloaded by sajedul Islam (sajedulnsu15@gmail.com)


lOMoARcPSD|3181314

Illustration 1
On 1.1.2012, a company borrowed $1m at a rate of 10% p.a. to finance the building of a factory,
which is expected to take one year to build. All the $1m was drawn down on 1.1.2012 but only
$500,000 of this $1m was used immediately on 1.1.2012. The remaining $500,000 was utilised
on 1.7.2012. As a result, these $500,000 had been invested temporarily at a rate of 8% p.a.
Calculate the borrowing costs which may be capitalised and the cost of the asset as at
31.12.2012.

Borrowing Costs
1m x 10% = $100,000

Investment income
500,000 x 8% x 6/12 = $20,000

Net borrowing costs = 100 - 20 = $80,000

Cost of Asset:
Total expenditure $1m + borrowing costs $80,000 = $1080,000
4.2.2 Revenue Items
Revenue items refer to the normal short term day by day costs of operating the business – buying
materials/products, paying wages, and all the normal overheads like rent, stationery, fuel bills.
These are charged to the Statement of Profit or Loss and Other Comprehensive Income each
month as they are used up.
4.2.3 Capital Items
Capital items refer mainly to assets bought to use long term in the business – desks, computers,
productive equipment and machinery, vehicles, shop-fitting costs, and so on24.
Lecture Example 1
On 1 October 20X6, Omega began the construction of a new factory. Costs relating to the
factory, incurred in the year ended 30 September 20X7, are as follows:
$000
Purchase of the land 10,000
Costs of dismantling existing structures on the site 500
Purchase of materials to construct the factory 6,000
Employment costs (Note 1) 1,800
Production overheads directly related to the construction (Note 2) 1,200
Allocated general administrative overheads 600

24 ACCA F7 Exam Dec 07 Q2; Jun 09 Q5


!32 acowtancy.com

Downloaded by sajedul Islam (sajedulnsu15@gmail.com)


lOMoARcPSD|3181314

Architects’ and consultants’ fees directly related to the construction 400 Costs of
relocating staff who are to work at the new factory 300 Costs relating to the formal
opening of the factory 200
Interest on loan to partly finance the construction
of the factory (Note 3) 1,200
Note 1
The factory was constructed in the eight months ended 31 May 20X7. It was brought into use on
30 June 20X7. The employment costs are for the nine months to 30 June 20X7.
Note 2
The production overheads were incurred in the eight months ended 31 May 20X7. They included
an abnormal cost of $200,000, caused by the need to rectify damage resulting from a gas leak.
Note 3
Omega received the loan of $12m on 1 October 20X6. The loan carries a rate of interest of 10%
per annum.
Note 4
The factory has an expected useful economic life of 20 years. At that time the factory will be
demolished and the site returned to its original condition. This is a legal obligation that arose on
signing the contract to purchase the land. The expected costs of fulfilling this obligation are $2m.
An appropriate annual discount rate is 8%.
Requirement
Compute the initial carrying value of the factory. 25
Lecture Example 229
Which of the following items should be capitalised within the initial carrying amount of an item
of plant?
i) Cost of transporting the plant to the factory ii) Cost of installing a new
power supply required to operate the plant iii) A deduction to reflect the
estimated realisable value iv) Cost of a three-year maintenance agreement
v) Cost of a three-week training course for staff to operate the plant

25 Article “Property, Plant and Equipment”, P Robins, Student Accountant, June/July 2007
http://www.accaglobal.com/content/dam/acca/global/pdf/sa_aug07_robins.pdf
!33 acowtancy.com

Downloaded by sajedul Islam (sajedulnsu15@gmail.com)


lOMoARcPSD|3181314

ACCA SYLLABUS GUIDE OUTCOME 3:-


Discuss the requirements of relevant accounting standards in relation to the
revaluation of non-current assets.
30

After initial recognition, the asset should be measured using either the cost model or the

The cost model requires an asset, after initial recognition, to be carried at cost less
accumulated depreciation and impairment losses.

A. (i) and (ii) only


B. (i), (ii) and (iii)
C. (ii), (iii) and (iv)
D. (i), (iv) and (v) 4.3
revaluation model.
The revaluation model requires an asset, after initial recognition, to be measured at a revalued
amount, which is its fair value less subsequent depreciation and impairment losses.
A revaluation surplus (gain) should be credited to a revaluation surplus (reserve) whereas a
revaluation deficit (loss) should be expensed immediately (assuming, in both cases, no previous
revaluation of the asset has taken place). A surplus on one asset cannot be used to offset a deficit
on a different asset (even in the same class of asset).

29
Specimen Exam Applicable from December 2014
30
Examined June 2014 Qs 4a
4.3.1 How often should you revalue an asset?
Regularly, so that the carrying amount of an asset does not differ materially from its fair value at
the end of the reporting period. Where the carrying amount of the asset differs significantly from
its fair value, a (new) revaluation should be carried out. Even if there are no significant changes,
assets should still be subject to a revaluation every three to five years.
4.3.2 What needs to be revalued?
If an item is revalued, the entire class of assets to which that asset belongs should be revalued.
4.3.3 Do we depreciate revalued assets?

!34 acowtancy.com

Downloaded by sajedul Islam (sajedulnsu15@gmail.com)


lOMoARcPSD|3181314

Yes, revalued assets are depreciated in the same way as under the cost model. The asset should
be depreciated based on its revalued amount (less any residual value) over its estimated
remaining useful life, which should be reviewed annually irrespective of whether it has been
revalued.
An entity may choose to transfer annually an amount of the revaluation surplus relating to a
revalued asset to retained earnings corresponding to the ‘excess’ depreciation caused by an
upwards revaluation. Alternatively, it may transfer all of the relevant surplus at the time of the
asset’s disposal. 26
4.4 ACCA SYLLABUS GUIDE OUTCOME 4:-
Compute depreciation based on the cost and revaluation models and on assets that have
two or more significant parts (complex assets).
IAS 16 defines depreciation as ‘the systematic allocation of the depreciable amount of an asset
over its useful life’.
4.4.1 How much depreciation? (June 2009 Q5)
The depreciable amount (cost less prior depreciation, impairment, and residual value) should be
allocated on a systematic basis over the asset’s useful life.
4.4.2 How often should we review the residual value?
The residual value and the useful life of an asset should be reviewed at least at each financial
year-end. If either changes significantly, then that change should be accounted for over the
remaining estimated useful economic life.
4.4.3 Which depreciation method should be used?
A number of methods can be used to allocate depreciation to specific accounting periods. Two of
the more common methods, specifically mentioned in IAS 16, are the straight line method, and
the reducing (or diminishing) balance method.
The depreciation method used should reflect the pattern in which the asset’s economic benefits
are consumed by the enterprise.
4.4.4 How often should we review of this method is still right?
The method of depreciation should be reviewed at least annually.
4.4.5 Where does depreciation go?
Depreciation should be charged to the Statement of Profit or Loss, unless it is included in the
carrying amount of another asset. An example of this practice would be the possible inclusion of
depreciation in the costs incurred on a construction contract that are carried forward and matched
against future income from the contract, under the provisions of IFRS 15.
4.4.6 When do we start and finish depreciating an asset?
Depreciation begins when the asset is available for use and continues until the asset is
derecognised, either on disposal or when no future economic benefits are expected from the asset
(in other words, it is effectively scrapped). A gain or loss on disposal is recognised as the
difference between the disposal proceeds and the carrying value of the asset (using the cost or

26 examined June 2014 Qs 4a


!35 acowtancy.com

Downloaded by sajedul Islam (sajedulnsu15@gmail.com)


lOMoARcPSD|3181314

revaluation model) at the date of disposal. This net gain is included in the statement of profit or
loss – the sales proceeds should not be recognised as revenue.
4.4.7 Significant parts depreciated separately
If the cost model is used, each part of an item of property, plant, and equipment with a significant
cost (in relation to the total cost) must be depreciated separately.
4.4.8 Parts which are regularly replaced
The replacement cost is added to the asset cost when the recognition criteria (mentioned above)
are met.
The carrying amount of the replaced parts is derecognised.
4.4.9 Major Inspections for faults (e.g. aircraft)

If necessary, the estimated cost of a future similar inspection may be used as an


indication of what the cost of the existing inspection component was when the item was
acquired or constructed.

$63,000,000
$42,200,000
1 October 2008
Building $8,000,000
Plant and equipment $19,700,000

The inspection cost is added to the asset cost when recognition criteria (above) are met.
Lecture Example 3
Freehold property – at cost 1 October 2000
Plant and equipment – at cost
Accumulated depreciation
The freehold property has a land element of $ 13 million. The building element is being
depreciated on a straight – line basis.
Plant and equipment is depreciated at 40% per annum using the reducing balance method.
Required:
Calculate the depreciation expenses for the building and plant and equipment for the year ended
30 September 2009.

!36 acowtancy.com

Downloaded by sajedul Islam (sajedulnsu15@gmail.com)


lOMoARcPSD|3181314

(ACCA F7 Dec 2009 QS 2 Part)


4.5 ACCA SYLLABUS GUIDE OUTCOME 5:-
Account for revaluation and disposal gains and losses for non-current assets.
4.5.1 Normal Rule
When an asset is revalued, any increase in the carrying amount should be credited to equity
under the heading “revaluation surplus”.
Accounting treatment:
(1) Adjust cost account to revalued amount.
(2) Remove accumulated depreciation charged on the asset to date.
(3) Put the balance to the revaluation reserve.
The required double-entry is:
Dr Non-current asset cost
Dr Accumulated Depreciation
Cr Revaluation Reserve
Lecture Example 427
A property was purchased on 1 January 20X0 for $2m (estimated depreciable amount $1m –
useful economic life 50 years). Annual depreciation of $20,000 was charged from 20X0 to 20X4
inclusive and on 1 January 20X5 the carrying value of the property was $1.9m. The property was
revalued to $2.8m on 1 January 20X5 (estimated depreciable amount $1.35m – the estimated
useful economic life was unchanged).
Show the treatment of the revaluation surplus and compute the revised annual depreciation
charge.
4.5.2 Revaluation downwards
Any reduction in value arising from a revaluation should:-
First:- be debited to any revaluation surplus relating to the same asset
Then:- treat the remainder as an expense in the statement of profit or loss
Lecture Example 528
The property referred to in Lecture Example 4 was revalued on 31 December 20X6. Its fair value
had fallen to $1.5m.
Compute the revaluation loss and state how it should be treated in the financial statements.
4.5.3 What happens to the revaluation surplus on disposal of the asset revalued?
Where assets are measured using the revaluation model, any remaining balance in the revaluation
surplus relating to the asset disposed of, is transferred directly to retained earnings. No recycling
of this balance into the statement of profit or loss is permitted.
4.5.4 Accounting for disposals of non-current assets

27 Article, “Property, Plant and Equipment and Tangible Assets”, by P. Robins, Student Accountant, August 2007
http://www.accaglobal.com/content/dam/acca/global/pdf/sa_aug07_robins.pdf
28 Article, “Property, Plant and Equipment and Tangible Assets”, by P. Robins, Student Accountant, August 2007
http://www.accaglobal.com/content/dam/acca/global/pdf/sa_aug07_robins.pdf
!37 acowtancy.com

Downloaded by sajedul Islam (sajedulnsu15@gmail.com)


lOMoARcPSD|3181314

When a non-current asset is sold, there is likely to be a profit or loss on disposal. This is the
difference between the net sale price of the asset and its net book value at the time of disposal.
If:
Sales proceeds > NBV → profit on disposal
Sales proceeds < NBV → loss on disposal
Accounting Treatment:-
(1) Remove the cost of the asset:
Dr Disposal account
Cr Non-current asset
(2) Remove the accumulated depreciation charged to date:
Dr Accumulated depreciation
Cr Disposal account
(3) Account for the sales proceeds:
Dr Cash
Cr Disposal account
(4) Balance off disposal account to find the profit or loss on disposal.
A profit on disposal is shown in the statement of profit or loss as sundry income, a loss as an
expense in the statement of profit or loss.
4.6 Change in the useful life of an asset
The useful life of an item of property, plant and equipment should be reviewed at least every
financial year-end and, if expectations are significantly different from previous estimates, the
depreciation charge for current and future periods should be revised.
This is achieved by writing the net book value off over the asset's revised remaining useful life.
NBV – residual value
Revised useful life
4.7 Change in method of depreciation
It is up to the business to decide which method of depreciation to apply to its noncurrent assets.
The chosen method of depreciation should be applied consistently from year to year. This is an
instance of the fundamental accounting assumption of consistency.
The depreciation method has to be reviewed. If there are any changes in the expected pattern of
use of the asset, then the method used should be changed. In such cases, the remaining net book
value is depreciated under the new method, i.e. only current and future periods are affected. The
change is prospective.
Further Questions 134
Question 1
On 1 March 2008 Yucca acquired a machine from Plant under the following terms:
$
List price of machine 82,000

!38 acowtancy.com

Downloaded by sajedul Islam (sajedulnsu15@gmail.com)


lOMoARcPSD|3181314

34
Article: “Accounting for Property, Plant and Equipment”, B.A. Retallack, Student Accountant
2010, http://www.accaglobal.com/content/dam/acca/global/PDF-students/2012/
sa_sept10_ias16.pdf
Import duty 1,500
Delivery fees 2,050
Electrical installation costs 9,500
Pre-production testing 4,900 Purchase
of a five-year maintenance contract with Plant 7,000
In addition to the above information Yucca was granted a trade discount of 10% on the initial list
price of the asset and a settlement discount of 5% if payment for the machine was received
within one month of purchase. Yucca paid for the plant on 25 March 2008.
How should the above information be accounted for in the financial statements?
Question 2
Construction of Deb and Ham’s new store began on 1 April 2009. The following costs were
incurred on the construction:
$000
Freehold land 4,500 Architect fees 620 Site
preparation 1,650
Materials 7,800
Direct labour costs 11,200
Legal fees 2,400
General overheads 940
The store was completed on 1 January 2010 and brought into use following its grand opening on
the 1 April 2010. Deb and Ham issued a $25m unsecured loan on 1 April 2009 to aid
construction of the new store (which meets the definition of a qualifying asset per IAS 23). The
loan carried an interest rate of 8% per annum and is repayable on 1 April 2012.
Required
Calculate the amount to be included as property, plant and equipment in respect of the new store
and state what impact the above information would have on the statement of profit or loss (if
any) for the year ended 31 March 2010.
Question 3
On 1 March 2010 Yucca purchased an upgrade package from Plant at a cost of $18,000 for the
machine it originally purchased in 2008 (Question 1). The upgrade took a total of two days
where new components were added to the machine. Yucca agreed to purchase the package as the
new components would lead to a reduction in production time per unit of 15%. This will enable
Yucca to increase production without the need to purchase a new machine.
Should the additional expenditure be capitalised or expensed?
Question 4

!39 acowtancy.com

Downloaded by sajedul Islam (sajedulnsu15@gmail.com)


lOMoARcPSD|3181314

An item of plant was purchased on 1 April 2008 for $200,000 and is being depreciated at 25% on
a reducing balance basis.
Prepare the extracts of the financial statements for the year ended 31 March 2010.
Question 5
A machine was purchased on 1 April 2007 for $120,000. It was estimated that the asset had a
residual value of $20,000 and a useful economic life of 10 years at this date.
On 1 April 2009 (two years later) the residual value was reassessed as being only $15,000 and
the useful economic life remaining was considered to be only five years.
How should the asset be accounted for in the years ending 31 March 2008/2009/2010?
Question 6
A company purchased a property with an overall cost of $100m on 1 April 2009.
The property elements are made up as follows:
$000 Land Estimated life
and buildings
(Land element $20,000) 65,000 50 years
Fixtures and fittings 24,000 10 years
Lifts 11,000 20 years
100,000
Calculate the annual depreciation charge for the property for the year ended 31 March 2010.
Question 7
A company purchased a building on 1 April 2007 for $100,000. The asset had a useful economic
life at that date of 40 years. On 1 April 2009 the company revalued the building to its current fair
value of $120,000.
What is the double entry to record the revaluation?
Question 8
The carrying value of Zen’s property at the end of the year amounted to $108,000. On this date
the property was revalued and was deemed to have a fair value of $95,000. The balance on the
revaluation reserve relating to the original gain of the property was $10,000.
What is the double entry to record the revaluation?
Question 9

A company revalued its property on 1 April 2009 to $20m ($8m for the land). The property
originally cost $10m ($2m for the land) 10 years ago. The original useful economic life of 40
years is unchanged. The company’s policy is to make a transfer to realised profits in respect of
excess depreciation.
How will the property be accounted for in the year ended 31 March 2010?
Question 10

!40 acowtancy.com

Downloaded by sajedul Islam (sajedulnsu15@gmail.com)


lOMoARcPSD|3181314

A company purchased a building on 1 April 2005 for $100,000 at which point it was considered
to have a useful economic life of 40 years. At the year end 31 March 2010 the company decided
to revalue the building to its current value of $98,000.
How will the building be accounted for in the year ended 31 March 2010?
Question 11
At 1 April 2009 HD Ltd carried its office block in its financial statements at its original cost of
$2 million less depreciation of $400,000 (based on its original life of 50 years). HD Ltd decided
to revalue the office block on 1 October 2009 to its current value of $2.2m. The useful economic
life remaining was reassessed at the time of valuation and is considered to be 40 years at this
date. It is the company’s policy to charge depreciation proportionally.
How will the office block be accounted for in the year ended 31 March 2010? Question 12
An asset that originally cost $16,000 and had accumulated depreciation on it of $8,000 was
disposed of during the year for $5,000 cash.
How should the disposal be accounted for in the financial statements?
Further Questions 2
Question 1
The following information is available for the year ended 31 October 2012:
$
Property
Cost as at 1 November 2011 102,000
Accumulated depreciation as at 1 November 2011 (20,400) Carrying Value
81,600
On 1 November 2011, the company revalued the property to $150,000.
The company’s policy is to charge depreciation on a straight-line basis over 50 years. On
revaluation there was no change to the overall useful economic life. It has also chosen not to
make an annual transfer of the excess depreciation on revaluation between the revaluation
reserve and retained earnings.

What should be the balance on the revaluation reserve and the depreciation charge as shown in
the financial statements for the year ended 31 October 2012?

Depreciation charge Revaluation reserve


$ $
A. 3,750 68,400
B. 3,750 48,000
C. 3,000 68,400
D. 3,000 48,000
CHAPTER 5:
IAS 20 – GOVERNMENT GRANTS

!41 acowtancy.com

Downloaded by sajedul Islam (sajedulnsu15@gmail.com)


lOMoARcPSD|3181314

5.1 ACCA SYLLABUS GUIDE OUTCOME 1:-


Apply the provisions of relevant accounting standards in relation to accounting for
government grants.
IAS 20 applies to all government grants and other forms of assistance by a government aimed at
providing an economic benefit to an entity or group of entities qualifying under certain criteria.
This government assistance can be of many types, including grants, forgivable loans, and indirect
or non-monetary form of assistance, such as technical advice.
5.1.1 When can government grants be brought into the accounts?
Government grants should only be recognized when there is reasonable assurance that:
• the enterprise will comply with any conditions attached to the grant and • the grant will
be received.
5.1.2 Where does it go in the accounts?
The grant is recognised as income over the period necessary to match them with the related costs,
for which they are intended to compensate, on a systematic basis.
5.1.3 Capital Grants
Non-monetary grants, such as a plot of land or a building in a remote area, (or money towards the
purchase of non-current assets) are usually accounted for at fair value.
They should be presented in the SFP in either of two ways:-
• as deferred income (a liability) and transfer a portion to revenue each year, or
• by deducting the grant in arriving at the asset’s carrying amount. Depreciate the asset on
the reduced cost.
When the grant is treated as deferred income, it is released to the statement of profit or loss over
the useful economic life of the asset.
5.1.4 Revenue Grants e.g. a grant for wages incurred
A grant relating to income may be reported separately as ‘other income’ (credit in the statement
of profit or loss) or deducted from the related expense.
5.1.5 Repayment of government grants
When a government grant becomes repayable (e.g. due to non-fulfillment of the terms of the
grant), it should be treated as a change in estimate under IAS 8 and accounted for prospectively.
Where the original grant related to income, the repayment should be dealt with as an expense.
Where the original grant related to an asset, the repayment should be treated as increasing the
carrying amount of the asset or reducing the deferred income balance.
The cumulative depreciation which would have been charged had the grant not been received
should be charged as an expense.
Lecture Example 1
The following is an extract of Errsea’s balances of property, plant and equipment and related
government grants at 1 April 2006.
Accumulated Carrying
Cost Depreciation Amount

!42 acowtancy.com

Downloaded by sajedul Islam (sajedulnsu15@gmail.com)


lOMoARcPSD|3181314

$’000 $’000 $’000


240 180 60
Property, plant and equipment
Non-current liabilities
Government grants 30
Current liabilities

Government grants 10
Details including purchases and disposals of plant and related government grants during the year
are:
(i) Included in the above figures is an item of plant that was disposed of on 1 April 2006 for
$12,000 which had cost $90,000 on 1 April 2003. The plant was being depreciated on a
straight-line basis over four years assuming a residual value of $10,000. A government
grant was received on its purchase and was being recognised in the statement of profit or
loss in equal amounts over four years. In accordance with the terms of the grant, Errsea
repaid $3,000 of the grant on the disposal of the related plant.
(ii) An item of plant was acquired on 1 July 2006 with the following costs:
$
Base cost 192,000
Modifications specified by Errsea 12,000
Transport and installation 6,000
The plant qualified for a government grant of 25% of the base cost of the plant, but it had
not been received by 31 March 2007. The plant is to be depreciated on a straight-line
basis over three years with a nil estimated residual value.
(iii)All other plant is depreciated by 15% per annum on cost.
(iv)$11,000 of the $30,000 non-current liability for government grants at 1 April 2006 should
be reclassified as a current liability as at 31 March 2007.
(v) Depreciation is calculated on a time apportioned basis.
Required:-
Prepare extracts of Errsea’s statement of profit or loss and statement of financial position
in respect of the property, plant and equipment and government grants for the year ended
31 March 2007.
Note: Disclosure notes are not required.

(10 marks)

(ACCA Paper 2.5 June 2007 Qs 5a)


Further Questions
Please indicate which you think is closest to the correct answer.

!43 acowtancy.com

Downloaded by sajedul Islam (sajedulnsu15@gmail.com)


lOMoARcPSD|3181314

Question 1
Government assistance includes:
A. Direct action to provide economic benefits to qualifying firms.
B. Imposing import tariffs.
C. Indirect help, such as improving local infrastructure.
Question 2
IAS 20 deals with:
A. Disclosure of government grants.
B. Government participation in the ownership of firms.
C. Tax benefits provided to a firm in relation to Government Grants.
Question 3
The impact of government assistance on financial statements is which of the following:
A. Financial Statements must be able to reflect the receipt of government assistance.
B. Financial statements must ignore all assistance.
C. Financial statements must show only 10% of total assistance.
Question 4
Which of the following may be categorized as purposes of government assistance? Tick all
that apply.
1. Reduce unemployment by subsidizing jobs and training.
2. Boosting capital by investing in specified assets.
3. Try to promote economic activity in specific regions
A. 1 and 2
B. 1,2 and 3
C. 2 and 3
Question 5
If a firm does not comply with the conditions of a government loan, then this may result in the
need:
1. To repay the loan.
2. To account for the loan on a cash basis only.
3. To record a contingent liability in the future.
A. 1,2 and 3
B. 1 and 2

!44 acowtancy.com

Downloaded by sajedul Islam (sajedulnsu15@gmail.com)


lOMoARcPSD|3181314

A qualifying firm, it may receive grants related to the assets from the government, when

A.Buys, builds, or acquires long-term assets.


B.Buys long-term assets.

A.Transactions with the government in the normal course of trade.


B.A transfer of resources to qualifying firms.
C. Provision of guarantees by the government.

C. 2 and 3
Question 6
it:
C. Acquires long-term assets.
D. Builds long-term assets.

Question 7
Government grants are defined as:
Question 8
If the grants are intended to compensate certain costs, then they should be:
A. Recognised as income over the periods when the related costs are incurred.
B. Only entered in the books when those costs are incurred.
C. Ignored.
Question 9
If grants relate to depreciable assets:
A. They should not be recognised at all since the asset will have no value eventually.
B. Recognised as income in the periods in which the depreciation is charged.
C. Credited immediately to other income.
Question 10

!45 acowtancy.com

Downloaded by sajedul Islam (sajedulnsu15@gmail.com)


lOMoARcPSD|3181314

Government grants may be given in more than one form. They may be given as:

1. Fo
rg
iv
ab
le

The recognition of government grants should only be made if:

1. The grants will never be repaid under any circumstances.


2. It is likely that the firm will comply with the qualifying conditions.
3. The grants will be received.
loans.
2. Grants related to income.
3. Grants related to assets.
A. 2 and 3 only
B. 1,2 and 3
C. 1 only
Question 11
A. 1 and 2
B. 1,2 and 3
C. 2 and 3

Question 12
On the notification that a firm will receive a grant:
A. An account receivable will be set up, but the grant will be recorded on a cash basis.
B. An account receivable will be set up, and the grant will be recorded on an accruals basis.
C. Do nothing until the cash arrives.
CHAPTER 6:
IAS 40 – INVESTMENT PROPERTIES
6.1 ACCA SYLLABUS GUIDE OUTCOME 1:-
Discuss why the treatment of investment properties should differ from other properties.
Investment property is property (land or a building or part of a building or both) held by the
owner or by the lessee under a finance lease to earn rentals or for capital appreciation or both.
An investment property may include property under an operating lease, if used for the same
purpose as other investment properties.

!46 acowtancy.com

Downloaded by sajedul Islam (sajedulnsu15@gmail.com)


lOMoARcPSD|3181314

Generally, non-investment properties generate cash flows in combination with other assets. A
property that meets the definition of an investment property will generate cash flows that are
largely independent of the other assets held by an entity29.
6.2 ACCA SYLLABUS GUIDE OUTCOME 2:-
Apply the requirements of relevant accounting standards for investment property.
Recognition
Investment property should be recognised as an asset when
• it is probable that the future economic benefits that are associated with the property will
flow to the enterprise, and
• the cost of the property can be reliably measured.
Initial Measurement
Investment property is initially measured at cost, including transaction costs.
Measurement subsequent to initial recognition
IAS 40 permits enterprises to choose between:
a. a fair value model; and
b. a cost model.
a. Fair value model
Under the fair value model:
• the asset is revalued to fair value at the end of each year • the gain or loss is
shown directly in the statement of profit or loss
• no depreciation is charged on the asset.
As discussed in Chapter 3, the measurement of the fair value of an investment property is likely
to require reference to current market rents for similar properties and market interest rates . This
may need to be adjusted to reflect the location and condition of the 30property and other factors
that market participants would take into account.
b. Cost model
Under the cost model the asset should be accounted for in line with the cost model laid out in
IAS 16.
• The property will be shown in the statement of financial position at cost less accumulated
depreciation.
One method must be adopted for all of an entity’s investment property.
Change is permitted only if this results in a more appropriate presentation. IAS 40 notes that it is
highly unlikely for a change from a fair value model to a cost model.
Gains or losses arising from changes in the fair value of investment property must be included in
net profit or loss for the period in which it arises
Transfers to or from investment property
Transfers to or from investment property should only be made when there is a change in use.

29 June 2013 Qs 5
30 An income approach with level 2 inputs as per IFRS 13
!47 acowtancy.com

Downloaded by sajedul Islam (sajedulnsu15@gmail.com)


lOMoARcPSD|3181314

When there is a transfer from investment property carried at fair value to owneroccupied
property or inventories, the property’s cost for subsequent accounting should be its fair value at
the date of change of use.
Where an owner-occupied property becomes an investment property, an entity should apply IAS
16 up to the date of change of use. Any difference at that date between the carrying amount of
the property under IAS 16 and its fair value is a revaluation.
Intra-company rentals
Property rented to a parent, subsidiary or fellow subsidiary is not investment property in
consolidated financial statements. The property is owner-occupied from the perspective of the
group. However, such property can qualify as investment property in the separate financial
statements of the lessor, if the definition of investment property is otherwise met.
Partial own use
If the owner uses part of the property for its own use and part to earn rentals or for capital
appreciation, and the portions can be sold or leased out separately, they are accounted for
separately. If the portions cannot be sold or leased out separately, the property is investment
property only if the owner-occupied portion is insignificant.
Disposal of Investment Property
An investment property should be derecognised on disposal. The gain or loss on disposal should
be calculated as the difference between the net disposal proceeds and the carrying amount of the
asset and should be recognised as income or expense in the statement of profit or loss.
Lecture Example 1
ABC Ltd, a manufacturing company, purchases a property for $1 million on 1 January 20X1 for
its investment potential. The land element of the cost is believed to be $400,000, and the
buildings element is expected to have a useful life of 50 years. At 31 December 20X1, local
property indices suggest that the fair value of the property has risen to $1.1 million.
Show how the property would be presented in the financial statements as at 31 December
20X1 if ABC Ltd adopts:
(a) the cost model
(b) the fair value model.
Lecture example 231
Speculator owns a number of properties. An independent surveyor has assessed their market
values as
Property Cost Valuation Valuation
01-Jul-99 30-Jun-00 30-Jun-01

31 Article “IAS 40 Investment Properties”, Student Accountant, Steve Scott, July 2001 (Amended)
http://www2.accaglobal.com/students/student_accountantx/archive/2001/18/57482?
session=fffffffeffffffff0a0121395395e0a2d32b3ca9cbf1f89ffbc61cf2cf55d5f8
!48 acowtancy.com

Downloaded by sajedul Islam (sajedulnsu15@gmail.com)


lOMoARcPSD|3181314

$ $ $

A 41,000 52,000 73,000

76,000 82,000 66,000

117,000 134,000 139,000

Both properties had an estimated life of 50 year when they were acquired. They are
both let on short leases under commercial terms. The policy is to adopt the fair value
model in IAS 40 for investment properties.

How should properties A and B be recorded in the financial statements of Speculator?

B
Total
Required:

CHAPTER 7:
IAS 38 – INTANGIBLE ASSETS
7.1 ACCA SYLLABUS GUIDE OUTCOME 1:-
Discuss the nature of internally generated and purchased intangibles.
An intangible asset is an identifiable non-monetary asset without physical substance. An asset is
a resource that is controlled by the entity as a result of past events (for example, purchase or self-
creation) and from which future economic benefits (inflows of cash or other assets) are expected.
Therefore, the three critical attributes of an intangible asset are:
• identifiability (capable of being separated and sold or transferred)
• control (power to obtain benefits from the asset)
• future economic benefits (such as revenues or reduced future costs)
Examples of possible intangible assets include:
• computer software
• patents
• copyrights
• motion picture films
• customer lists
• franchises
Intangibles can be acquired:

!49 acowtancy.com

Downloaded by sajedul Islam (sajedulnsu15@gmail.com)


lOMoARcPSD|3181314

• by separate purchase
• as part of a business combination
• by a government grant
• by exchange of assets
• by self-creation (internal generation)
7.2 ACCA SYLLABUS GUIDE OUTCOME 2:-
Distinguish between goodwill and other intangible assets.
Goodwill is calculated at the acquisition date as:
$
Fair value of consideration paid (shares issued plus cash paid plus
direct costs) X
Non-controlling interest
(valued either at fair value or as a proportion of net assets) X
X
Fair value of net assets of subsidiary acquired (X) X
Goodwill may exist because of any combination of a number of possible factors:
• reputation for quality or service
• technical expertise
• possession of favourable contracts
• good management and staff.
Negative goodwill
If the difference above is negative, the resulting gain is recognised as a bargain purchase in the
statement of profit or loss.
What are the main characteristics of goodwill which distinguish it from other intangible
assets?
• It is a ‘balancing figure’. Goodwill itself is not valued but a comparison is made between
the fait value of the whole business and the fair value of the separable net assets of the
business. It cannot be valued on its own.
• Goodwill cannot be disposed of as a separate asset.
• The factors contributing to the value of goodwill cannot be valued, e.g. how can one
value the benefit of an experienced workforce?
• The value of goodwill is volatile – it can only be given a numerical value at the time of
acquisition of the whole business.
7.3 ACCA SYLLABUS GUIDE OUTCOME 3:-
Describe the criteria for the initial recognition and measurement of intangible assets.
IAS 38 requires an enterprise to recognise an intangible only if:
• it is probable that the future economic benefits will flow to the enterprise; and
• the cost of the asset can be measured reliably.
If an intangible item does not meet these criteria, it should be written off as an expense.

!50 acowtancy.com

Downloaded by sajedul Islam (sajedulnsu15@gmail.com)


lOMoARcPSD|3181314

How are intangible assets initially measured?


Intangible assets are initially measured at cost.
7.4 ACCA SYLLABUS GUIDE OUTCOME 4:-
Describe the subsequent accounting treatment, including the principle of impairment
tests in relation to goodwill.
How are intangible assets measured at each year end?
After initial recognition an entity must choose either the cost model or the revaluation model for
each class of intangible asset.
Cost model
After initial recognition, intangible assets should be carried at cost less any amortisation and
impairment losses.
Revaluation model
Intangible assets may be carried at a revalued amount (based on fair value) less any subsequent
amortisation and impairment losses.
The revaluation model is only used if fair value can be determined by reference to an active
market (however, such active markets are expected to be uncommon for intangible assets). In
fact, the fair values of customer contracts, brands or patents cannot be measured using a market
approach as per IFRS 13. Very often, an income approach is used but this is not accepted under
IAS 38.
Should intangible assets be amortised?
An intangible asset with a finite useful life must be amortised over that life, normally using the
straight line method with a zero residual value
What if intangible assets have an indefinite useful life?
If an intangible asset has an indefinite useful life: -
• it should not be amortised
• its useful life should be reviewed at each reporting period.
What about goodwill?
Goodwill is not amortised; it should be tested for impairment annually. An impairment loss is the
amount by which the carrying amount of an asset exceeds its recoverable amount.
7.5 ACCA SYLLABUS GUIDE OUTCOME 5:-
Describe and apply the requirements of relevant accounting standards to research and
development expenditure.
Research is original and planned investigation undertaken with the prospect of gaining new
scientific or technical knowledge and understanding.
Development is the application of research findings or other knowledge to a plan or design for
the production of new or substantially improved materials, devices, products, processes, systems,
or services, before the start of commercial production or use.
Accounting Treatment of Research and Development
Research phase

!51 acowtancy.com

Downloaded by sajedul Islam (sajedulnsu15@gmail.com)


lOMoARcPSD|3181314

IAS 38 states that all expenditure incurred at the research stage should be written off to the
statement of profit or loss as an expense when incurred, and will never be capitalised as an
intangible asset.
Development phase
Under IAS 38, an intangible asset must demonstrate all of the following criteria:
▪ Probable future economic benefits
▪ Intention to complete and use or sell the asset
▪ Resources (technical, financial and other resources) are adequate and available to complete
and use the asset
▪ Ability to use or sell the asset
▪ Technical feasibility of completing the intangible asset (so that it will be available for use
or sale)
▪ Expenditure can be measured reliably
If any of the recognition criteria are not met then the expenditure must be charged to the
statement of profit or loss as incurred.
Note that if all the recognition criteria have been met, capitalisation must take place: Treatment
of Capitalised Development Costs
Once development costs have been capitalised, the asset should be amortised in accordance with
the accruals concept over its finite life.
Amortisation must only begin when commercial production has commenced (hence matching the
income and expenditure to the period in which it relates). It is an expense in the statement of
profit or loss.
Each development project must be reviewed at the end of each accounting period to ensure that
the recognition criteria are still met. If the criteria are no longer met, then the previously
capitalised costs must be written off to the statement of profit or loss immediately.
What if the research / development split is unclear?
The enterprise treats the expenditure for that project as if it were incurred in the research phase
only.

Lecture Example 1
How would you account for these 2 scenarios ?
Scenario 1
The following trial balance (extract) relates to Candel at 30 September 2008:
$’000
Capitalised development expenditure – at 1 October 2007 20,000
Development expenditure –
accumulated amortisation at 1 October 2007 6,000
Non-current assets – intangible:

!52 acowtancy.com

Downloaded by sajedul Islam (sajedulnsu15@gmail.com)


lOMoARcPSD|3181314

In addition to the capitalised development expenditure (of $20 million), further research and
development costs were incurred on a new project which commenced on 1 October 2007. The
research stage of the new project lasted until 31 December 2007 and incurred $1.4 million of
costs. From that date the project incurred development costs of $800,000 per month. On 1 April
2008 the directors became confident that the project would be successful and yield a profit well
in excess of its costs. The project is still in development at 30 September 2008.
Capitalised development expenditure is amortised at 20% per annum using the straightline
method. All expensed research and development is charged to cost of sales.
ACCA F7 December 2008 Qs 2(ii)
Scenario 2
Trial Balance extract as at 30 June 2015 :
Research and development costs $7.8m
Moston commenced a research and deveopment project on 1 January 2015. It spent $1m per
month on research until 31 March 2015, at which date the project passed into the development
stage. From this date, it spent $1.6m per month until the year end (30 June 2015), at which date
development was completed. However, i twas not until 1 May 2015 that the directors of Moston
were confident that the new product would be a commercial success.
ACCA F7 Sept/Dec 2015 Qs 1 part (ii)
Further Questions32
Question 1
Dempsey’s year end is 30 September 2014. Dempsey commenced the development stage of a
project to produce a new pharmaceutical drug on 1 January 2014.
Expenditure of $40,000 per month was incurred until the project was completed on 30 June 2014
when the drug went into immediate production. The directors became confident of the project’s
success on 1 March 2014. The drug has an estimated life span of five years; time apportionment
is used by Dempsey where applicable.

32 Specimen Exam Applicable from December 2014


!53 acowtancy.com

Downloaded by sajedul Islam (sajedulnsu15@gmail.com)


lOMoARcPSD|3181314

What amount will Dempsey charge to profit or loss for development costs, including any
amortisation, for the year ended 30 September 2014?

A. $12,000
B. $98,667
C. $48,000
D. $88,000

!54 acowtancy.com

Downloaded by sajedul Islam (sajedulnsu15@gmail.com)


lOMoARcPSD|3181314

CHAPTER 8:
IAS 36 – IMPAIRMENT OF ASSETS
8.1 ACCA SYLLABUS GUIDE OUTCOME 1:-
Define an impairment loss.
An impairment loss is the amount by which the carrying amount of an asset exceeds its

The amount at which an asset is recognised in the SFP after deducting accumulated
depreciation and accumulated impairment losses.

The higher of an asset’s fair value less costs to sell and its value in use

The amount obtainable from the sale of an asset in a bargained transaction between

The discounted present value of estimated future cash flows expected to arise from:
recoverable amount.
Carrying amount
Recoverable amount
Fair value
knowledgeable, willing parties.
Value in use
• the continuing use of an asset, and from • its
disposal at the end of its useful life.
8.2 ACCA SYLLABUS GUIDE OUTCOME 2:-
Identify the circumstances that may indicate impairments to assets.
At the end of each reporting period, review all assets to look for any indication that an asset may
be impaired.
If there is an indication that an asset may be impaired, then you must calculate the asset’s
recoverable amount.
Indications of Impairment
1. Losses / worse economic performance
2. Market value declines

!55 acowtancy.com

Downloaded by sajedul Islam (sajedulnsu15@gmail.com)


lOMoARcPSD|3181314

3. Obsolescence or physical damage


4. Changes in technology, markets, economy, or laws
5. Increases in market interest rates
6. Loss of key employees

an intangible asset with an indefinite useful life.


an intangible asset not yet available for use.
goodwill acquired in a business combination.

If fair value less costs to sell cannot be determined, then the recoverable amount is the

For assets to be disposed of, the recoverable amount is fair value less costs to sell.

If there is a binding sale agreement, use the price under that agreement less costs of

7. Restructuring / re-organisation
Always check for impairment of:
••

Determining Recoverable Amount
value in use.
Fair Value Less Costs to Sell
disposal.
If there is an active market for that type of asset, use market price less costs of disposal.
The market price means current bid price if available, otherwise the price in the most recent
transaction.
If there is no active market, use the best estimate of the asset’s selling price less costs of
disposal.
Costs of Disposal
The direct added costs only (not existing costs or overhead)
Value in Use
The calculation of value in use should reflect the following elements:

!56 acowtancy.com

Downloaded by sajedul Islam (sajedulnsu15@gmail.com)


lOMoARcPSD|3181314

• an estimate of the future cash flows the entity expects to derive from the asset in an arm’s
length transaction;
• expectations about possible variations in the amount or timing of those future cash flows;
• the time value of money, represented by the current market risk-free rate of interest;
• the price for bearing the uncertainty inherent in the asset.
Cash Flow Projections
• based on reasonable and supportable assumptions, the most recent budgets and forecasts,
and extrapolation for periods beyond budgeted projections.
• IAS 36 presumes that budgets and forecasts should not go beyond five years; for
periods after five years, extrapolate from the earlier budgets.
• should relate to the asset in its current condition – future restructurings to which the
entity is not committed and expenditures to improve or enhance the asset’s performance
should not be anticipated.
• should not include cash from financing activities, or income tax.
What happens if the recoverable amount is higher than the carrying value in the SFP?
The unit and the goodwill allocated to that unit is not impaired.
What happens if the recoverable amount is lower than the carrying value in the SFP?
The entity must recognise an impairment loss.
The impairment loss is allocated to the CGU in the following order:
1. Reduce the carrying amount of any goodwill allocated to the cash-generating unit
2. Reduce the carrying amounts of the other assets of the unit pro rata
The carrying amount of an asset should not be reduced below the highest of:
• its fair value less costs to sell
• its value in use
.
Lecture Example 133
A company has an asset that has a carrying value of $800. The asset has not been revalued. The
asset is subject to an impairment review. If the asset was sold then it would sell for $610 and
there would be associated selling costs of $10.
The estimate of the present value of the future cash flows to be generated by the asset if it were
kept is $750.
Required:-
Determine the outcome of the impairment review.
8.3 ACCA SYLLABUS GUIDE OUTCOME 3:-
Describe what is meant by a cash generating unit (CGU)34.

33 Article “Impairment of Goodwill”, Student Accountant, T Clendon and S Baker, August 2009.
http://www.accaglobal.eu/pubs/students/publications/student_accountant/archive/sa_aug09_clendon_baker2.pdf
34 ACCA F7 Exam June 2012 Qs 4
!57 acowtancy.com

Downloaded by sajedul Islam (sajedulnsu15@gmail.com)


lOMoARcPSD|3181314

The CGU is the smallest identifiable group of assets that generates cash inflows which are
(largely) independent of other asssets..
Take the example of a restaurant, the assets on their own do not generate cash, but all together
they do. We call the restaurant as a whole a CGU.
Where an asset forms part of a CGU, any impairment review must be made on the group of
assets as a whole. How should this impairment loss be allocated? This will be described in the
following section.
8.4 ACCA SYLLABUS GUIDE OUTCOME 4:-
State the basis on which impairment losses should be allocated, and allocate an
impairment loss to the assets of a cash generating unit.
An impairment loss should be recognised whenever recoverable amount is below carrying
amount.
The impairment loss is an expense in the statement of profit or loss (unless it relates to a revalued
asset where the value changes are recognised directly in equity). It is important to adjust
depreciation for future periods.
How should an impairment loss be allocated?
A cash-generating unit to which goodwill has been allocated shall be tested for impairment at
least annually by comparing the carrying amount of the unit, including the goodwill, with
the recoverable amount of the CGU.
If the carrying amount of the unit exceeds the recoverable amount of the unit, the entity must
recognise an impairment loss.
The impairment loss is allocated to reduce the carrying amount of the assets of the unit in the
following order:
1) reduce the carrying amount of any goodwill allocated to the cash-generating unit. 2)
reduce the carrying amounts of the other assets of the unit pro rata.
Note:-
The carrying amount of an asset should not be reduced below the highest of:
• its fair value less costs to sell
• its value in use
Reversal of an Impairment Loss
The increased carrying amount due to reversal should not be more than what the depreciated
historical cost would have been if the impairment had not been recognised.
Reversal of an impairment loss is recognised as income in the statement of profit or loss. If the
reversal relates to a cash generating unit, the reversal is allocated to the assets of the unit on a
pro-rata basis according to their carrying value except goodwill. Reversal of an impairment loss
for goodwill is prohibited.
Lecture Example 2
The following carrying amounts were recorded in the books of a restaurant immediately prior to
the impairment: -

!58 acowtancy.com

Downloaded by sajedul Islam (sajedulnsu15@gmail.com)


lOMoARcPSD|3181314

The fair value less costs to sell of these assets is $260m whereas the value in use is

Show the impact of the impairment.

On 1 January 20x0, Cloud Co suffers a failure of its technology. The following carrying
values, were recorded in the books immediately prior to the impairment:

$m
Goodwill 100
Property, plant and equipment 100 Furniture
and fixtures 100
$270m.
Required:-
Lecture Example 3

$m
Goodwill 20
Technology 5 Brands
10 Land
50 Buildings
30 Other net assets
40
The recoverable value of the unit is estimated at $85 million. The technology is worthless,
following its complete failure. The other net assets include inventory, receivables and payables.
It is considered that the book value of other net assets is a reasonable representation of its net
realisable value.
Required:-
Show the impact of the impairment on 1 January.

!59 acowtancy.com

Downloaded by sajedul Islam (sajedulnsu15@gmail.com)


lOMoARcPSD|3181314

Further Questions 35
Question 1
Riley acquired a non-current asset on 1 October 2009 at a cost of $100,000 which had a useful
economic life of ten years and a nil residual value. The asset had been correctly depreciated up to
30 September 2014.
At that date the asset was damaged and an impairment review was performed. On 30 September
2014, the fair value of the asset less costs to sell was $30,000 and the expected future cash flows
The current cost of capital is 10% and a five year annuity of $1 per annum at 10% would

What amount would be charged to profit or loss for the impairment of this asset for the

were $8,500 per annum for the next five years.


have a present value of $3·79.

year ended 30 September 2014?


A. $17,785
B. $20,000
C. $30,000
D. $32,215
CHAPTER 9:
GROUP ACCOUNTING – THE MAIN PRINCIPLES
9.1 ACCA SYLLABUS GUIDE OUTCOME 1:-
Describe the concept of a group as a single economic unit.
The key principle underlying group accounts is the need to reflect the economic substance of the
relationship.

35 Specimen Exam Applicable from December 2014


!60 acowtancy.com

Downloaded by sajedul Islam (sajedulnsu15@gmail.com)


lOMoARcPSD|3181314

Owns more than 50% of equity shares


i.e.
P controls S S
P is an individual legal entity, known as the parent. The parent is “an entity that controls one or
more entities”36.
S is an individual legal entity, known as the subsidiary.
P owns more than 50% of the ordinary shares of S. It has enough voting power to appoint all the
directors of S. P has the power to govern the financial and operating policies of an entity so as to
obtain benefits from its activities.
Although P and S remain distinct, in economic substance, they can be regarded as a single unit,
the group.
9.2 ACCA SYLLABUS GUIDE OUTCOME 2:-
Explain the objective of consolidated financial statements.
The objective of the consolidated financial statements is to show the position of the group as if it
were a single economic entity, therefore:
• Assets and liabilities of P and S are included in the consolidated statement of financial
position
• Income and expenses of P and S are included in the consolidated statement of profit or
loss.
• All the other comprehensive income of P and S is included in the consolidated statement
of profit or loss and other comprehensive income showing other comprehensive income.
• Intra-group balances are eliminated
• The parent’s investment in each subsidiary is offset against the parent’s portion of equity
of each subsidiary.
9.3 ACCA SYLLABUS GUIDE OUTCOME 3:-
Explain and apply the definition of a subsidiary within relevant accounting standards.
IAS 27 Consolidated and Separate Financial Statement defines:
• Subsidiary – an entity that is controlled by another entity (known as the parent)
• Control – the power to govern the financial and operating policies of an entity so as to
obtain benefits from its activities.
9.4 ACCA SYLLABUS GUIDE OUTCOME 4:-
Identify and outline using accounting standards and other applicable regulation the
circumstances in which a group is required to prepare consolidated financial statements.
IAS 27 outlines the circumstances in which a group is required to prepare consolidated financial
statements.

36 IFRS 10 – published in May 2011


!61 acowtancy.com

Downloaded by sajedul Islam (sajedulnsu15@gmail.com)


lOMoARcPSD|3181314

Consolidated financial statements should be prepared when the parent company has control over
the subsidiary. Control is usually based on ownership of more than 50% of voting power.
However, IAS 27 lists the following situations where control exists, even when the parent owns
only 50% or less of the voting power of an enterprise.
(a) The parent has power over more than 50% of the voting rights by virtue of agreement
with other investments
(b) The parent has power to govern the financial and operating policies of the enterprise
by statute or under an agreement
(c) The parent has the power to appoint or remove a majority of members of the board
of directors (or equivalent governing body)
(d) The parent has power to cast a majority of votes at meetings of the board of
directors
As per IFRS 10, “an investor controls an investee if and only if the investor has all of the
following elements: -
• power over the investee i.e. the investor has existing rights that give it the ability to direct
the relevant activities (the activities that significantly affect the investee’s returns)
• exposure, or rights to variable returns from its involvement with the investee
• the ability to use its power over the investee to affect the amount of the investor’s
returns.”
9.5 ACCA SYLLABUS GUIDE OUTCOME 5:-
Describe the circumstances when a group may claim exemption from the preparation of
consolidated financial statements.
A parent is required to present consolidated financial statements using uniform accounting
policies for like transactions and other events in similar circumstances.
A parent is not required to (but may) present consolidated financial statements if and only if all
of the following four conditions are met:
1. The parent is itself a wholly-owned subsidiary, or is a partially-owned subsidiary of
another entity and its other owners, including those not otherwise entitled to vote, have
been informed about, and do not object to, the parent not presenting consolidated
financial statements;
2. The parent's debt or equity instruments are not traded in a public market;
3. The parent did not file, nor is it in the process of filing, its financial statements with a
securities commission or other regulatory organisation for the purpose of issuing any
class of instruments in a public market; and
4. The ultimate or any intermediate parent of the parent produces consolidated financial
statements available for public use that comply with International Financial Reporting
Standards.
9.6 ACCA SYLLABUS GUIDE OUTCOME 6:-

!62 acowtancy.com

Downloaded by sajedul Islam (sajedulnsu15@gmail.com)


lOMoARcPSD|3181314

Explain why directors may not wish to consolidate a subsidiary and outline using
accounting standards and other applicable regulation the circumstances where this is
permitted.
The directors of a parent company may not wish to consolidate some subsidiaries due to: -
a. Poor performance of the subsidiary
b. Poor financial position of the subsidiary
c. Differing activities (nature) of the subsidiary from the rest of the group These
reasons are not permitted according to IFRSs.
As already mentioned, consolidated financial statements should include all subsidiaries of the
parent.
IFRS 3 requires exclusion from consolidation only if the parent has lost control over its
investment. An entity loses control when it loses the power to govern its financial and operating
policies. This could occur, for e.g., where a subsidiary becomes subject to the control of the
government, a regulator, a court of law, or as a result of a contractual agreement.
If a parent loses control of a subsidiary, the parent: -
a. derecognizes the assets and liabilities of the former subsidiary from the consolidated SFP
b. recognizes any investment retained in the former subsidiary at its fair value
c. recognizes the gain or loss associated with the loss of control attributable to the former
controlling interest.
If, on acquisition, a subsidiary meets the criteria to be classified as ‘held for sale’ in accordance
with IFRS 5 (i.e. there should be evidence that the subsidiary has been acquired with the
intention to dispose of it within 12 months, and that management is actively seeking a buyer),
then it must still be included in the consolidation but accounted for in accordance with that
standard. The parent’s interest will be presented separately as a single figure on the face of the
consolidated SFP, rather than being consolidated like any other subsidiary. This will be
described in more detail when we do IFRS 3.
This might occur when a parent has acquired a group with one or more subsidiaries that do not fit
into its long-term strategic plans are therefore likely to be sold.
A subsidiary that has previously been excluded from consolidation and is not disposed of within
the 12 month period must be consolidated from the date of acquisition.
9.7 ACCA SYLLABUS GUIDE OUTCOME 7:-
Explain the need for using coterminous year ends and uniform accounting policies when
preparing consolidated financial statements.
Some companies in the group may have differing accounting dates. In practice, such companies
will often prepare financial statements up to the group accounting date for consolidation
purposes.
Where impracticable, the most recent financial statements of the subsidiary are used, adjusted for
the effects of significant transactions or events between the reporting dates of the subsidiary and

!63 acowtancy.com

Downloaded by sajedul Islam (sajedulnsu15@gmail.com)


lOMoARcPSD|3181314

consolidated financial statements. The difference between the date of the subsidiary’s financial
statements and that of the consolidated financial statements shall be no more than three months37.
IAS 27 allows the use of financial statements made up to a date not more than three months
earlier or later than the parent’s reporting date, with due adjustment for significant transactions
or other events between the dates.
CHAPTER 10:
GROUP ACCOUNTING – SUBSIDIARY
CONSOLIDATED STATEMENT OF FINANCIAL POSITION
10.1 ACCA SYLLABUS GUIDE OUTCOME 1:-
Prepare a consolidated statement of financial position for a simple group (parent and
one subsidiary) dealing with pre and post acquisition profits, non-controlling interests
and consolidated goodwill.
Describe and apply the required accounting treatment of consolidated goodwill.
Explain why it is necessary to eliminate intra-groups transactions.
Account for goodwill impairment.
Indicate the effect that the related party relationship between a parent and subsidiary
may have on the subsidiary’s entity statements and the consolidated financial statements.
Explain why it is necessary to use fair values for the consideration for an investment in a
subsidiary together with the fair values of a subsidiary’s identifiable assets and liabilities
when preparing consolidated financial statements.
Explain and account for other reserves (e.g. share premium and revaluation reserves).
Account for the effects of fair value adjustments (including their effect on consolidated
goodwill) to:
• Depreciating and non-depreciating non-current assets
• Inventory
• Monetary liabilities
• Assets and liabilities not included in the subsidiary’s own statement of financial
position, including contingent assets and liabilities.
10.1.1 Preparing a consolidated SFP:
1. The investment in the subsidiary (S) shown in the parent’s (P) statement of financial position
is replaced by the net assets of S.
2. The cost of the investment in S is effectively cancelled with the ordinary share capital and
reserves of the subsidiary.
The consolidated statement of financial position shows:
• The net assets of the whole group (P + S)
• The share capital of the group which always equals the share capital of P only and
• The retained profits, comprising profits made by the group (i.e. all of P’s historical profits
+ profits made by S post-acquisition).

37 IFRS 10
!64 acowtancy.com

Downloaded by sajedul Islam (sajedulnsu15@gmail.com)


lOMoARcPSD|3181314

10.1.2 Goodwill
The value of a company will normally exceed the value of its net assets. The difference is
goodwill. This goodwill represents assets not shown in the statement of financial position of the
acquired company such as the reputation of the business and the loyalty of staff.
Where less than 100% of the subsidiary is acquired, the value of the subsidiary comprises two
elements:
• The value of the part acquired by the parent;
• The value of the part not acquired by the parent, known as the non-controlling interest.
There are two methods in which goodwill may now be calculated following the update to IFRS
3.
(i) Proportion of net assets method (Partial Goodwill) Only the parent’s
share of goodwill is calculated.
(ii) Fair value of non-controlling interest method (Full Goodwill)
This results in 100% of the goodwill being shown in the group statement of financial
position – that belonging to the shareholders of the parent and that belonging to the non-
controlling interest.
Positive goodwill is:-
1. An intangible non-current asset in the SFP
2. Tested annually for impairment (amortisation of goodwill is not permitted)
Negative goodwill:-
1. Arises where the cost of the investment is less that the value of net assets purchased.
2. Negative goodwill is credited directly to the statement of profit or loss.
Impairment of Goodwill
Goodwill should be checked for impairment annually. The examiner will tell you the amount of
impairment. How you account for it depends on the goodwill method used:-
1. Proportion of net assets method (Partial Goodwill)
Reduce Group Retained Earnings
Reduce Goodwill in group SFP with the
full amount of impairment given
2. Fair value of non-controlling interest method (Full Goodwill)
Reduce Goodwill in group SFP
Reduce NCI calculation by the NCI% x impairment amount
Reduce Group Retained Earnings by the % holding in S x impairment amount
10.1.3 Pre- and Post-Acquisition Profits
Pre-acquisition profits are the reserves which exist in a subsidiary company at the date when it
is acquired.
These are included in the goodwill calculation.
Post-acquisition profits are profits made and included in the retained earnings of the subsidiary
company since acquisition.

!65 acowtancy.com

Downloaded by sajedul Islam (sajedulnsu15@gmail.com)


lOMoARcPSD|3181314

They are included in group reserves.


Only the group share of the post-acquisition reserves of S is included in the group statement of
financial position, i.e. the reserves of S which arose after acquisition by P.
N.B. Where the acquisition occurs during the financial year, it is important to calculate the value
of profits at the date of acquisition using time-apportionment, unless stated otherwise (see Pilot
Paper Qs 1).
10.1.4 Non-controlling Interest (NCI)
A parent may not own all of the shares in the subsidiary, e.g. if P owns only 70% of the ordinary
shares of S, there is a non-controlling interest of 30%.
NCI is presented in the consolidated statement of financial position within equity, separately
from the equity of the owners of the parent.
A reporting entity must attribute the profit or loss and each component of other comprehensive
income to the owners of the parent and to the NCI.
Accounting treatment of a non-controlling interest
• In the consolidated statement of financial position, include all of the net assets of S
• Transfer back the net assets of S which belong to the non-controlling interest within the
capital and reserves section of the consolidated statement of financial position. Where the full
goodwill method is used to value the NCI (i.e. NCI is valued at its fair value), a proportion of
goodwill on acquisition is also transferred back to the NCI (as per full goodwill method of
goodwill, described above).
Lecture Example 1:
The following balances relate to H and S on 31 December 2009.
Holding Co. Subsidiary Co.
Investment in S 200 Other
Net Assets 100 400
Share Capital 100 100 Retained
Earnings 200 300
H acquired 80% of S on 1 January 2009 when S’s retained earnings were $80.

a) Prepare the consolidated SFP of the group using the proportion of net assets method
to value the non-controlling interest.
b) H’s policy is to value the non-controlling interest of S at the date of acquisition at its
fair value which the directors determined to be $50.
Calculate: -
i. Goodwill ii.
NCI
iii. Group Retained Earnings

!66 acowtancy.com

Downloaded by sajedul Islam (sajedulnsu15@gmail.com)


lOMoARcPSD|3181314

10.1.5 Fair Value of Assets and Liabilities


The fair value of assets and liabilities is defined in IFRS 3 as ‘the amount for which an asset
could be exchanged or a liability settled between knowledgeable, willing parties in an arm’s
length transaction’.
IFRS 3 requires that the subsidiary’s assets and liabilities are recorded at their fair value for the
purposes of the calculation of goodwill and production of consolidated accounts.
Adjustments will therefore be required where the subsidiary’s accounts themselves do not reflect
fair value.
(1) Adjust both columns of the net assets calculation to bring the net assets to fair value at
acquisition and reporting date.
(2) At the reporting date, make the adjustment on the face of the SFP when adding across
assets and liabilities.
An adjustment should also be made for depreciation when calculating the net assets at reporting
date only. The amount of depreciation may have to be calculated based on the remaining life
(and depreciation policy) at the date of acquisition.
Lecture Example 2:
Draft SFPs of H and S on 31 March 2009 are as follows:
Holding Co. Subsidiary Co.
Cost of investment S 200
Other Net Assets 800 500
Share Capital 100 100
Retained Earnings 900 400
Two years ago H acquired 90% of S when S’s retained earnings were $100. At acquisition, the
fair value of S’s net assets exceeded their book value by $10. Any difference in fair value is due
to PPE which has a 10 year remaining useful economic life at that date.
On the date of acquisition, the fair value of the non-controlling shareholding in S was $26.
The group is to value the non-controlling interest at its fair value at date of acquistion.
There has been no impairment of goodwill since the acquisition.
Prepare the consolidated SFP of the group.
Lecture Example 3:
Below are the SFP of H and S as at 31 December 2004.
Holding Co. Subsidiary Co.
Investment in S 100
Other Net Assets 200 140
Share Capital 100 40 Retained Earnings 200 100
H acquired 80% of S two years ago when S’s retained earnings were $50. At that date, S’s PPE
had a fair value of $10 in excess of the carrying value and 10 years useful economic life.
a) Prepare the consolidated SFP of the group using the proportion of net assets method
to value the non-controlling interest.

!67 acowtancy.com

Downloaded by sajedul Islam (sajedulnsu15@gmail.com)


lOMoARcPSD|3181314

b) Calculate goodwill, NCI and group retained earnings if the group uses the full
goodwill method to value NCI. On the date of acquisition, the fair value of non-
controlling shareholding in S was $40.
10.1.6 Share for share exchanges
These are often required in the F7 exam. They form part, or all, of the cost of investment which
is used in the goodwill calculation.
If this exchange has yet to be accounted for, the double entry is always: -
Dr Cost of Investment
Cr Share capital (with the nominal value of P shares given out)
Cr Share premium (with the premium)
Lecture Example 4:
H acquired 80% of S shares via a 2 for 1 share exchange. At the date of acquisition, the
following balances were in the books of H and S:
Holding Co. Subsidiary Co.
Share Capital ($1) $400 (0.50c) $400 Share
Premium $100 $50
The share price of H was $2 at the date of acquisition. This has not been accounted for.
The group uses the proportion of net assets method to value the non-controlling interest. There
has been no impairment of goodwill since the acquisition.
Show the accounting treatment required to account for the share exchange.
Lecture Example 5:
Holding Co. Subsidiary Co.
Share Capital ($1) $100 $100
Share Premium ($1) $100 $100
H acquired 80% of S shares via a 3 for 2 share exchange. The share price of H at acquisition was
$3. This has not been accounted for.
The group uses the proportion of net assets method to value the non-controlling interest. There
has been no impairment of goodwill since the acquisition.
Show the accounting treatment required to account for the share exchange.
10.1.7 Deferred and contingent consideration
Part of the purchase consideration may not be paid at the date of the acquisition, but is deferred
until a later date – deferred consideration.
Deferred consideration should be measured at fair value at the date of the acquisition.
Where the deferred period is significant (usually one or more years), the amount of the cash
consideration will need to be discounted to a present value, at the rate for cost of capital given in
the question. In the period after the acquisition, the parent should accrue a finance charge (at
the rate of the cost of capital) in its statement of profit or loss (which is consolidated) and add
this to the carrying amount of the deferred consideration (a liability) in its SFP (which is also
consolidated):-

!68 acowtancy.com

Downloaded by sajedul Islam (sajedulnsu15@gmail.com)


lOMoARcPSD|3181314

Debit consolidated retained earnings (with the finance charge)


Credit deferred consideration
Sometimes, the parent company may agree to pay a further amount, depending on the post-
acquisition performance of the subsidiary – contingent consideration. The payment of contingent
consideration may be in the form of equity, a liability (issuing a debt instrument) or cash.
Any contingent consideration should always be included as long as it can be measured reliably.
It must be recognised at its fair value.
Where the contingent consideration involves the issue of shares, this should be recognised as part
of shareholders' funds under a separate caption representing shares to be issued.
Lecture Example 6:38
Holdrite purchased 80% of the issued share capital of Staybrite on 1 April 2005. Details of the
purchase consideration given at the date of purchase are:
▪ a share exchange of three shares in Holdrite for every five shares in Staybrite
▪ the issue to the shareholders of Staybrite 8% loan notes, redeemable at par on 31 March
2008 on the basis of $100 loan note for every 125 shares held in Staybrite
▪ a cash sum of $121 for every 100 shares in Staybrite, payable on 1 April 2007. Holdrite’s
cost of capital is 10% per annum.
The market price of Holdrite’s shares at 1 April 2005 was $4.50 per share. In order to help fund
the acquisition of new operating capacity for Staybrite, Holdrite also subscribed for a 10% $4m
loan note (2008) issued by Staybrite immediately after the acquisition. A fair value exercise was
carried out at the date of acquisition of Staybrite, with the following results:
Carrying amount Fair value
$000 $000

Land 20,000 23,000


Plant 25,000 30,000
Inventory 5,000 6,000
The fair values have not been reflected in Staybrite’s financial statements.
In addition, a note to Staybrite’s financial statements gave details of a contingent liability in
respect of outstanding litigation. The directors of Holdrite considered that $5m would be a
reliable measurement of this contingent liability. The details of each company’s share capital and
reserves at 1 April 2005 are:
Holdrite Staybrite
$000 $000

Equity shares of $1 each 20,000 10,000


Share premium 5,000 4,000
Retained earnings 18,000 8,000

38 Article “Consolidations” by S. Scott – Student Accountant June/July 2006


http://www2.accaglobal.com/pubs/students/publications/student_accountant/archives/sa_jj06_scott_ACCA.pdf
!69 acowtancy.com

Downloaded by sajedul Islam (sajedulnsu15@gmail.com)


lOMoARcPSD|3181314

Required:-
Calculate the goodwill arising on the acquisition of Staybrite.

10.1.8 Intra Group Balances


If the companies within the same group trade with each other, then this will probably lead to:
• A receivables account in one company’s SFP • A
payables account in the other company’s SFP.
These are amounts owing within the group rather than outside the group and therefore they must
not appear in the consolidated statement of financial position. They are therefore cancelled
against each other on consolidation.
10.1.9 Cash/goods in Transit
When there are goods or cash in transit, the rules are:
- If the goods or cash are in transit between P and S, make the adjusting entry to the
statement of financial position of the receiving company.
- Cash in transit adjusting entry is:
o Dr Cash in transit (increase) o Cr Receivables account (decrease) -
Goods in transit adjusting entry is:
o Dr Inventory (increase)
o Cr Payables account (increase)
10.1.10 Unrealised Profit
Unrealised profit may arise within a group scenario on:
a. Inventory where companies trade with each other
b. Non-current assets where one company has transferred an asset to the other company
within the same group.
a. Adjustment for unrealised profit in inventory
(1) Determine the value of closing inventory which has been purchased from the other
company in the group.
(2) Use mark-up or margin to calculate how much of that value represents profit earned by
the selling company.
(3) Make the adjustments according to who the seller is.
If the seller is the parent company:
Dr Group retained earnings
Cr Group inventory (deduct the profit when adding P’s inventory + S’s inventory on the
face of the consolidated SFP).
If the seller is the subsidiary:
Dr Subsidiary retained earnings
Cr Group inventory (deduct the profit when adding P’s inventory + S’s inventory on the
face of the consolidated SFP).
Lecture Example 7:

!70 acowtancy.com

Downloaded by sajedul Islam (sajedulnsu15@gmail.com)


lOMoARcPSD|3181314

H sells to S goods worth $600. H makes 20% profit margin. S sells $200 of these goods at cost.
What is the unrealised profit?
Lecture Example 8:
S sells goods to H for $600. S makes a 20% mark up. H has goods at cost left in stock worth
$200.
What is the unrealised profit?
Lecture Example 9:
H sells goods to S for $100 per month making a 20% margin. H acquired 80% of S, 9 months
ago. S has still in stock the last month’s goods transferred from H.
What is the unrealised profit?
Lecture Example 10:
H sold goods to S at a price of $12 million. These goods cost H $9 million. During the year S
sold $10 million (at the cost to S) of these goods for $15 million.
Calculate the unrealised profit.
Lecture Example 11:
At 1 January 2009, H acquired 80% of S shares when S’s retained earnings were $60. The fair
value of S’s net assets was $180. The difference is due to land. H sold goods to S for $40 making
a 20% margin. S has sold goods for $30 (at cost).
The SFPs of the two companies at 31 December 2009 are:
Holding Co. Subsidiary Co.
Investment in S $200
Other Net Assets $400 $300
Share Capital $100 $100 Retained Earnings $500
$200
The group uses the full goodwill method to value the non-controlling interest. On the date of
acquisition, the fair value of non-controlling interest of S was $36. There has been no
impairment of goodwill since the acquisition.
Prepare the consolidated SFP of the group.
Lecture Example 12:
The balances of H and S at 31 December 2009:
Holding Co. Subsidiary Co.
Investment in S $1000
Other Net Assets $8000 $4000
Share Capital $2000 $1000
Retained Earnings $7000 $3000
H acquired 80% of S when S’s retained earnings were $100. The fair value of S’s net assets was
equal to their carrying values at that time. S sold goods to H for $600 making 20% margin profit.
H has sold 2/3 of these goods.

!71 acowtancy.com

Downloaded by sajedul Islam (sajedulnsu15@gmail.com)


lOMoARcPSD|3181314

P’s policy is to value the non-controlling interest of S at the date of acquisition at its fair value
which the directors determined to be $250.
Prepare the consolidated SFP of the group.
b. Adjustment for unrealised profit in the transfer of non-current assets
Occasionally, a non-current asset is transferred within the group (say from a parent to a
subsidiary). The parent may have manufactured the asset as part of its normal production (and
therefore included the sale in revenue), or it may have transferred an asset previously used as
part of its own non-current assets. If the transfer is done at cost, then, in the first case, the cost of
the asset must be removed from both revenue and cost of sales. In the second case, no
elimination would be required.
If one company sells non-current assets to another company in the same group at a profit,
adjustments must be made for:
1. Profit on sale
2. Depreciation
The whole scenario has to be recreated as if the sales have never occurred.
Carrying value at reporting date X
Carrying value at reporting date if intra-group transfer
had not occurred X Adjustment
X
The double-entry of this adjustment is: -
Dr Retained Earnings of the seller
Cr Non-Current Assets (P’s NCA + S’s NCA – Adjustment for UP)
Lecture Example 13:
H sells PPE to S costing $1000 for a selling price of $1500, depreciation at 10% per annum.
What is the unrealised profit?
Lecture Example 14:
S sold a machine with a NBV of $100 000 to H at a transfer price of $120 000 at the year start.
Group policy dictates that the machine is depreciated over its remaining life of 5 years.
Calculate the unrealised profit on the sale of the machine.
10.1.11 Intangible Assets
The subsidiary may have intangible assets, e.g. development expenditure. These assets can be
recognised separately from goodwill if they are identifiable.
The subsidiary may also have internally-generated assets which have not yet been recognised as
intangible assets, e.g. brand. The parent will recognise this as an asset in the consolidated
financial statements.
Refer to lecture example 15 below.
10.1.12 Contingent liabilities
Contingent liabilities of the parent are recognised if their fair value can be measured reliably.
After their initial recognition, the parent should measure contingent liabilities are the higher of:

!72 acowtancy.com

Downloaded by sajedul Islam (sajedulnsu15@gmail.com)


lOMoARcPSD|3181314

i. The amount that would be recognised as per IAS 37 ii. The


amount initially recognised
10.1.13 Future Losses
The parent should not recognise any liabilities for future losses. It should not recognise a liability
for any plans to restructure a subsidiary.
10.1.14 Professional fees
Fees and other costs attributable to the combination should be expensed when incurred.
Lecture Example 15:39
Highveldt, a public listed company, acquired 75% of Samson’s ordinary shares on 1 April 2005.
The purchase consideration consisted of:
• a share exchange of one share in Highveldt for two shares in Samson. The market price of
Highveldt shares at the date of acquisition was $4 each
• an immediate $1.75 per share in cash
• a further amount of $81m payable on 1 April 2006. Highveldt’s cost of capital is 8% per
annum.
Highveldt has only recorded the consideration of $1.75 per share.
The summarised SFPs of the two companies at 31 March 2006 are shown below:
Highveldt Samson
$m $m $m $m
Tangible non-current assets 570 380
Investments 150 nil
720 380
Current assets 130 90
Total assets 850 470
Share capital and reserves:

Ordinary shares of $1 each Reserves: 270 80

Share premium 80 40
Revaluation reserve 40 nil
Retained earnings
- 1 April 2005 160 120
- Year to 31 March 2006 190 350 101 221
740 341
Non-current liabilities
10% loan note nil 60
Current liabilities 110 69
Total equity and liabilities 850 470

39 Article “Consolidations” by S. Scott, Student Accountant - June/July 2006 (adjusted for full goodwill method)
http://www2.accaglobal.com/pubs/students/publications/student_accountant/archives/sa_jj06_scott_ACCA.pdf
!73 acowtancy.com

Downloaded by sajedul Islam (sajedulnsu15@gmail.com)


lOMoARcPSD|3181314

The following information is relevant:


i. Highveldt has a policy of revaluing land and buildings to fair value. At the date of
acquisition, Samson’s land and buildings had a fair value of $20m in excess of their
carrying amounts, and at 31 March 2006 this had increased by a further $4m (ignore any
additional depreciation).
ii. Samson had established a line of products under the brand name of Titanware.
Acting on behalf of Highveldt, a firm of specialists had valued the brand name at $40m
with an estimated life of 10 years as at 1 April 2005. The brand is not included in
Samson’s statement of financial position.
iii. Immediately after acquisition, Highveldt sold Samson an item of plant for $15m that it
had manufactured at a cost of $10m. The plant had an estimated life of five years
(straight-line depreciation) and no residual value.
iv. On 1 October 2005 Samson issued $60m 10% (actual and effective rate) loan notes.
Highveldt subscribed for $20m of this issue. Samson has not paid any interest on this
loan, but it has recorded the amount due as a current liability. Highveldt has also accrued
for its interest receivable on this loan.
v. Post-acquisition, Samson sold goods at a price of $18m to Highveldt; $5m of these goods
were still in the inventory of Highveldt at 31 March 2006. Samson applied a mark-up on
cost of 25% to these goods.
vi. Due to poor trading performance, consolidated goodwill was impaired by $20m.
vii. Highveldt’s policy is to value the non-controlling interest of Samson at the date of
acquisition at its fair value. The directors determined this to be $75m.
Required:-
Prepare the consolidated statement of financial position of Highveldt at 31 March 2006.
10.2 ACCA SYLLABUS GUIDE OUTCOME 2:-
Goodwill and Non-Controlling Interests.
As discussed in Section 10.1.2, the parent measures any non-controlling interest either:
(i) at fair value as determined by the directors of the acquiring company (often called the
‘full goodwill’ method); or
(ii) at the non-controlling interest’s proportionate share of the acquiree’s (subsidiary’s)
identifiable net assets.
Lecture Example 1646
Parent pays $100m for 80% of Subsidiary which has net assets with a fair value of $75m. The
directors of Parent have determined the fair value of the NCI at the date of acquisition was $25m.
Required:-

!74 acowtancy.com

Downloaded by sajedul Islam (sajedulnsu15@gmail.com)


lOMoARcPSD|3181314

Calculate goodwill using both the proportionate and the full goodwill method.

What happens when there is an impairment?

47

Parent owns 80% of Subsidiary (a CGU). Its identifiable net assets at 31 March 2010

Net assets included in the consolidated statement

Consolidated goodwill (calculated under method (i))

10.2.1
Lecture Example 17
are $500.
Scenario 1: Full Goodwill Method
$
of financial position 500
200
700
NCI 140
Scenario 2: Proportionate Method

$
Net assets included in the consolidated statement
of financial position 500
Consolidated goodwill (calculated under method (ii)) 160

46
Article “IFRS 3, Business Combinations” by S. Scott, Student Accountant, July 2010
http://www.accaglobal.com/content/dam/acca/global/PDF-students/2012/
sa_jul10_F7_IFRS3.pdf
47
Article “IFRS 3, Business Combinations” by S. Scott, Student Accountant, July 2010
http://www.accaglobal.com/content/dam/acca/global/PDF-students/2012/
sa_jul10_F7_IFRS3.pdf
660
NCI 100
An impairment review of Subsidiary was carried out at 31 March 2010.
!75 acowtancy.com

Downloaded by sajedul Islam (sajedulnsu15@gmail.com)


lOMoARcPSD|3181314

Required:-
For scenarios 1 and 2, calculate the impairment losses and show how they would be
allocated if the impairment review concluded that the recoverable of Subsidiary was:
(i) $450
(ii) $550
Further Questions
Question 1 40
Consolidated financial statements are presented on the basis that the companies within the group
are treated as if they are a single (economic) entity.
Which of the following are requirements of preparing group accounts?
(i) All subsidiaries must adopt the accounting policies of the parent
(ii) Subsidiaries with activities which are substantially different to the activities of
other members of the group should not be consolidated
(iii) All entity financial statements within a group should (normally) be prepared to
the same accounting year end prior to consolidation
(iv) Unrealised profits within the group must be eliminated from the consolidated
financial statements

A. All four
B. (i) and (ii) only
C. (i), (iii) and (iv)
D. (iii) and (iv)
Question 2
Venus Co acquired 75% of Mercury Co’s 100,000 $1 ordinary share capital on 1 November
2011. The consideration consisted of $2 cash per share and 1 share in Venus Co for every 1 share
acquired in Mercury Co. Venus Co shares have a nominal value of $1 and a fair value of $1.75.
The fair value of the non-controlling interest was $82,000 and the fair value of net assets
acquired was $215,500.
What should be recorded as goodwill on acquisition of Venus Co in the consolidated financial
statements?

40 Specimen Exam Applicable from December 2014


!76 acowtancy.com

Downloaded by sajedul Islam (sajedulnsu15@gmail.com)


lOMoARcPSD|3181314

Honey Co acquired 75% of Bee Co on 1 April 2013, paying $2 for each ordinary share
acquired. The fair value of the non-controlling interest at 1 April 2013 was $300. Bee
Co’s individual financial statements as at 30 September 2013 included:
$

1,000
710
1,710

A. $16,500
B. $147,750
C. $91,500
D. $63,375
Question 3

Statement of financial position


Ordinary share capital ($1 each)
Retained earnings

Income statement
Profit after tax for the year 250

Profit accrued evenly throughout the year.

What is the goodwill on acquisition on 1 April 2013?


A. $715
B. $90
C. $517
D. $215

!77 acowtancy.com

Downloaded by sajedul Islam (sajedulnsu15@gmail.com)


lOMoARcPSD|3181314

CHAPTER 11:
GROUP ACCOUNTING:
CONSOLIDATED STATEMENT OF FINANCIAL POSITION
ASSOCIATE
11.1 ACCA SYLLABUS GUIDE OUTCOME 1:-
Define an associate and explain the principles and reasoning for the use of equity
accounting.
IAS 28 defines an associate as:
An entity over which the investor has significant influence and that is neither a subsidiary nor an
interest in joint venture.
Significant influence is the power to participate in the financial and operating policy decisions
of the investee but is not control or joint control over those policies.
There are several indicators of significant influence, but the most important are usually
considered to be a holding of between 20% and 50% of the voting shares and board
representation.
Equity accounting brings an associate investment into the parent company’s financial
statements initially at cost.
The carrying amount of the investment is then adjusted in each period by the group share of the
profit of the associate less any impairment losses. 41
11.2 ACCA SYLLABUS GUIDE OUTCOME 2:-
Prepare a consolidated statement of financial position to include a single subsidiary and
an associate.
The associate is included as a non-current asset investment in the SFP calculated as:
$’000
Cost of investment X
P’s share of post acquisition profits of A X
Less: impairment losses of A (X)

41 If a company can no longer exert significant influence over another company, it will be treated under IAS 39. It
should no longer be equity accounted from the date of loss of significant influence. Its carrying amount at that date
will be its initial recognition value under IAS 39 and thereafter it will be carried at fair value.
!78 acowtancy.com

Downloaded by sajedul Islam (sajedulnsu15@gmail.com)


lOMoARcPSD|3181314

The group share of the associate’s post acquisition profits /losses and the impairment of
goodwill must also be included in group
the retained earningscalculation.

S – group share of post-acquisition reserves


A – group share of post-acquisition reserves
Less: impairment losses to date (S + A)

X
Group retained earnings
$
Parent (100%) X
X
X
X
X
Fair Values
If the fair values of the associate’s net assets at acquisition are materially different from their
book value, the net assets should be adjusted in the same way as for a subsidiary.
Trading with the Associate
Generally, the associate is considered to be outside the group. Therefore if a group company
trades with the associate, the resulting payables and receivables will remain in the consolidated
statement of financial position.
Unrealised profit in inventory
(1) Determine the value of closing inventory which is the result of a sale to or from the associate.
(2) Use mark-up/margin to calculate the profit earned by the selling company.
(3) Make the required adjustments. These will depend upon who the seller is:
Parent company selling to associate – the profit element is included in the parent company’s
accounts.

!79 acowtancy.com

Downloaded by sajedul Islam (sajedulnsu15@gmail.com)


lOMoARcPSD|3181314

Associate selling to parent company


– the profit element is included in the associate

N.B. Don’t forget to take the unrealised profit and multiply by the percentage holding
which the parent has in the associate.

H sold goods to A worth $10 for $40. A sold 2/3 of these goods. H’s share in A is 40%.

Dr Group retained earnings


Cr Investment in associate
company’s accounts.
Dr Group retained earnings
Cr Group inventory
Lecture Example 1:
What is the unrealised profit?
Lecture example 2:42
On 1 October 2006, Plateau acquired the following non-current investments:
Three million equity shares in Savannah by an exchange of one share in Plateau for every two
shares in Savannah, plus $1.25 per acquired Savannah share in cash. The market price of each
Plateau chare at the date of acquisition was $6, and the market price of each Savannah share at
the date of acquisition was $3.25.
Thirty per cent of the equity shares of Axle at a cost of $7.50 per share in cash.
Only the cash consideration of the above investments has been recorded by Plateau. In addition,
$500,000 of professional costs relating to the acquisition of Savannah are included in the cost of
the investment.
The summarised draft statements of financial position of the three companies at 30 September
2007 are:

42 December 2007 Question 1 revised by S. Scott in Article “IFRS 3, Business Combinations” July 2010
http://www.accaglobal.com/content/dam/acca/global/PDF-students/2012/sa_jul10_F7_IFRS3.pdf
!80 acowtancy.com

Downloaded by sajedul Islam (sajedulnsu15@gmail.com)


lOMoARcPSD|3181314

Plateau Savannah Axle


$’000 $’000 $’000
Assets
18 ,400 10 ,400
Non-current
assets: Property,
13 ,250 nil
plant
6,500 nil
and equipment 18,000
38 ,150 10 ,400
Investments in Savannah
and Axle nil
6,900 6,200
Financial asset
3,200 1,500
investments nil
48 ,250 18 ,100
18,000
Current assets:
Inventory 3,600
10 ,000 4,000
Trade receivables 2,400 Total
assets 24,000 16 ,000 6,000
Equity and liabilities
Equity shares of 9,250 2,900 $1 each 4,000
Retained earnings 35 ,250 12 ,900
- at 30 September 2006
11,000
- for year ended
30 September 2007 5,000
20,000
Non-current liabilities
7% Loan notes 5,000 1,000 1,000

Current liabilities 8,000 4,200 3,000


Total equity and liabilities 48,250 18,100 24,000

The following information is relevant:


(i) At the date of acquisition, Savannah had five years remaining of an agreement to supply
goods to one of its major customers. Savannah believes it is highly likely that the
agreement will be renewed when it expires. The directors of Plateau estimate that the
value of this customer based contract has a fair value of $1m, an indefinite life, and has
not suffered any impairment.
(ii) On 1 October 2006, Plateau sold an item of plant to Savannah at its agreed fair value of
$2.5m. Its carrying amount prior to the sale was $2m. The estimated remaining life of the
plant at the date of sale was five years (straight-line depreciation).
(iii)During the year ended 30 September 2007, Savannah sold goods to Plateau for $2.7m.
Savannah had marked up these goods by 50% on cost. Plateau had a third of the goods

!81 acowtancy.com

Downloaded by sajedul Islam (sajedulnsu15@gmail.com)


lOMoARcPSD|3181314

still in its inventory at 30 September 2007. There were no intragroup


payables/receivables at 30 September 2007.
(iv)At the date of acquisition the non-controlling interest in Savannah is to be valued at its
fair value. For this purpose Savannah’s share price at that date can be taken to be
indicative of the fair value of the shareholding of the non-controlling interest. Impairment
tests on 30 September 2007 concluded that neither consolidated goodwill nor the value of
the investment in Axle had been impaired.
(v) The financial asset investments are included in Plateau’s statement of financial position
(above) at their fair value on 1 October 2006, but they have a fair value of $9m at 30
September 2007.
(vi)No dividends were paid during the year by any of the companies.
Required:-

Prepare the consolidated statement of financial position for Plateau as at 30 September


2007.
(20 marks)
Lecture Example 3
On 1 October 2005 Pumice acquired the following non-current investments:
• 80% of the equity share capital of Silverton at a cost of $13.6 million
• 50%of Silverton’s 10% loan note at par
• 1.6 million equity shares in Amok at a cost of $6.25 each.
The summarised draft statement of financial positions of the three companies at 31 March 2006
are:
Pumice Silverton Amok
$’000 $’000 $’000
Non-current assets
Property. Plant and equipment 20,000 8,500 16,500
Investments 26,000 Nil 1,500

46,000 8,500 18,500


Current assets 15,000 8,000 11,000
Total assets 61,000 16,500 29,000
Equity and liabilities
Equity
Equity sharesof$1 each 10,000 3,000 4,000 Retained earnings 37,000 8,000 20,000
47,000 11,000 24,000
Non-currents liabilities
8% loan note 4,000 Nil Nil 10% loan note Nil 2,000 Nil
Current liabilities 10,000 3,500 5,000
Total equity and liabilities 61,000 16,500 29,000
!82 acowtancy.com

Downloaded by sajedul Islam (sajedulnsu15@gmail.com)


lOMoARcPSD|3181314

The following information is relevant:


(i) The fair value of Silverton’s assets were equal to their carrying amounts with the exception
of land and plant. Silverton’s land had a fair value of $400,000 in excess of its carrying
amount and plant had fair value of $1.6 million in excess of its carrying amount. The plant
had a remaining life of four years (straight-line depreciation) at the date of acquisition.
(ii) In the post acquisition period Pumice sold goods to Silverton at a price of $6 million. These
goods had cost Pumice $4 million. Half of these goods were still in the inventory of
Silverton at 31 March 2006. Silverton had a balance of $1.5 million owing to Pumice at 31
March 2006 which agreed with Pumice’s records.
(iii) The net profit after tax for the year ended 31 March 2006 was $2 million for Silverton and
$8 million for Amok. Assume profits accrued evenly throughout the year.

!83 acowtancy.com

Downloaded by sajedul Islam (sajedulnsu15@gmail.com)


lOMoARcPSD|3181314

(iv) Pumice’s policy is to value the non-controlling interest at fair value at the date of
acquisition. The fair value of the non-controlling interest at acquisition was determined to
be $3 million.

An impairment test at 31 March 2006 concluded that the consolidated goodwill was impaired by
$500,000 and the investment in Amok was impaired by $200,000.

(v) No dividends were paid during the year by any of the companies.
Required:-
(a) Discuss how the investments purchased by Pumice on 1 October 2005 should be
treated in its consolidated financial statements.
(b) Prepare the consolidated statement of financial position for Pumice as at 31 March
2006.
(ACCA F7 Exam Pilot Paper Qs 1 revised {FV of NCI method})
Further Questions
Question 1 43
An associate is an entity in which an investor has significant influence over the investee.
Which of the following indicate(s) the presence of significant influence?
(i) The investor owns 330,000 of the 1,500,000 equity voting shares of the investee
(ii) The investor has representation on the board of directors of the investee (iii) The
investor is able to insist that all of the sales of the investee are made to a subsidiary of the
investor
(iv) The investor controls the votes of a majority of the board members
A. (i) and (ii) only
B. (i), (ii) and (iii)
C. (ii) and (iii) only
D. All four
Question 2 44
The Caddy group acquired 240,000 of August’s 800,000 equity shares for $6 per share on 1
April 2014. August’s profit after tax for the year ended 30 September 2014 was $400,000 and it
paid an equity dividend on 20 September 2014 of $150,000.
On the assumption that August is an associate of Caddy, what would be the carrying amount of
the investment in August in the consolidated statement of financial position of Caddy as at 30
September 2014?
A. $1,455,000

43 Specimen Exam Applicable from December 2014


44 Specimen Exam Applicable from December 2014

84! acowtancy.com

Downloaded by sajedul Islam (sajedulnsu15@gmail.com)


lOMoARcPSD|3181314

According to IAS 28, Investments in associates, which, if any, of the following

(1) Equity accounting will always be used when an investing company holds between
20% - 50% of the equity shares in another company
(2) Dividends received from an investment in associate will be presented as investment
income in the consolidated accounts.

Statement 1 Statement 2
Correct
Incorrect
Correct
Incorrect

B. $1,500,000
C. $1,515,000 D. $1,395,000 Question 3
statements are correct?

A. Correct B.
Correct
C. Incorrect D.
Incorrect
Question 4
Which of the following investments of Coffee Co should be equity accounted in the consolidated
financial statements?
(1) 40% of the non-voting preference share capital in Tea Co
(2) 18% of the ordinary share capital in Café Co with two of the five directors of Coffee
Co on the board of Café Co
(3) 50% of the ordinary share capital of Choc Co, with five of the seven directors of
Coffee Co on the board of Choc Co
A. 1 and 2
B. 2 only
C. 1 and 3 only
D. 2 and 3 only

85! acowtancy.com

Downloaded by sajedul Islam (sajedulnsu15@gmail.com)


lOMoARcPSD|3181314

CHAPTER 12:
GROUP ACCOUNTING:
CONSOLIDATED STATEMENT OF PROFIT OR LOSS - SUBSIDIARY
12.1 ACCA SYLLABUS GUIDE OUTCOME 1:-
Prepare a consolidated statement of profit or loss and consolidated statement of profit or
loss and other comprehensive income for a simple group dealing with an acquisition in
the period and non-controlling interest.
Basic principles
1. From sales revenue to profit after tax, include all of P’s income and expenses plus all of S’s
income and expenses (where a mid-year acquisition has occurred, these must be time-
apportioned).
2. Once the profit after tax is calculated, deduct share profits due to the non-controlling interest.
Impairment of goodwill
The charge for the year, re impairment of goodwill, will be passed through the consolidated
statement of profit or loss, usually through operating expenses, unless stated otherwise.
Non-controlling interest
This is calculated as: NCI% x subsidiary’s profit after tax (taken from S’s column of
consolidation schedule).
Dividends
A payment of a dividend by S to P must be cancelled. Any dividend income shown in the
consolidated statement of profit or loss must arise from investments other than those in
subsidiaries or associates.
Fair Values
If a depreciating non-current asset of the subsidiary has been revalued, this will result in an
adjustment to the consolidated statement of profit or loss.
Unrealised Profits
The adjustment to unrealised profit should be shown as an increase to cost of sales. It affects the
books of the SELLER.

86! acowtancy.com

Downloaded by sajedul Islam (sajedulnsu15@gmail.com)


lOMoARcPSD|3181314

Intra-group trading must be eliminated from the consolidated statement of profit or loss.

Consolidated sales revenue = P’s revenue + S’s revenue – intra-group sales

Consolidated cost of sales = P’s COS + S’s COS – intra-group sales

If loans are outstanding between group companies, intra-group loan interest will be paid

and received. Both the loan and loan interest must be excluded from the consolidated

Sales and Purchases


Interest on loan
results.
Transfers of non-current assets
If one group company sells a non-current asset to another group company, the following
adjustments are needed in the statement of profit or loss:-
1. Any profit or loss arising on the transfer must be deducted
2. The depreciation charge must be adjusted so that it is based on the cost of the asset to
the group
Mid-year acquisitions
If a subsidiary is acquired part way through the year, then it is important to time apportion the
results of S in the year of acquisition. Unless indicated otherwise, assume that revenue and
expenses accrue evenly.
Lecture Example 1
In the post-acquisition period, Holdrite sold goods to Staybrite for $72,000. Holdrite achieved a
mark-up on these goods of 20% on cost. At the year end, Staybrite, still had $42,000 (at the
transfer price) of these goods in its inventory.
On 1 April 2005, Holdrite sold an item of plant to Staybrite for $120,000. Holdrite had
manufactured this plant at a cost of $100,000 and treated it as a normal sale. Staybrite is
depreciating this plant on a straight-line basis over a five-year life with no estimated residual
value.

87! acowtancy.com

Downloaded by sajedul Islam (sajedulnsu15@gmail.com)


lOMoARcPSD|3181314

On 1 October 2005, Staybrite issued a $2m 8% (actual and effective rate) loan note,
redeemable in 2010. Holdrite had subscribed for $800,000 of this issue. All due interest
had been paid by 31 March 2006.

Show how the above transactions should be accounted for in the consolidated
financial statements for the year ended 31 March 2006.

How to prepare extracts for Retained Earnings and NCI

Required:-
Example:
P controls 80% of S
RE b/fwd: P $500, S $100
RE of S at date of acquisition: $80
Consolidated Profit for the year (from Consolidated Statement of Profit or Loss): $300
Attributable to owners of P: $250
Attributable to NCI: $50

Share Capital of S at date of acquisition: $1000

Retained Earnings

Balance b/fwd (500 + 80% of post-acq profits (100 - 80)) = 516Profit for
the year (attributable to owners of P) 250less any dividends
paid by P 0 Balance c/fwd
766NCI

Bal b/fwd –
take NA of S at beg of year x NCI share (SC 1000 + RE 100)x 20% 220add Profit
attributable to NCI 50less any dividends
which are paid outside the group (not to P) 0
Balance c/fwd 270

88! acowtancy.com

Downloaded by sajedul Islam (sajedulnsu15@gmail.com)


lOMoARcPSD|3181314

Further Questions
Question 1 45
On 1 January 2014, Viagem acquired 80% of the equity share capital of Greca.
Extracts of their statements of profit or loss for the year ended 30 September 2014 are:
Viagem Greca
$’000 $’000
Revenue 64,600 38,000 Cost
of sales (51,200) (26,000)
Sales from Viagem to Greca throughout the year ended 30 September 2014 had consistently been
$800,000 per month. Viagem made a mark-up on cost of 25% on these sales. Greca had $1·5
million of these goods in inventory as at 30 September 2014.
What would be the cost of sales in Viagem’s consolidated statement of profit or loss for the year
ended 30 September 2014?
A. $59·9 million
B. $61·4 million C. $63·8 million D. $67·9 million
Question 2
Tulip Co acquired 70% of the voting share capital of Daffodil Co on 1 March 2012.
The following extracts are from the individual profit or loss statements of the two companies for
the year ended 31 August 2012:
Tulip Co Daffodil Co
$ $
Revenue 61,000 23,000
Cost of sales (42,700) (13,800) Gross
Profit 18,300 9,200
What should be the consolidated gross profit for the year ended 31 August 2012?

A. $21,520
B. $22,900
C. $27,500 D. $24,740
Question 3
Panther Co acquired 80% of the equity shares in Seal Co on 31 August 2012.

The profit or loss statements of Panther Co and Seal Co for the year ended 31 December 2012
showed:

Panther Co Seal Co
$ $

45 Specimen Exam Applicable from December 2014

89! acowtancy.com

Downloaded by sajedul Islam (sajedulnsu15@gmail.com)


lOMoARcPSD|3181314

Revenue 100,000 62,000


Cost of sales 25,000 16,000

During October 2012, sales of $6,000 were made by Panther Co to Seal Co. None of these items
remained in inventory at the year-end.

What is the consolidated revenue for Panther Group for the year ended 31 December 2012?

A. $156,000
B. $118,667
C. $144,800
D. $114,667
CHAPTER 13:
GROUP ACCOUNTING:
CONSOLIDATED STATEMENT OF PROFIT OR LOSS - ASSOCIATE
13.1 ACCA SYLLABUS GUIDE OUTCOME 1:-
Prepare consolidated financial statements to include a single subsidiary and an
associate.
The consolidated statement of profit or loss includes:
• 100% of the income and expenses of the parent and subsidiary company on a line by line
basis.
• One line ‘share of profit of associates’ which includes the group share of any
associate’s profit after tax.
Equity accounting
The equity method of accounting requires that the consolidated statement of profit or loss:
• Does not include dividends from the associate
• Instead includes group share of the associate’s profit after tax less any impairment of the
associate in the year.
Trading with the associate
Generally the associate is considered to be outside the group.
Therefore any sales or purchases between group companies and the associate are not normally
eliminated and will remain part of the consolidated figures in the statement of profit or loss.
Unrealised Profit (UP)
If the associate sells goods to the parent at a profit and some of these goods are still in inventory
at year end, it is important to calculate the share of profits of associate company as follows:-
Share of Associate Profit after Tax =

90! acowtancy.com

Downloaded by sajedul Islam (sajedulnsu15@gmail.com)


lOMoARcPSD|3181314

(% holding in A x profit of A for the year) - (% holding in A x UP) – any impairment of

Associate selling to parent company– the profit element is included in the associate

Dr Share of Associate Profit after Tax


Cr P Inventory

Dividends from associates are excluded from the consolidated statement of profit or
loss. Only the group share of the associate’s profit is included.

the investment in A
Parent company selling to associate –
Dr P Cost of sales
Cr Investment in associate
company’s accounts.
Dividends
Lecture Example 1:
Several years ago H acquired 80% of the ordinary share capital of S and 30% of A. Their results
for the year ended 31 December 2005 were as follows:
Statement of Profit or Loss

H S A
(80%) (30%)
Turnover 100 100 100
COS (40) (40) (40)
Expenses (40) (40) (40)
Profit after Tax 20 20 20

91! acowtancy.com

Downloaded by sajedul Islam (sajedulnsu15@gmail.com)


lOMoARcPSD|3181314

Prepare the consolidated statement of profit or loss for the year ended 31 December 2005.
Lecture Example 2:
Several years ago H acquired 80% of the ordinary share capital of S and 30% of A. Their results
for the year ended 31 December 2005 were as follows:
Statement of Profit or Loss
H S A

Turnover 1000 800 400


COS (600) (200) (100)
Expenses (100) (100) (40)
Tax (100) (100) (60) Profit after Tax 200 400 200
H acquired 80% of S. At that date, 3 years ago, S’s PPE had a fair value of $100 in excess of the
carrying value and a 5 year useful economic life.
Prepare the consolidated statement of profit or loss for the year ended 31 December 2005.
Lecture Example 3:
The following are the summarised accounts of H, S and A for the year ended 31 December 2009.
Statement of Profit or Loss
H S A
$ $ $
Sales Revenue 573 600 314 000 150 000
Operating costs (300 000) (200 000) (90 000)
Operating profit 273 600 114 000 60 000
Interest payable (20 000) (14 000) (8 000)
Dividend income from S 14 400
Dividend income from A 4 000
Dividend income from other sources 10 000
Profit before tax 282 000 100 000 52 000
Tax (72 000) (30 000) (16 000)

Profit for the financial year 210 000 70 000 36 000


Statements of financial position as at 31 December 2009
H S A
$ $ $
Investment in S Ltd (60%) 60 000
Investment in A Ltd (25%) 50 000
Other assets 300 000 120 000 100 000 410 000 120 000
100 000
$ $ $
Ordinary shares 20 000 30 000 10 000 Retained earnings 330
000 66 000 70 000
!92 acowtancy.com

Downloaded by sajedul Islam (sajedulnsu15@gmail.com)


lOMoARcPSD|3181314

Current liabilities 60 000 24 000 20 000


410 000 120 000 100 000
1. The shares in S and A were acquired on 1 January 2009, when the balances of the
retained earnings accounts were:
S $20 000
A $50 000
2. Goodwill in the subsidiary has suffered an impairment of 20% of its value, and the
investment in associate has suffered an impairment of $7 000.
Subsidiary goodwill impairment is recognised in operating costs and impairment of the
associate is charged against associate profits. H has accounted for the dividends from
subsidiary and associate.
3. The H group values the non-controlling interest at its proportionate share of the fair value
of the subsidiary’s identifiable net assets.
Prepare the consolidated statement of profit or loss for the year ended 31 December 2009
and the consolidated statement of financial position as at that date.
CHAPTER 14:
INVENTORIES AND BIOLOGICAL ASSETS
IAS 2:- INVENTORIES
14.1 ACCA SYLLABUS GUIDE OUTCOME 1:-
Describe and apply the principles of inventory valuation.

!93 acowtancy.com

Downloaded by sajedul Islam (sajedulnsu15@gmail.com)


lOMoARcPSD|3181314

Inventories should be measured at the lower of cost and net realisable value

The cost of inventories will consist of all the following costs: -

3. Other costs incurred in bringing the inventories to their present location and
condition, e.g. carriage inwards

3) Administration overheads
4) Selling costs

Example
14.1.1 Cost
1. Purchase
2. Costs of conversion
Do NOT include:
1) Abnormal amounts
2) Storage costs
Item A has the following costs:
Direct Labour 100
Raw Materials 200
Depreciation on production machines 10
Factory Manager wage 10
Other production Overheads 8
Admin Overheads 5
What is the cost?
Direct Labour 100 Materials 200
Depreciation (PRODUCTION) 10 Factory
Manager Wage (production) 10
Production overheads 8
328
Do not include admin costs 14.1.2
Net Realisable Value

!94 acowtancy.com

Downloaded by sajedul Islam (sajedulnsu15@gmail.com)


lOMoARcPSD|3181314

The net realisable value of an item is essentially its net selling proceeds after all costs

$
Estimated selling price X
Less: estimated costs of completion (X)
Less: estimated selling and distribution costs (X)
X

In the post balance sheet period, prior to authorising for issue the financial statements
of Tentacle for the year ended 31 March 2007, the following material information has

have been deducted.


It is calculated as:
Lecture Example 1
arisen.
Sales of some items of product W32 were made at a price of $5.40 each in April and May 2007.
Sales staff receive a commission of 15% of the sales price on this product. At 31 March 2007
Tentacle had 12,000 units of product W32 in inventory included at cost of $6 each.
Required:-
State and quantify how the item above should be treated when finalising the financial
statements of Tentacle for the year ended 31 March 2007.
(ACCA Paper 2.5 June 2007 Qs 5b (ii))
14.2 ACCA SYLLABUS GUIDE OUTCOME 2:-
Apply the requirements of relevant accounting standards for biological assets
IAS 41: Agriculture
IAS 41 Agriculture sets out the accounting for agricultural activity – it introduces a fair value
model to agriculture accounting.

!95 acowtancy.com

Downloaded by sajedul Islam (sajedulnsu15@gmail.com)


lOMoARcPSD|3181314

Living animals or plants, e.g. trees in a plantation or orchard, cultivated plants,


sheep, cattle that are in the process of growing, degenerating, regenerating and/
or procreating and which are expected to eventually result in agricultural
produce;

the harvested product obtained from a biological asset

Examples of biological assets and agricultural produce are:

Agricultural Produce

Grapes

Eggs

IAS 41 applies to: - 1.


Biological assets:

2. Agricultural produce:

Biological Asset
Vine
Chicken
Dairy cattle Milk
Sheep Wool
Pigs Meat
The standard distinguishes between two broad categories of biological assets: -
Consumable biological assets: those that are to be harvested as agricultural produce or sold as
biological assets. E.g. livestock intended for the production of meat, livestock held for sale, fish
in farms, crops such as maize and wheat.
Bearer biological assets: those other than consumable biological assets. E.g. livestock from
which milk is produced or livestock held for breeding, vines, fruit trees.
The standard does not apply to agricultural land or intangible assets related to agricultural
activity. After harvest, IAS 2 is applied.
14.2.1 Initial Recognition
An entity should recognise a biological asset or agriculture produce only when: -

!96 acowtancy.com

Downloaded by sajedul Islam (sajedulnsu15@gmail.com)


lOMoARcPSD|3181314

1. CONTROL - The entity controls the asset as a result of past events,


2. VALUE - It is probable that future economic benefits will flow to the entity, and 3.
MEASUREMENT - The fair value or cost of the asset can be measured reliably.
14.2.2 Measurement:
14.2.2.1 Biological assets
Biological assets should be measured on initial recognition and at subsequent reporting dates at
fair value less estimated point-of-sale costs, unless fair value cannot be reliably measured.
They should be classified as a separate class of assets in the SFP.
14.2.2.2 Agricultural produce
Agricultural produce should be measured at fair value less estimated point-of-sale costs at the
point of harvest. The point of harvest represents the transition between accounting for
agricultural produce assets under IAS 41 and IAS 2: - Fair value less estimated point-of-sale costs
at the point of harvest forms ‘cost’ for the purposes of IAS
2.
Agricultural produce should be classified as inventory in the statement of financial position and
disclosed separately either in the SFP or in the notes.
14.2.2.3 Point of sale costs
Point-of-sale costs include commissions to brokers and dealers, levies by regulatory agencies and
commodity exchanges, and transfer taxes and duties. Point-of-sale costs exclude transport and
other costs necessary to get assets to a market.
14.2.2.4 Fair Value
IAS 41 presumes that fair value can be reliably measured for most biological assets and
agricultural produce. However, that presumption can be rebutted when: -
1. There is lack of market-determined prices;
2. Values are not otherwise available
3. Alternative estimates of fair value are clearly unreliable
In such a case, the asset is measured at cost less accumulated depreciation and impairment losses.
This method is only allowed on initial recognition.
The entity must still measure all of its other biological assets at fair value less costs to sell. If
circumstances change and fair value becomes reliably measurable, a switch to fair value less
costs to sell is required.
14.2.3 Gains and Losses:
The gain or loss arising on initial recognition of biological assets at fair value less costs to sell is
reported as a gain or loss in the statement of profit or loss.
The change in fair value less costs to sell of a biological asset between two reporting dates is also
reported as a gain or loss in the statement of profit or loss.
A gain or loss arising on initial recognition of agricultural produce at fair value less costs to sell
should be included in net profit or loss for the period in which it arises.
14.2.4 Other issues:

!97 acowtancy.com

Downloaded by sajedul Islam (sajedulnsu15@gmail.com)


lOMoARcPSD|3181314

The change in fair value of biological assets is part physical change (growth, etc.) and part price
change.
The aggregate gain or loss is analysed as follows:
1. The price change: -

the value of the biological asset at prices prevailing as at the current reporting date less the
value of the biological asset at prices prevailing as at the previous reporting date

2. The physical change: -

the value of the biological asset in its state as at the current reporting dateless the value
of the biological asset in its state as at the previous reporting date:
Separate disclosure of the two components is encouraged but not required.
14.2.5 Government Grants:
Government grants received in respect of biological assets measured at fair value less costs to
sell are reported as income when the grant becomes receivable. If such a grant is conditional
(including where the grant requires an entity not to engage in certain agricultural activity), the
entity recognises it as income only when the conditions have been met.
Lecture Example 246

46 Article “IAS 41, Agriculture”, Simon Riley, March 2002


http://www.accaglobal.com/ie/en/student/dipifr/dipifr-resources/technical-articles/ias-41.html
!98 acowtancy.com

Downloaded by sajedul Islam (sajedulnsu15@gmail.com)


lOMoARcPSD|3181314

As at 31 December 20X1, a plantation consists of 100 Pinus Radiata trees that were planted 10

Only mature trees have established fair values by reference to a quoted price in an

active market. The fair value (inclusive of current transport costs to get 100 logs to

market) for a mature tree of the same grade as in the plantation is:

Calculate the aggregate gain or loss arising during the current period, according to IAS

years earlier. Pinus Radiata takes 30 years to mature, and will ultimately be processed into
building material for houses or furniture. The enterprise’s weighted average cost of capital is 6%
p.a.

As at 31 December 20X1: 171As at


31 December 20X2: 165 Required:
41.
CHAPTER 15:
FINANCIAL INSTRUMENTS
15.1 ACCA SYLLABUS GUIDE OUTCOME 1:-
Explain the need for an accounting standard on financial instruments
In recent years, there has been a significant growth worldwide in the number and complexity of
financial instruments in international financial markets. There were numerous concerns about the
accounting treatment of financial instruments which led to demands for an accounting standard.
There are four reporting standards that deal with financial instruments: -
• IAS 32 – Financial instruments: Presentation
• IAS 39 – Financial instruments: Recognition and Measurement
• IFRS 7 – Financial instruments: Disclosures
• IFRS 9 – Financial instruments

!99 acowtancy.com

Downloaded by sajedul Islam (sajedulnsu15@gmail.com)


lOMoARcPSD|3181314

On 12 November 2009, the IASB issued IFRS 9 Financial Instruments as the first step in its
project to replace IAS 39. IFRS 9 introduces new requirements for classifying and measuring
financial assets that must be applied starting 1 January 2013, with early adoption permitted.
On 28 October 2010, the IASB reissued IFRS 9, incorporating new requirements on accounting
for financial liabilities, and carrying over from IAS 39 the requirements for derecognition of
financial assets and financial liabilities. The IASB intends to expand IFRS 9 to add new
requirements for impairment of financial assets measured at amortised cost, and hedge
accounting.
15.2 ACCA SYLLABUS GUIDE OUTCOME 2:-
Define financial instruments in terms of financial assets and financial liabilities.
Financial Instrument - any contract that gives rise to a financial asset in one entity and a
financial liability or equity instrument of another.
Financial Asset – any asset that is
• Cash
• A contractual right to receive cash or another financial asset from another enterprise
• A contractual right to exchange financial instruments with another enterprise under
conditions that are potentially favourable
• An equity instrument of another enterprise e.g. investments in shares of another entity
Examples: - Cash, trade receivables, investment in equity shares
Financial Liability – a liability that is a contractual obligation:
• To deliver cash or another financial asset to another enterprise; or
• To exchange financial instruments on potentially unfavourable terms
Examples: - Trade payables, debenture loans, redeemable preference shares
And the trickier stuff…..IAS 39 also applies to derivatives such as call and put options,
forwards, futures, and swaps. IAS 39 does not apply to an entity’s own shares.
15.3 ACCA SYLLABUS GUIDE OUTCOME 3:-
Indicate how financial instruments should be initially measured
15.3.1 Initial Recognition of Financial Instruments
Financial assets and liabilities should be recognised on entering into the contract NOT when the
contract is settled. The asset or liability is measured at fair value – the actual transaction price on
the reporting date.
15.3.2 Derecognition of Financial Instruments
A financial asset should be derecognised (removed from the SFP) when: -
1. The contractual rights to the cash flows of the financial asset have expired or
2. The financial asset has been sold and all the risks and rewards of ownership have been
transferred from the seller to the buyer
A financial liability should be derecognised when: -
1. The obligation specified in the contract is discharged, cancelled or expires
15.4 ACCA SYLLABUS GUIDE OUTCOME 4:-

!100 acowtancy.com

Downloaded by sajedul Islam (sajedulnsu15@gmail.com)


lOMoARcPSD|3181314

Indicate for the following categories of financial instruments how they should be
measured and how any gains and losses from subsequent measurement should be treated
in the financial statements: -
i) Amortised cost
ii) Fair value through other comprehensive income (including where an irrevocable
election has been made for equity instruments that are not held for trading)
iii) Fair value through profit or loss
15.4.1 Financial Liabilities – Subsequent Measurement
After initial recognition, an entity should measure financial liabilities as follows: -
1. Liabilities that are held for trading and derivatives – re-measured to fair value at each
reporting date. Any gains and losses are included in the statement of profit or loss.
2. All other financial liabilities – measured at amortised cost using the effective interest
rate method.
Illustration 1
A company issues 5.9% loan notes at their nominal value of $1,000. The loan notes are repayable
at a premium of $250 after 5 years. The effective interest rate is 10%.
What amount will be recorded as a financial liability when the loan notes are issued?
What amounts will be shown in the Statement of Profit or Loss and SFP for years 1 – 5?
Year Op Balance Finance Cost Payment Cl Balance
10% 5.9%
1 1,000 100 (59) 1,041
2 1,041 104 (59) 1,086
3 1,086 109 (59) 1,136
4 1,136 113 (59) 1,190
5 1,190 119 (1,250 + 59) 0
When the loan notes are issued:
Dr Bank $1,000
Cr Loan notes $1,000
Statement of Profit or Loss

1 2 3 4 5
Finance
(100) (104) (109) (113) (119)
Costs
1 2 3 4 5
Non-current
liabilities
1,041 1,086 !101 1,136 acowtancy.com
Current
1,190
Downloaded by sajedul Islam (sajedulnsu15@gmail.com) 0
liabilities
lOMoARcPSD|3181314

SFP

Deep Discount
A deep discount bond is issued at a significant discount to its par value. Typically, it has a
coupon rate much lower than market rates of interest, e.g. a 2% bond when the market interest is
6% p.a.
This deep discount must simply be taken off from the opening value.
Illustration 2:
5% $1,000 loan with a deep discount of $200 creating an effective interest rate of 9%.
Year Op Balance Finance Cost Payment Cl Balance
9% 5%
1 800 72 (50) 822
2 822 74 (50) 846

15.5 ACCA SYLLABUS GUIDE OUTCOME 5:-


Explain the distinction between debt and equity capital.
Loans go to debt (liabilities); ordinary shares go to equity. Why?
It is back to the conceptual framework again and also to the important concept of substance over
form. The definition of liability includes the need for a present obligation. As interest MUST be
paid but dividends may not, only loans have this obligation and so go to liabilities.
If an entity issues preference shares that pay a fixed rate of dividend and are redeemed at a future
date (redeemable preference shares), then there is a contractual obligation to deliver cash and,
therefore, should be recognised as a liability.
In contrast, normal preference shares do not have a fixed maturity (irredeemable preference
shares), and the issuer does not have a contractual obligation to make any payment. Therefore,
they are equity.
What about new complex items such as convertible loans? These are known as compound
instruments and are partly debt and partly equity.
15.6 ACCA SYLLABUS GUIDE OUTCOME 6:-
Apply the requirements of relevant accounting standards to the issue and finance costs of
convertible debt.
15.6.1 Compound financial instruments (Convertible loans)
A compound instrument is a financial instrument that has characteristics of both equity and
liabilities, for e.g. 2% Convertible Loan $1,000.
This basically means the company has offered the bank the option to convert the loan at the end
into shares instead of simply taking $1,000.

!102 acowtancy.com

Downloaded by sajedul Islam (sajedulnsu15@gmail.com)


lOMoARcPSD|3181314

The important thing to notice is that that the bank has the option to do this. Should the share
price not prove favourable then it will simply take the $1,000 as normal.
If the bank wants cash rather than shares, it may still accept the option. The terms of conversion
are normally quite generous and the bank will accept the conversion and then sell the shares on
the market for a profit.
Better Interest rate
In exchange for these favourable terms of conversion, the bank will offer the company a
favourable interest rate compared to normal loans. Higher Fair Value of loan
This lower interest rate has effectively increased the fair value of the loan to the company. IAS
32 suggests this fair value needs to be shown immediately on inception.
Lower loan figure in SFP
Important: If the fair value of a liability has increased, the amount payable shown will be lower.
After all, fair value increases are good news and we all prefer lower liabilities!
Illustration 3
2% $1,000 loan repayable after 4 years
Normal (non-convertible) loans carry an interest rate of 5%.
The present value of $1 receivable at the end of each year, based on discount rates of 2% and 5%
are:
2% 5%
End of year 1 0·980 0·952
2 0·961 0·907
3 0·942 0·864
4 0·924 0·823
Take what the company pays and discount them using the figures above as follows:
Capital $1,000 discounted @ 5% (4 years SINGLE discount figure):
1,000 x 0.823 = 823
Interest $20 discounted @ 5% (4 years CUMULATIVE):
20 x 3.546 = 71
894
This $894 represents the fair value of the loan and this is the figure we use in the SFP initially.
The remaining $106 (1,000 - 894) goes to equity.
Op Balance Finance Cost Payment Cl Balance
894 45 -20 919
Lecture Example 1
8% Convertible loan note (2010)
On 1 April 2006 Wellmay issued an 8% convertible loan note with a nominal value of $600,000
at par. It is redeemable on 31 March 2010 at par or it may be converted into equity shares of
Wellmay on the basis of 100 new shares for each $200 of loan note. An equivalent loan note

!103 acowtancy.com

Downloaded by sajedul Islam (sajedulnsu15@gmail.com)


lOMoARcPSD|3181314

without the conversion option would have carried an interest rate of 10%. Interest of $48,000 has
been paid on the loan and charged as a finance cost.
The present value of $1 receivable at the end of each year, based on discount rates of 8% and
10% are:
8% 10%
End of year 1 0·93 0·91
2 0·86 0·83
3 0·79 0·75
4 0·73 0·68
Show the accounting treatment in the statement of profit or loss and statement of financial
position as at 31 March 2007.
(June 2007 Qs 2 Part v) 15.6.2 Interest, dividends, transaction costs
The accounting treatment of interest, dividends, losses and gains relating to a financial
instrument follows the treatment of the instrument itself.
For example, dividends paid in respect of preference shares classified as a liability will be
charged as a finance expense in the statement of profit or loss. Dividends paid on shares
classified as equity will be reported in the statement of changes in equity (SOCIE). They are
deducted from retained earnings.
Category Treatment
FVTPL Expense
FVTOCI Add to Initial Opening Balance
Amortised Cost - Asset Add to Initial Opening Balance
Amortised Cost - Liability Deduct from Initial Opening Balance
.
15.7 ACCA SYLLABUS GUIDE OUTCOME 7:-
Indicate how any gains and losses from subsequent measurement of financial assets
should be treated in the financial statements
As stated in Section 15.3.1, financial assets are initially measured at fair value – this is likely to
be the purchase consideration paid and will normally exclude transaction costs.
15.7.1 Subsequent measurement of financial assets
Subsequent measurement of financial instruments depends upon whether the financial asset is an
investment in a debt instrument or an equity instrument.
Debt instruments
Debt instruments would normally be measured at fair value through profit or loss (FVTPL) but
could be measured at amortised cost, provided these two tests are passed: -
a) Business model test

!104 acowtancy.com

Downloaded by sajedul Islam (sajedulnsu15@gmail.com)


lOMoARcPSD|3181314

The asset is held within a business model whose objective is to hold the assets to collect
the contractual cashflows
b) Contractual cash flow characteristics test
This test determines whether the contractual terms of the financial asset give rise to cash
flows on specified dates that are solely of principal and interest based upon the principal
amount outstanding.
If an asset is not held for trading AND the cash flows are principal and interest only BUT the
business model is also to sell these loans, then these debt instruments are measured at fair value
through other comprehensive income. 47 Even if a financial asset passes both tests, it is still
possible to designate a debt instrument as FVTPL if doing so eliminates or significantly reduces
an accounting mismatch.
Equity instruments
Equity instruments are measured at either: - • Fair
value through profit or loss or
• Fair value through other comprehensive income (FVTOCI) if these conditions are met: - o
The equity instrument cannot be held for trading. Hence, the intention is to hold it to
collect dividend income
o There must be an irrevocable choice for this designation upon initial recognition. In
this situation, initial recognition will also include directly attributable transaction
costs. Dividend income is still recognised in profit or loss.
Lecture Example 2
A company invests $5,000 in 10% loan notes. The loan notes are repayable at a premium after 3
years. The effective rate of interest is 12%. The company intends to hold the loan notes till
maturity and the contractual cash flows consist solely of repayments of interest and principal.
Hence, it has chosen to record the financial asset at amortised cost.
What amounts will be shown in the Statement of Profit or Loss and SFP for years 1-3?
Lecture Example 3
A company invested in 20,000 shares of a listed company in November 2010 at a cost of $5.20
per share. At 31 December 2010, the shares have a market value of $5.90.
Situation 1 – the company is planning on selling these shares in March 2011.
Situation 2 – the company is not planning on selling these shares in the short term.
For both situations, prepare extracts from the statement of profit or loss for the year ended
31 December 2010 and a SFP as at that date.
15.8 Factoring of Receivables
Factoring of receivables is where a company transfers its receivables balances to another
organisation (a factor) for management and collection and receives an advance on the value of
those receivables in return.
47 In 2014 the IASB published the complete version of IFRS 9, Financial Instruments, which replaces most of the guidance in IAS 39. It
includes amended guidance for the classification and measurement of financial assets by introducing a FVTOCI category for certain debt
instruments.

!105 acowtancy.com

Downloaded by sajedul Islam (sajedulnsu15@gmail.com)


lOMoARcPSD|3181314

The key point here is:


Have the risks of the debts been passed on to the factor?
If the entity transfers ownership of receivables to the factor, derecognise receivables from SFP.
The entity is not holding these receivables to collect the contractual cash flows but to sell them.
The only entry is:
Dr Cash
Cr Trade receivables
If the entity retains ownership of receivables, it means that the significant risks and
rewards of the receivables are not transferred. Hence, keep receivables in the entity’s
SFP and record the proceeds received from the factor as a loan:
Dr Cash
Cr Loan

When the factor collects cash from the customers:


Dr Loan
Cr Trade Receivables
If the factor returns any debts to the seller:
Dr Loan
Cr Cash
Dr Bad debts
Cr Trade Receivables
Lecture Example 4
In September 2006 Angelino sold (factored) some of its trade receivables to Omar, a finance
house. On selected account balances Omar paid Angelino 80% of their book value. The
agreement was that Omar would administer the collection of the receivables and remit a
residual amount to Angelino depending upon how quickly individual customers paid. Any
balance uncollected by Omar after six months will be refunded to Omar by Angelino.
Required:.
Describe how the above transaction should be treated in the financial statements of Angelino for
the year ended 30 September 2006.
(Paper 2.5 December 2006 Qs 3b)
15.9 Impairment of Financial Assets
Impairment loss: - The
difference between
• acquisition cost and
• current fair value (for equity instruments) or recoverable amount (for debt instruments)
15.9.1 Financial assets held at amortised cost and FVTOCI
i. Provide for expected losses

!106 acowtancy.com

Downloaded by sajedul Islam (sajedulnsu15@gmail.com)


lOMoARcPSD|3181314

On initial recognition of the asset, the entity must create a credit loss allowance. This

If, subsequent to initial recognition, credit risk increases significantly, provide an

allowance for lifetime expected credit losses. This will replace the 12 month

expected losses mentioned in (i) above.

15.9.2Financial assets carried at FVTPL

Impairment loss recognised in profit or loss for the year.

must be equal to 12 months’ expected credit losses, i.e. multiply the probability that a
default will occur in the next 12 months by the expected losses arising from this default.
ii.
i.

!107 acowtancy.com

Downloaded by sajedul Islam (sajedulnsu15@gmail.com)


lOMoARcPSD|3181314

Financial Liabilities and Equity Instruments


Issuing Financial
Instruments

Equity instruments
Financ ial liabilities

Contain an Evidence of an
obligation ownership interest
in the residual
net assets

If classified as If classified as FVTPL; Initial measurement


amortised cost; initial measurement is is at fair value less
initial measurement at fair value, and then issue costs and,
is at fair value less so is subsequent subsequently, no
issue costs and then measurement with change as equity
subsequent gains and losses being instruments are not
measurement is at recognised in the re-measured
amortised cost statement of profit or los s

Financial Assets
Financial Assets

Debt instruments Equity instruments

!108 acowtancy.com

Downloaded by sajedul Islam (sajedulnsu15@gmail.com)


lOMoARcPSD|3181314

Contain an Evidence of an
obligation ownership interest in
to be repaid the residual net asset

If classified as If classified as If classified as


amortised cost: FVTPL: initial FVTOCI: initial
initial measurement measurement is at measurement is at
is at fair value plus fair value, and then fair value plus
transaction costs and so is subsequent transaction cost and
then subsequent measurement with subsequent
measurement is at gains and losses measurement is at
amortised cost being recognised in fair value with gains
income and losses being
recognised directly
in reserves and OCI

The business model The default An irrevocable


and cash flow test accounting treatment election at inception
have to be met – also used where has to be made. On
the asset is acquired disposal no recycling
for trading purposes of previously
or to avoid an recognised gains or
accounting mismatch losses to income

Both diagrams extracted from Articles: What is a Financial Instrument? Part 1 & 2 by T.
Clendon, Student Accountant, 2012, http://www.accaglobal.com/content/dam/acca/global/PDF-
students/2012s/sa_jul12_7p2_fininstruments.pdf &
http://www.accaglobal.com/content/dam/acca/global/PDFstudents/2012s/
sa_aug12_f7p2_instruments.pdf

!109 acowtancy.com

Downloaded by sajedul Islam (sajedulnsu15@gmail.com)


lOMoARcPSD|3181314

Further Questions48
Question 1

Explain and illustrate how the issue of shares is accounted for in the financial

Laxman raises finance by issuing zero coupon bonds at par on the first day of the
current accounting period with a nominal value of $10,000. The bonds will be redeemed
after two years at a premium of $1,449. The effective rate of interest is 7%.

Explain and illustrate how the loan is accounted for in the financial statements of

Dravid issues 10,000 $1 ordinary shares for cash consideration of $2.50 each. Issue costs are
$1,000. Required statements of Dravid.
Question 2
Required Laxman.
Question 3
Broad raises finance by issuing $20,000 6% four-year loan notes on the first day of the current
accounting period. The loan notes are issued at a discount of 10%, and will be redeemed after
four years at a premium of $1,015. The effective rate of interest is 12%. The issue costs were
$1,000.
Required
Explain and illustrate how the loan is accounted for in the financial statements of Broad.
Question 4
Graham Gooch issues a 3% $200,000 two-year convertible bond at par. The effective rate of
interest of the instrument is 8%. The terms of the convertible bond is that the holder of the bond,
on the redemption date, has the option to convert the bond to equity shares at the rate of 10
shares with a nominal value of $1 per $100 debt rather than being repaid in cash. Transaction

48 Articles: What is a Financial Instrument? Part 1 & 2 by T. Clendon, Student Accountant, 2012, http://
www.accaglobal.com/content/dam/acca/global/PDF-students/2012s/sa_jul12_7p2_fin-instruments.pdf &
http://www.accaglobal.com/content/dam/acca/global/PDF-students/2012s/sa_aug12_f7p2_instruments.pdf
!110 acowtancy.com

Downloaded by sajedul Islam (sajedulnsu15@gmail.com)


lOMoARcPSD|3181314

costs can be ignored. Graham Gooch will account for the financial liability arising using
amortised cost.
Required
Explain the accounting for the issue of the convertible bond.

CHAPTER 16:
IFRS 16 – LEASING
IFRS 16 was issued in January 2016 and supersedes IAS 17, ‘Leases’.
IAS 17 required lessees to classify their leases as either finance leases or operating leases and
account for those two types of leases differently.
IFRS 16 introduces a single lessee accounting model for the lessee and requires a lessee to
recognise assets and liabilities for all leases with a term of more than 12 months, unless the
underlying asset is of low value (we will be discussing this later on.). This means that all leases
(excluding the two exceptions of less than 12 months and low value) will be showing on the SFP
as an asset and liability.
What is a lease?
A contract, or part of a contract, that conveys the right to use an asset (the underlying asset) for a
period of time in exchange for consideration.
The contract is a lease if it meets 3 criteria: -
1. The asset must be identifiable either explicitly specified in the contract (e.g. the serial
number of the asset is included) or implicitly specified (there is only one asset which is
capable of being used to meet the terms of the contract).
2. A customer is not considered to have the right to use the asset if the supplier has the
right to substitute the asset during the period of use.

!111 acowtancy.com

Downloaded by sajedul Islam (sajedulnsu15@gmail.com)


lOMoARcPSD|3181314

3. The customer has the right to control the use of an identified asset. The customer must
have both: -
a. The right to get substantially all of the benefits from using the asset
b. The right to direct the use of the asset: how and for what the asset is used.
Examples from IFRS 16
• A customer enters into a contract with an airport operator (supplier) to use a space in the
airport to sell its goods (coffee, tea, sandwiches etc.) for a threeyear period.
1. Identified asset?
No, as no area is specified in the contract. The customer may be allocated a number of areas.
2. Substantive substitution rights?
Yes and the supplier will benefit by using its retail space in the most efficient manner.
3. Customer has the right to control the use of the identified asset?
No, it is the supplier who allocates the space to the customer and obtains all of the economic
benefits from use of the concession space.
This is not a lease but is a service. The customer is purchasing space which can be changed by
the supplier.
• A customer enters into a contract with a supplier to use retail space Z (which is part of a
large shopping complex) for three years.
1. Identified asset?
Yes, specific retail space identified.
2. Substantive substitution rights?
No, although the supplier has the practical ability to substitute another retail outlet. However, it
will be required to pay for relocation costs and the circumstances, at inception date, are not
considered likely to arise.
3. Customer has the right to control the use of the identified asset?
Yes, the customer has exclusive use and the right to obtain all of the economic benefits from the
use of the retail space.
This is a lease.
Who is the lessor?
An entity that provides the right to use an underlying asset for a period of time in exchange for
consideration.
Who is the lessee?
An entity that obtains the right to use an underlying asset for a period of time in exchange for
consideration.
What is the lease term?
The lease term is the non-cancellable period for which a lessee has the right to use an underlying
asset, together with both:

!112 acowtancy.com

Downloaded by sajedul Islam (sajedulnsu15@gmail.com)


lOMoARcPSD|3181314

(b) periods covered by an option to terminate the lease if the lessee is reasonably

16.1 ACCA SYLLABUS GUIDE OUTCOME 1:-


Account for right-of-use assets and lease liabilities in the records of the lessee

At the beginning of the lease, the lessee should recognise a lease liability and a right-of-

It is an asset that represents a lessee’s right to use an underlying asset for the lease

(a) periods covered by an option to extend the lease if the lessee is reasonably certain to
exercise that option; and certain not to exercise that option.
Main Principle: -
use asset.
What is a right-of-use asset?
term.
Lease Liability
The lease liability is initially measured at the present value (PV) of the lease payments that
have not been paid. It should include: -
1. Fixed payments
2. Variable payments that depend on an index or rate
3. Residual value guarantees (i.e. the lessor is guaranteed that the underlying asset will not
be worth less than a specified amount at the end of the lease term)
4. Options to purchase the asset that are reasonably certain to be exercised
5. Termination penalties, if the lease term reflects the expectation that these will be
incurred.
To find the PV of the lease payments, we will need a discount rate, which should be the rate
implicit in the lease. If this cannot be determined, the entity should use the incremental
borrowing rate (the rate at which it could borrow funds to purchase a similar asset).
Right-of-use Asset
The right-of-use asset is initially measured at cost.
This includes: -
!113 acowtancy.com

Downloaded by sajedul Islam (sajedulnsu15@gmail.com)


lOMoARcPSD|3181314

1. The amount of the lease liability (the PV of lease payments)


2. Lease payments made at or before the lease started
3. Any initial direct costs
4. The estimated costs of removing or dismantling the underlying asset as per the
the amount of the lease liability plus any initial direct costs incurred by the lessee. Adjustments
may also be required for lease incentives, payments at or prior to commencement and restoration
obligations or similarthe amount of the lease liability plus any initial direct costs incurred by the
lessee. Adjustments may also be required for lease incentives, payments at or prior to
commencement and restoration obligations or similarconditions of the lease.
Subsequent Measurement
Lease Liability
After initial recognition, the lease liability is increased by the interest charge and reduced by the
cash payments.
It is important to establish whether the payments are being made at the start of the year (i.e. in
advance) or at the end of the year (i.e. in arrears).
Illustration 1: Payment in Arrears
• Lease term 3 years
• Rental of $5,000 paid on 31 December each year
• PV of lease payments is $12,000
• Interest rate implicit in the lease is 12.04%
Initially,
Dr Right-of-use asset $12,000
Cr Lease liability $12,000
Year Opening Finance cost (Payment) Closing
balance ($) ($) ($) balance ($)

1 12,000 1,445 -5,000 8,445


2 8,445 1,017 -5,000 4,463
3 4,463 537 -5,000 0
Finance cost of $1,445 goes to the P/L in year 1.

!114 acowtancy.com

Downloaded by sajedul Islam (sajedulnsu15@gmail.com)


lOMoARcPSD|3181314

The lease liability must be split between the amount that is to be paid within a year and

the remainder which is payable in more than one year.

$4,463

$3,982

Current vs Non-current Liability


At the end of year 1,
Non-Current Liability Lease
liability
Current Liability
Lease liability (8,445 – 4,463)
Illustration 2: Payment in Advance
• Lease term 4 years
• Rental of $8,000 payable in advance
• PV of lease payments is $28,000
• Interest rate implicit in the lease is 9.8%
Year Opening balance (Payment) ($) Balance Finance Closing
($) cost ($) balance ($)

1 28,000 -8,000 20,000 1,960 21,960


2 21,960 -8,000 13,960 1,368 15,328
3 15,328 -8,000 7,328 718 8,046
4 8,046 -8,000 0
The difference of $46 is due to rounding of interest rate.
Finance cost of $1,960 goes to P/L in year 1.
In SFP, at the end of year 1,

!115 acowtancy.com

Downloaded by sajedul Islam (sajedulnsu15@gmail.com)


lOMoARcPSD|3181314

At the end of year 1,


Non-Current Liability Lease liability
$13,960
Current Liability
Accrued interest $1,960
Lease liability (21,960 – 13,960) – 1,960 $6,040
Right-of-Use Asset
The right-of-use asset is measured using the cost model (cost less accumulated depreciation less
accumulated impairment losses) unless: -
• the asset is an investment property and the lessee applies fair value model to its
investment property; or
• the asset relates to a class of PPE for which the lessee applies the revaluation model.
Depreciation on the right-of-use asset
• If ownership of the asset is transferred to the lessee at the end of the lease term – charge
depreciation over the asset’s remaining useful economic life.
• If ownership of the asset is not transferred to the lessee at the end of the lease term –
charge depreciation on the shorter of the useful life and the lease term.
Lecture example 1
On 1 January 2017, AB Co entered into a two year lease for a machine. The contract contains
the option
Calculate thetoinitial
extend the lease
carrying amountterm for right-of-use
of the a further year. At the lease
asset and commencement
liability. date, AB Co is
reasonably certain to exercise this option. The machine has a useful economic life of eight years.
Lease payments due at the end of each year are:
Year 1 and 2: $10,000 each
Year 3: $15,000
To obtain the lease, AB Co incurred initial direct costs of $5,000. However, the lessor
Prepare extracts
reimbursed fromofAB
$3,000 Co’scosts.
these financial statements for the year ended 31 December
The interest rate implicit in the lease is not readily determinable. AB Co’s incremental
borrowing rate is 5 % p.a.
Question 1
Question 2
2017.
Question 3
the
Howend of the leasecalculated
is depreciation term? if ownership of the asset is transferred to the lessee at
Lecture example 2
On 1 October 2015 Nina Co entered into an agreement to lease a machine that had an estimated
life of four years. The lease period is also four years with annual rentals of $10,000 payable in
advance from 1 October 2015. The machine is expected to have a nil residual value at the end of
its life. The present value of the lease payments is $35,000. The interest rate implicit in the lease
is 10%.

!116 acowtancy.com

Downloaded by sajedul Islam (sajedulnsu15@gmail.com)


lOMoARcPSD|3181314

How should the lease be accounted for in the financial statements of Nina for the year ended 31
March 2017?
16.2 ACCA SYLLABUS GUIDE OUTCOME 2:-
Explain the exemption from the recognition criteria for leases in the records of the lessee.
A lessee may choose not to apply the requirements mentioned above for: -
1. Short-term leases
2. Leases for which the underlying asset is of low value
When the exemption is applied, lease payments are recognised as an expense over the lease
term on a straight-line basis or on another systematic basis, if that basis represents better the
pattern of the lessee’s benefits.
What is a short-term lease?
A short-term lease is defined as “a lease that, at the commencement date, has a lease term of 12
months or less”.
A lease that contains a purchase option cannot be classified as a short-term lease.
What is a low-value asset?
IFRS 16 does not give an explicit definition of a low-value asset. However, it states that low-
value assets are assets with a value, when new, of $5,000 or less . 49
E.g. of low-value assets are tablet and personal computers, small items of office furniture and
telephones. However, a motor vehicle would not qualify as a low-value asset even if the leased
vehicle is old at the beginning of the lease. Why? The vehicle’s value, when new, is expected to
be more than $5,000.
Illustration 3
• Lease term 5 years
• Total value of office equipment when new is $4,000
• Entity elects to apply the low-value asset exemption
• Lease payments are $1,200 for the first 2 years, $1,000 for the last 3 years
• The lessee’s benefit under the lease arises on a straight-line basis over the full lease term
Total payments = ($1,200 x 2) + ($1,000 x 3) = $5,400
Length of lease = 5 years
Lease expense recognised each year = $5,400/5 = $1,080
In the first year, there is a prepayment of $120 ($1,200 - $1,080) which is included in the SFP.
Lecture example 3
A Co leases telephones for 5 years. The total value of the telephones when new is $4,500. A
Co elects to apply the low-value asset exemption.
Lease payments are payable as follows:
Year 1 Nil : rent-free period
Years 2 & 3 $1,500 per year

49 IFRS 16, Basis for conclusion


!117 acowtancy.com

Downloaded by sajedul Islam (sajedulnsu15@gmail.com)


lOMoARcPSD|3181314

Years 4 & 5 $1,300 per year


The lessor provides a lease incentive with a value of $500.
The lessee’s benefit under the lease arises on a straight-line basis over the full lease term.
Calculate the lease expense to be recognised each year. For year 1, calculate any accruals
or prepayments relating to the lease.
16.3 ACCA SYLLABUS GUIDE OUTCOME 3:-
Account for sale and leaseback agreements.
Let’s start with an example to illustrate a sale and leaseback agreement.
P Co sells a machine to Y Co on 1 January 2017. On the same date, P Co enters into a contract
with Y Co for the right to use the asset for the next 5 years.
In this example, P is the seller of the machine but then he leases it back – he becomes a lessee.
C is the buyer but then becomes the lessor.
The important question here is this: Has a performance obligation been satisfied?
We discussed performance obligations in IFRS 15, Revenue from Contracts with Customers.
When is a performance obligation satisfied?
When the buyer obtains control of the asset, i.e. it has the ability to obtain substantially all of
the remaining benefits.
If the transfer is a sale: -
• The seller-lessee must measure the right-of-use asset at the proportion of the previous
carrying amount that relates to the rights retained.
• The profit or loss on disposal is based on the rights transferred to the buyerlessor.
If the transfer is not a sale: -
• The seller-lessee will continue to recognise the asset in its SFP
• It will recognise a financial liability (a loan) for the amount of the proceeds obtained
from the transfer.
Illustration 4
On 1 January 2017, A Co sells an item of machinery to a finance company and leases it back for
a period of four years. The remaining useful life of the asset is four years. Is this a sale?
No, A Co is retaining the asset for the remainder of its useful life. Hence, a performance
obligation has not been satisfied. The “sales proceeds” effectively represents a loan: - a financial
liability should be recognised in the books of A Co and a finance cost recognised each year in the
P/L.
Illustration 5
A seller-lessee sells an office building for its fair value of $1.8m. Its carrying amount at that
time was $1m.
On the same date, the seller-lessee leases back the building for 18 years, paying $120,000 p.a. in
arrears.
The interest rate implicit in the lease is 4.5%.
The present value of the annual payments, discounted at this rate of 4.5%, is $1,459,000.

!118 acowtancy.com

Downloaded by sajedul Islam (sajedulnsu15@gmail.com)


lOMoARcPSD|3181314

The sale of the machine has been assessed to meet the satisfaction of a performance obligation
under IFRS 15.
Hence, the transfer is a sale.
1. Recognise the right-of-use asset at the proportion of the previous carrying
amount that relates to the rights retained.
Rights retained = (1,459,000/1,800,000) x 1,000,000 = $810,556
2. Calculate the lease liability (= PV of the lease payments) These are given as
$1,459,000
Dr Cash $1,800,000
Dr Right-of-use asset $810,556
Cr Asset $1,000,000
Cr Lease liability $1,459,000
Cr Gain on sale (P/L) $151,556 (balancing figure)
The gain on sale of asset is the proportion of the total gain on disposal of $800,000($1.8m - $1m)
that relates to the rights transferred to the customer: -
(1,800,000 – 1,459,000) = 341,000/1,800,000 x 800,000 = $151,556.
The right-of-use asset and the lease liability will then be accounted for similar to the rules
described above.
Lecture example 4
On 1 January 2017, P Co sells an item of plant to Q Co for its fair value of $300,000. Prior to the
sale, the plant had a carrying value in P’s books of $120,000.
On the same date, P Co enters into a contract with Q Co to lease back the asset for the next five
years, paying $50,000 at the end of each year.
The interest rate implicit in the lease is 10%.
The sale of the asset has been assessed to meet the satisfaction of a performance obligation under
IFRS 15.
How should P Co account for this transaction on 1 January 2017?
CHAPTER 17:
IFRS 15: Revenue from Contracts with Customers
17.1 ACCA SYLLABUS GUIDE OUTCOME 1
Explain and apply the principles of recognition of revenue:
i. Identification of contracts ii. Identification
of performance obligations iii.
Determination of transaction price
iv. Allocation of the price to performance obligations
v. Recognition of revenue when/as performance obligations are satisfied
Introduction
IFRS 15, Revenue from contracts with customers, was issued in May 2014. It replaces both IAS
18, Revenue, and IAS 11, Construction Contracts. It applies to most contracts with customers.

!119 acowtancy.com

Downloaded by sajedul Islam (sajedulnsu15@gmail.com)


lOMoARcPSD|3181314

Please note the definition of revenue given in this standard and also in the Framework:
‘Revenue’ is income arising in the course of an entity’s ordinary activities.
17.2 Recognition and Measurement – The Revenue Model
17.2.1 Core Principle
An entity should recognise revenue: -
1. to record the transfer of promised goods or services to the customer
2. in an amount that reflects the consideration to which the entity expects to be entitled in
exchange for those goods or services.
This core principle is set in a five-step model framework.
17.2.2 The five-step model framework
Step 1 Identify the contract(s) with the customer
A contract with a customer will fall within the scope of IFRS 15 when all the following criteria
are met:
• The parties to the contract have approved the contract;
• Each party’s rights can be identified;
• The payment terms and conditions can be identified;
• The contract has commercial substance; and
• The collection of an amount of consideration to which the entity is entitled to in exchange
for the goods or services is probable.
A contract can be oral, written or implied.
Step 2 Identify the performance obligations in the contract
A contract includes promises to transfer the goods or services to the customer. These promises
are called performance obligations. An entity would account for a performance obligation
separately only if the promised good or service is distinct.
A good or service is distinct if the following criteria are met:
1. The customer can benefit from the good or service on its own or together with other
readily available resources;
2. The entity’s promise to transfer the good or service to the customer is separately
identifiable from other promises in the contract.
Lecture Example 150
A software developer entered into a contract with a customer to supply the following:
1. A software licence,
2. An installation service
3. Software updates for 2 years
4. Technical support, both online and by telephone, for 2 years
The developer sells the above separately. The installation service is routinely performed by other
entities and does not significantly modify the software. The software remains functional without
the updates and the technical support.

50 Adapted from, Example 11, Illustrative examples, IFRS 15


!120 acowtancy.com

Downloaded by sajedul Islam (sajedulnsu15@gmail.com)


lOMoARcPSD|3181314

Are the goods or services promised to the customer distinct in accordance with IFRS 15
‘Revenue from contracts with customers’?
Step 3 Determine the transaction price
The transaction price would be the amount of consideration that an entity expects to be entitled
to in exchange for transferring promised goods or services to a customer. If a contract contains a
variable amount, the entity will estimate the amount to which it will be entitled under the
contract. The consideration can also vary if an entity’s right to consideration is contingent on the
occurrence of a future event.
These variable contingent amounts should only be included where it is highly probable that there
will not be a reversal of revenue when any uncertainty associated with the variable consideration
is resolved.
Examples of where a variable consideration can arise: - discounts, rebates, refunds, credits, price
concessions, incentives, performance bonuses, penalties.
Lecture Example 251
PC Ltd enters in a contract with a customer to sell computers for $200 per computer on 1 January
20x5. If the customer purchases more than 1000 computers in a calendar year, the contract states
that the price per unit is retrospectively reduced to $190 per computer. As a result of this, the
consideration in the contract is variable. PC’s year end is 31 December.
As at 31 March 20x5, PC Ltd sells 80 computers to the customer. Therefore it estimates that the
customer’s purchase will not exceed the 1000 unit threshold required for the volume discount in
the calendar year.
However, at the beginning of May 20x5, the customer acquired another company and at the end
of the second quarter, 30 June 20x5, PC Ltd sells an additional 500 computers to the customer.
Now, PC Ltd estimates that the customer’s purchases will exceed the 1000 unit threshold for the
calendar year and therefore it would have to retrospectively reduce the price per unit.
Calculate the revenue PC Ltd would recognise in: -
a) The first quarter ending 31 March 20x5
b) The second quarter ending 30 June 20x5
Step 4 Allocate the transaction price to the performance obligations
Where a contract has many performance obligations, an entity shall allocate the transaction price
to the performance obligations in the contract by reference to their relative stand-alone selling
prices. If a standalone selling price is not directly observable, an entity will need to estimate it.
How?
E.g. A telephone company gives a free telephone set to customers when they sign a two-year
contract. The contract is for $50 per month and the telephone set has a standalone price of $200.
Under IAS 18 Revenue, the company would have recognised a total of $600 p.a. ($50 x 12
months) but it would not have recongised any revenue in relation to the handset.

51 Adapted from, Example 24, Illustrative examples, IFRS 15


!121 acowtancy.com

Downloaded by sajedul Islam (sajedulnsu15@gmail.com)


lOMoARcPSD|3181314

However, under IFRS 15, revenue will be allocated both to the telephone set and the contract.
Why? Because the telephone set constitutes a performance obligation.
$ %
Contract (24 months x $50) 1200 85.71
Telephone set 200 14.29
Total value 1400 100
Revenue is included as follows: -
Year 1
Telephone set (14.29% x 1200) $171.48 Contract
(1200 – 171.48 = 1028.52/2 years) $514.26
Year 2 Contract
$514.26
Sometimes the transaction price may include a discount. Any overall discount should be
allocated between the performance obligations on a relative stand-alone selling price basis. In
some circumstances, it may be appropriate to allocate the discount to some but not all of the
performance obligations.
Lecture Example 352
White Goods Ltd regularly sells washing machines, refrigerators and ovens individually, thereby
establishing the following stand-alone selling prices:
Washing Machine $400
Refrigerator $550
Oven $450
$1400
In addition, it regularly sells refrigerators and ovens together for $600.
White Goods Ltd enters into a contract with a customer to sell all three products in exchange for
$1000. It will satisfy the performance obligations for each of the products at different points in
time.
How should this transaction be accounted for?
Step 5 Recognise revenue when (or as) a performance obligation is satisfied
An entity shall recognise revenue when (or as) it satisfies a performance obligation, i.e. when
control of a promised good or service is passed to the customer, either over time or at a point in
time.
An entity recognises revenue over time if one of the following criteria is met:
a) The customer simultaneously receives and consumes the benefit provided by the entity as
the entity performs;

52 Adapted from, Example 34, Illustrative examples, IFRS 15


!122 acowtancy.com

Downloaded by sajedul Islam (sajedulnsu15@gmail.com)


lOMoARcPSD|3181314

b) The entity’s performance creates or enhances an asset that the customer controls as the
asset is created or enhanced; or
c) The entity’s performance does not create an asset with an alternative use to the entity and
the entity has an enforceable right to payment for the performance completed to date.
An entity must be able to reasonably measure the outcome of a performance obligation before
the related revenue can be recognised.
17.3 ACCA SYLLABUS GUIDE OUTCOME 2 / 3
Explain and apply the criteria for recognising revenue generated from contracts where
performance obligations are satisfied over time or at a point in time.
Describe the acceptable methods for measuring progress towards complete satisfaction
of a performance obligation.
Performance obligations satisfied over time
In this type of contract, an entity often has a right to payment for performance completed to date,
i.e. an amount that approximates the selling price of the goods or services transferred to date.
How can we measure the amount of performance completed to date?
1. Output methods: these recognise revenue on the basis of the value of goods or services
transferred to the customer, e.g. surveys of performance completed, appraisal of units
produced etc.
2. Input methods: these recognise revenue on the basis of the entity’s inputs, e.g. costs
incurred, labour hours.
Performance obligations satisfied at a point in time
If a performance obligation is not satisfied over time, it will be satisfied at a point in time, i.e. at
the point in time when the customer obtains control of the asset and the entity satisfies a
performance obligation.
Factors which may indicate that control is passed at a point in time include, but are not limited
to:
1. The entity has a present right to payment for the asset;
2. The customer has legal title to the asset;
3. The entity has transferred physical possession of the asset;
4. The customer has significant risks and rewards related to the ownership of the asset; and
5. The customer has accepted the asset.
17.4 ACCA SYLLABUS GUIDE OUTCOME 4
Explain and apply the criteria for recognition of contract costs.
Incremental costs of obtaining a contract
If the entity expects to recover incremental costs of obtaining a contract with a customer, the
entity shall recognise those costs as an asset.
What are these incremental costs?
Those costs that an entity would not have incurred if the contract had not been successfully
obtained, for example, a sales commission.

!123 acowtancy.com

Downloaded by sajedul Islam (sajedulnsu15@gmail.com)


lOMoARcPSD|3181314

If the amortisation period would be one year or less, then the entity is allowed to expense these
incremental costs.
Costs, that would have been incurred regardless of whether the contract was obtained or not, are
recognised as an expense when incurred.
Illustration 1 53
A consulting services entity, wins a competition bid to provide consulting services to a new
customer. The following costs were incurred by the entity to obtain the contract:
External legal fees for due diligence $15,000
Travel costs to deliver the proposal $25,000
Commissions paid to sales employees $10,000

Total costs incurred $50,000


The entity must recognise an asset for $10,000 commission. Why? It is an incremental cost of
obtaining the contract and the entity expects to recover these costs through future fees for
consulting services.
The legal fees and travel costs are recognised as expenses when incurred. Why? Because they
would have been incurred whether the bid was won or not.
Costs to fulfil a contract
Costs incurred to fulfil a contract with a customer are recognised as an asset only if all of the
following criteria are met:
a) The costs relate directly to an identifiable contract
b) The costs generate or enhance resources of the entity that will be used in satisfying
performance obligations in the future; and
c) The costs are expected to be recovered.
The asset shall be amortised on a systematic basis that is consistent with the pattern of
transfer of the goods or services to which the asset relates.
Illustration 2 54
An entity enters into a service contract to manage a customer’s information technology data
centre for five years. The contract is renewable for subsequent one-year periods. The average
customer term is seven years. The entity pays an employee a $10,000 sales commission upon
customer signing the contract.
Before providing the services, the entity designs and builds a technology platform for the entity’s
internal use that interfaces with the customer’s systems. That platform is not transferred to the
customer, but will be used to deliver services to the customer.
The entity recognises an asset for the $10,000 incremental costs of obtaining the contract for the
sales commission because the entity expects to recover those costs through future fees for the
services to be provided.

53 Adapted from, Example 36, Illustrative examples, IFRS 15


54 Adapted from, Example 37, Illustrative examples, IFRS 15
!124 acowtancy.com

Downloaded by sajedul Islam (sajedulnsu15@gmail.com)


lOMoARcPSD|3181314

The entity amortises the asset over seven years: - the contract term of five years + anticipation
that the contract will be renewed for two subsequent one-year periods.
Illustration 3 55
The initial costs incurred to set up a technology platform are as follows:
Design services 40,000
Hardware 120,000
Software 90,000
Migration and testing of data centre 100,000
Total costs 350,000
The initial setup costs relate primarily to activities to fulfil the contract but do not transfer goods
or services to the customer. The entity accounts for the initial setup costs as follows:
a) Hardware costs – accounted for in accordance with IAS16 Property, plant and
equipment.
b) Software costs – accounted for in accordance with IAS38 intangible assets.
c) Costs of the design, migration and testing of the data centre – we need to determine
whether an asset can be recognised for the costs to fulfil the contract. Any resulting asset
would be amortised on a systematic basis over the sevenyear period.
In addition to the initial costs to set up the technology platform, the entity also assigns two
employees who are primarily responsible for providing the service to the customer.
Although the costs for these two employees are incurred as part of providing the service to the
customer, the entity concludes that the costs do not generate or enhance resources of the entity.
Therefore, the costs cannot be recognised as an asset. They are recognised as a payroll expense
when incurred.
17.5 ACCA SYLLABUS GUIDE OUTCOME 5
Apply the principles of recognition of revenue, and specifically account for the following
types of transaction: i) principal versus agent ii) repurchase agreements iii) bill and
hold arrangements iv) consignments
17.5.1 Warranties
IFRS 15 distinguishes between two types of warranties: - 1.
Assurance type (apply IAS 37):
a. An assurance to the customer that the good or service will function as specified
b. The customer cannot purchase this warranty separately from the entity.
2. Service type (accounted for separately in accordance with IFRS 15):
a. A service is provided in addition to an assurance to the customer that the good or
service will function as specified
b. This applies regardless of whether the customer is able to purchase this warranty
separately from the entity.
In determining the classification of a warranty, an entity considers:

55 Adapted from, Example 37, Illustrative examples, IFRS 15


!125 acowtancy.com

Downloaded by sajedul Islam (sajedulnsu15@gmail.com)


lOMoARcPSD|3181314

• Legal requirements: (warranties required by law are usually assurance type)


• Length: (the longer the length of coverage, more likely additional services are being
provided)
• Nature of tasks: (do they provide a service or are they related to assurance (e.g. return
shipping for defective goods)).
Illustration 4 56
A manufacturer provides its customer with a warranty with the purchase of a product. The
warranty provides assurance that the product complies with agreed-upon specifications and will
operate as promised for one year from the date of purchase. The contract also provides the
customer with the right to receive up to 20 hours of training services on how to operate the
product at no additional cost.
1. Are the goods and services in the contract distinct?
Yes, both the product and the training services are distinct and give rise to two separate
performance obligations. The customer can benefit from the product on its own without the
training services. The manufacturer regularly sells the product separately without the training
services.
2. How should the warranty be accounted for?
The warranty provides the customer with the assurance that the product will function as intended
for one year. The warranty does not provide the customer with a good or service in addition to
that assurance. Hence, the manufacturer accounts for the assurance-type warranty in accordance
with IAS 37.
Hence, the entity allocates the transaction price to the two performance obligations, the product
and the training services, and recognises revenue when or as those performance obligations are
satisfied.
17.5.2 Principal versus agent
Is the entity acting as principal or agent?
A principal controls the promised good or service before it is transferred to the customer. When
the performance obligation is satisfied, the entity recognises revenue: the gross amount of the
consideration.
An agent arranges for the provision of goods or services by another party. Revenue will be in the
form of fees or commissions.
The following points indicate that an entity is an agent rather than a principal: -
1. Another party is responsible for fulfilling the contract
2. The entity does not have inventory risk
3. The entity does not have discretion in establishing prices for the other party’s goods or
services
4. The entity’s consideration is in the form of a commission
5. The entity is not exposed to credit risk for the amount receivable from the customer.

56 Adapted from, Example 44, Illustrative examples, IFRS 15


!126 acowtancy.com

Downloaded by sajedul Islam (sajedulnsu15@gmail.com)


lOMoARcPSD|3181314

Lecture Example 4 57
PQ Ltd operates a website that enables customers to purchase goods from a range of suppliers
who deliver the goods directly to the customers. When a good is purchased via the website, PQ is
entitled to a commission that is equal to 10% of the sales price. PQ’s website facilitates payment
between the supplier and the customer at prices that are set by the supplier.
PQ requires payment from customers before orders are processed and all orders are non-
refundable. PQ has no further obligations to the customer after arranging for the products to be
provided to the customer.
Is PQ a principal or an agent?
17.5.3 Repurchase Agreements
A repurchase agreement arises when an entity sells an asset to a customer and promises, or has
the option, to repurchase the asset.
There are 3 forms of repurchase agreements: -
1. Forward contract – an entity has an obligation to repurchase the asset
2. Call option – an entity has the right to repurchase the asset
3. Put option – an entity must repurchase the asset if requested to do so by the customer
17.5.4 Forward contract or call option
Here, the customer does not obtain control of the asset, even if it has physical possession. How
should the entity account for this contract?
1. Repurchase price is below the original selling price – record as a lease as per IAS 17
2. Repurchase price is equal or greater than the original selling price – the entity will
recognise this as a financing arrangement
Illustration 5 – Call Option58
An entity enters into a contract with a customer for the sale of a tangible asset on 1 January 2015
for $1 million
The contract includes a call option that gives the entity the right to repurchase the asset for $1.1
million on or before 31 December 2015.
Control of the asset does not transfer to the customer on 31 December 2015 because the entity
has a right to repurchase the asset. The customer is limited in its ability to use the asset.
Consequently, the entity accounts for the transaction as a financing arrangement, because the
exercise price ($1.1m) is more than the original selling price ($1m).
The entity does not derecognise the asset and instead recognises the cash received as a financial
liability. The entity also recognises interest expense for the difference between the exercise price
($1.1 million) and the cash received ($1 million), which increases the liability
Dr Cash Cr
Loan (SFP)
Every year:

57 Adapted from, Example 45, Illustrative examples, IFRS 15


58 Adapted from, Example 62 Case A, Illustrative examples, IFRS 15
!127 acowtancy.com

Downloaded by sajedul Islam (sajedulnsu15@gmail.com)


lOMoARcPSD|3181314

Record interest as finance cost. Increase value of loan


Dr Finance costs (P/L)
Cr Loan (SFP)
On 31 December 2015, the option lapses unexercised; therefore, the entity derecognises the
liability and recognises revenue of $1.1 million
17.5.5 Put Option
If the entity must repurchase the asset if requested to do so by the customer, it must consider
whether or not the customer is likely to exercise that option.
1. Repurchase price is lower than the original selling price –
Does the customer have significant economic incentive to exercise the option?
Yes: entity should account for the agreement as a lease as per IAS 17 No: contract
accounted for as an outright sale, with a right of return.
2. If the repurchase price is greater than or equal to the original selling price and above the
expected market value of the option – treat as financing arrangement
Illustration 6 – Put Option59
From previous illustration...
Instead of having a call option, the contract includes a put option that obliges the entity to
repurchase the asset at the customer’s request for $900,000 on or before 31 December 2015. The
market value is expected to be $750,000 on 31 December 2015.
At the inception of the contract, the entity assesses whether the customer has a significant
economic incentive to exercise the put option. The entity concludes that the customer has a
significant economic incentive to exercise the put option because the repurchase price
significantly exceeds the expected market value of the asset at the date of repurchase.
The entity concludes that control of the asset does not transfer to the customer, because the
customer is limited in its ability to direct the use of, and obtain substantially all of the remaining
benefits from, the asset.
The entity accounts for the transaction as a lease in accordance with IAS 17 Leases.
17.5.6 Consignment Arrangements
An entity may deliver a product to another party, e.g. dealer or retailer. The main question here
is:
Has this party obtained control of the product?
1. No
If this party has not obtained control, the product may be held in a consignment arrangement.
• No revenue is recognised upon delivery
• Inventory not included in the other party’s SFP
• Any deposit paid included under ‘other receivables’
2. Yes
If control has been transferred to the other party,

59 Adapted from, Example 62 Case B, Illustrative examples, IFRS 15


!128 acowtancy.com

Downloaded by sajedul Islam (sajedulnsu15@gmail.com)


lOMoARcPSD|3181314

• In the other party’s SFP, recognise inventory as an asset, together with a corresponding
liability to the manufacturer
• Deduct any deposit paid from the liability. Any excess is classified as a trade payable.
The following indicate the existence of a consignment arrangement: -
a) The product is controlled by the entity until a specified event occurs (e.g. sale of the
product to a customer of the dealer or retailer, or until a specified period expires)
b) The entity can require the return of the product, or transfer it to another party
c) The dealer or retailer does not have an unconditional obligation to pay for the product
Lecture Example 5 60
Angelino is a motor car dealer selling vehicles to the public. Most of its new vehicles are
supplied on consignment by two manufacturers, Monza and Capri, who trade on different terms.
Monza supplies cars on terms that allow Angelino to display the vehicles for a period of three
months from the date of delivery or when Angelino sells the cars on to a retail customer if this is
less than three months. Within this period Angelino can return the cars to Monza or can be asked
by Monza to transfer the cars to another dealership (both at no cost to Angelino). Angelino pays
the manufacturer’s list price at the end of the three month period (or at the date of sale if sooner).
In recent years Angelino has returned several cars to Monza that were not selling very well and
has also been required to transfer cars to other dealerships at Monza’s request.
Capri’s terms of supply are that Angelino pays 10% of the manufacturer’s price at the date of
delivery and 1% of the outstanding balance per month as a display charge. After six months (or
sooner if Angelino chooses), Angelino must pay the balance of the purchase price or return the
cars to Capri. If the cars are returned to the manufacturer, Angelino has to pay for the
transportation costs and forfeits the 10% deposit. Because of this Angelino has only returned
vehicles to Capri once in the last three years.
Required:
Describe how the above transactions and events should be treated in the financial statements of
Angelino for the year ended 30 September 2006.
17.5.7 Bill-and-Hold Arrangements
Bill-and-hold arrangements involve the seller invoicing a customer for a product but, instead of
delivering it to the customer, the seller retains physical possession with the product being
delivered to the customer at a later date, e.g. the customer does not have enough storage space.
When has the customer obtained control of the product?
Is it when the goods have been delivered to the customer? In some circumstances, a customer
may obtain control even though the goods remain in the entity’s possession.
For a customer to have obtained control of a product in a bill-and-hold arrangement, all these
criteria must be met: -
a) The reason for the bill-and-hold arrangement must be substantive (e.g. requested by the
customer due to lack of physical space to store the goods)

60 ACCA Paper 2.5 December 2006 Q3


!129 acowtancy.com

Downloaded by sajedul Islam (sajedulnsu15@gmail.com)


lOMoARcPSD|3181314

b) The product must be identified separately as belonging to the customer


c) The product must currently be ready for physical transfer to the customer
d) The entity cannot have the ability to use the product or to transfer it to another customer
Illustration 7 61
An entity enters into a contract with a customer on 1 January 2014 for the sale of a machine and
spare parts. The delivery time for the machine and spare parts is two years.
Upon completion of manufacturing, the entity demonstrates that the machine and spare parts
meet the agreed-upon specifications in the contract. The promises to transfer the machine and
spare parts are distinct and result in two performance obligations that each will be satisfied at a
point in time.
On 31 December 2015, the customer pays for the machine and spare parts, but only takes
physical possession of the machine. Although the customer inspects and accepts the spare parts,
the customer requests that the spare parts be stored at the entity’s warehouse because of its close
proximity to the customer’s factory.
The customer has legal title to the spare parts and the parts can be identified as belonging to the
customer. Furthermore, the entity stores the spare parts in a separate section of its warehouse and
the parts are ready for immediate shipment at the customer’s request. The entity expects to hold
the spare parts for two to four years and the entity does not have the ability to use the spare parts
or direct them to another customer.
The entity identifies the promise to provide custodial services as a performance obligation
because it is a service provided to the customer and it is distinct from the machine and spare
parts.
1. The entity accounts for three performance obligations in the contract: the promises to
provide: - • the machine
• the spare parts
• the custodial services
2. The transaction price is allocated to the three performance obligations
3. Revenue is recognised when (or as) control transfers to the customer
• Control of the machine is transferred to the customer on 31 December 2015 when the
customer takes physical possession.
• It recognises revenue for the spare parts on 31 December 2015 when control transfers to
the customer.
• The performance obligation to provide custodial services is satisfied over time as the
services are provided. The entity considers whether the payment terms include a
significant financing component: - interest expense.
17.6 ACCA SYLLABUS GUIDE OUTCOME 6
Prepare financial statement extracts for contracts where performance obligations are
satisfied over time.

61 Adapted from, Example 63, Illustrative examples, IFRS 15


!130 acowtancy.com

Downloaded by sajedul Islam (sajedulnsu15@gmail.com)


lOMoARcPSD|3181314

17.6.1Performance obligations satisfied over time

Where performance obligations are satisfied over time, an entity must determine what
amounts to include as revenue and costs in each accounting period. These should be
recorded in profit or loss as the contract activity progresses.

Contract Price 1,000

Estimated total costs 800

Costs incurred to date 600

Value of performance obligations satisfied


700

Amount invoiced and paid up 600

Lecture Example 6
The company calculates satisfaction of performance obligations based on work certified as a
percentage of contract price.
Required:-
Calculate the effect of the above contract on the financial statements.
Construction contracts affect the following parts of the statement of profit or loss and SFP:
Statement of Profit or Loss
Sales x

!131 acowtancy.com

Downloaded by sajedul Islam (sajedulnsu15@gmail.com)


lOMoARcPSD|3181314

COS (x)

Profit x

Contract Asset(presented separately under current assets)

x/(x)

(x)

Contract asset (amount due from customer) x

If the net amount above is negative, that shows a net amount due to the customer.
Hence it is a contract liability, which is included separately under current liabilities.
Receivables (under current assets)
SFP
Costs to date
Profits/(Losses) to date
Less amount invoiced
Contract Liability
This represents amounts invoiced but not yet paid up.
Important points: -
Remember that
1. IFRS 15 states that an entity’s right to payment for performance completed to date should
approximate the selling price of the service completed to date. The selling price would be a
recovery of costs incurred plus a reasonable profit margin.
2. Where no profit can be estimated, revenue is limited to recoverable costs.
3. Where a loss is anticipated, a proportion of the entity’s costs will not be recovered, and this
needs to be recognised.
Lecture Example 7
$
Contract Price 1,000
Estimated total costs 800
Costs incurred to date 600
Value of performance obligations satisfied 700
Amount invoiced and paid up 600

!132 acowtancy.com

Downloaded by sajedul Islam (sajedulnsu15@gmail.com)


lOMoARcPSD|3181314

The company calculates satisfaction of performance obligations based on the proportion

Calculate the effect of the above contract on the financial statements.

Contract Price 1,000

Costs incurred to date 500

Estimated costs to complete the contract


300

Amounts invoiced and paid up 540

Estimated % of obligations satisfied 60%

of costs incurred.
Required:- Lecture
Example 8
Required:-
Calculate the effect of the above contract on the financial statements.
Lecture Example 9 - Loss-Making Contract
The first thing you should actually do is check to see if the contract still looks like it is going to
be profitable. If it isn’t you must bring the FULL loss in immediately.

!133 acowtancy.com

Downloaded by sajedul Islam (sajedulnsu15@gmail.com)


lOMoARcPSD|3181314

Contract Price 1,000

Costs incurred to date 550

Estimated costs to complete the contract


550

Amounts invoiced 475

Amounts paid up 400

Estimated % of obligations satisfied 60%

Calculate the effect of the above contract on the financial statements.

Where a contract is already part way through

Where a contract is already part way through, i.e. in its second year, some revenue and
costs have previously been recognised. Therefore, it is important to take this into
account in the calculations to make sure they show the current year revenue and costs.
Required:-
1) Do contract for year 2 as normal.
2) Take away the Sales and COS figures that were in Yr 1 on the statement of profit or loss.
FURTHER EXAMPLES62
EXAMPLE 1
Profit-making contract
Lily is a construction company that prepares its financial statements to 31 December each year.
During the year ended 31 December 2008, the company commenced a contract that is expected
to take more than one year$000to complete. The contract summary at 31 December 2008 is as
follows: 1,400
Progress payments 2,736
Contract price
1,824
Work certified complete
Contract costs incurred to
31 December 2008 2,160
Estimated total cost at
31 December 2008* 2,520
62 Examples 1 and 2 are extracted from the article “Construction Contracts” by B. Retallack, Student Accountant,
November 2008. These have been adapted to IFRS 15. http://www.accaglobal.com/content/dam/acca/global/PDF-
*students/2012/sa_novdec08_retallack.pdf
The examiner sometimes presents information in this manner – ‘estimated total cost’
means costs incurred plus costs to complete. !134 acowtancy.com

The company calculates satisfaction of performance


Downloaded by sajedul Islamobligations based on the work
(sajedulnsu15@gmail.com)
lOMoARcPSD|3181314

Required:-
Calculate the effect of the above contract on the financial statements at 31 December 2008.
Statement of Profit or Loss extract – 31 December 2008
$000
Revenue (work certified) 1,824
COS (ß) 1,680
Gross profit (W2) 144
Asset on a construction contract (W3) 904 Statement of
financial position
extract – 31
December 2008
Current assets
WORKINGS:-
(W1) Expected
$000 outcome
2,736 Contract price
Total costs
2,520
Expected profit
216 (W2) Percentage
of completion

Accounting policy = Work certified complete

Work certified to date Contract


price
1,824 = 66.67%
2,736
(As a round percentage was not found, use the fraction to complete workings instead)
Profit to be recognised = $216 (W1) x $1,824 /$2,736 = $144
(W3) Asset on construction contract
Costs incurred to date 2160
Profit recognised to date 144
Less: Progress payments (1400)
904
EXAMPLE 2
Loss-making contract

!135 acowtancy.com

Downloaded by sajedul Islam (sajedulnsu15@gmail.com)


lOMoARcPSD|3181314

Gladioli is a construction company that prepares its financial statements to 31 March each year.
The contract summary at 31 March 2008 is as follows:

$000
3,780
4,500
Contract costs incurred to 31 March 2008 3,600
Estimated cost to complete at
1,200

The performance obligation satisfied is calculated using the proportion of costs incurred

Calculate the effect of the above contract in the financial statements at 31 March

During the year ended 31 March 2008, the company commenced a contract that is expected to
take more than one year to complete.
Amounts invoiced and paid up
Contract price 31
March 2008
method. Required:-
2008.
Solution 1
Statement of Profit or Loss extract – 31 March 2008
$000
Revenue (0.75 x 4500) 3,375
COS 3,675
Gross loss (W1) (300)

Statement of financial position extract – 31 March 2008


Current liabilities
Liability on a construction contract 480
WORKINGS:-
(W1) Expected outcome

!136 acowtancy.com

Downloaded by sajedul Islam (sajedulnsu15@gmail.com)


lOMoARcPSD|3181314

$000
4,500
(4,800) Contract price
(300)

(W2) Liability on construction contract

3,600
(300)
(3,780)
(480)

Costs to date
Total estimated costs
=

3600 = 75%

Total cost (3,600 + 1,200) Expected


loss
Costs incurred to date
Loss recognised to date
Less: Progress payments Liability on
construction
(W3) Percentage of completion
4800
CHAPTER 18:
IAS 37 – PROVISIONS AND CONTINGENCIES
18.1 ACCA SYLLABUS GUIDE OUTCOME 1:-
Explain why an accounting standard on provisions is necessary.
The objective of IAS 37 is to ensure that:
1. appropriate recognition criteria and measurement bases are applied to provisions,
contingent liabilities and contingent assets and
2. sufficient information is disclosed in the notes to the financial statements to enable users
to understand their nature, timing and amount.
18.2 ACCA SYLLABUS GUIDE OUTCOME 2:-
State when provisions may and may not be made and demonstrate how they should be
accounted for.
A provision is a liability of uncertain timing or amount.
IAS 37 requires a provision be recognised when all of the following apply:
1. an entity has a present obligation (legal or constructive) as a result of a past event

!137 acowtancy.com

Downloaded by sajedul Islam (sajedulnsu15@gmail.com)


lOMoARcPSD|3181314

2. it is probable that an outflow of resources embodying economic benefits will be required


to settle the obligation
3. a reliable estimate can be made of the amount of the obligation
Double entry
Dr Expense Cr Provision
(Liability SFP)
If it is part of a cost of an asset (e.g. Decommissioning costs ) 63
Dr Asset
Cr Provision (Liability SFP)
18.3 ACCA SYLLABUS GUIDE OUTCOME 3:-
Distinguish between legal and constructive obligations.
A legal obligation is an obligation that derives from:
• a contract
• legislation
• other operation of law.
A constructive obligation arises if past practice creates a valid expectation on the part of a third
party, for example, a retail store that has a long-standing policy of allowing customers to return
merchandise within, say, a 30-day period.
18.4 ACCA SYLLABUS GUIDE OUTCOME 4:-
Explain how provisions should be measured64.
The amount recognised as a provision should be the best estimate of the expenditure required to
settle the present obligation at the end of the reporting period.
Provisions for one-off events (restructuring, environmental clean-up, settlement of a lawsuit) are
measured at the most likely amount.
Provisions for large populations of events (product warranties, customer refunds) are measured at
a probability-weighted expected value. Provisions should be discounted.
Example
A future liability of $1,000 in 2 years time (discount rate 10%)
1,000 x 1/1.10 x 1/1.10 = $826
Dr Expense $826
Cr Provision $826
Then the discount is unwound
Year 1
$826 x 10% = $83
Dr Interest $83
Cr Provision $83
Year 2

63 To remove something such as a ship, nuclear power station, machinery, or weapons from service
64 Examined June 2014 Qs 5ii
!138 acowtancy.com

Downloaded by sajedul Islam (sajedulnsu15@gmail.com)


lOMoARcPSD|3181314

(826 + 83) x 10% $91 = $91


Dr Interest $91
Cr Provision

!139 acowtancy.com

Downloaded by sajedul Islam (sajedulnsu15@gmail.com)


lOMoARcPSD|3181314

18.5 ACCA SYLLABUS GUIDE OUTCOME 5:-


Identify and account for:
i) warranties/guarantees ii)
onerous contracts
iii) environmental and similar provisions iv)
provisions for future repairs or refurbishments
Circumstance Accrue a Provision?
Accrue a provision (past event was the sale of
Warranties/guarantees defective goods)
Accrue if the established policy is to give refunds
Customer refunds

Onerous (loss-making) contract Accrue a provision

Accrue a provision if the company's policy is

to clean up even if there is no legal


Land contamination requirement to do so

Future operating losses No provision (no present obligation)


No provision (there is no obligation to provide the
Firm offers staff training training)
Major overhaul or repairs No provision (no obligation)
Restructuring by sale of an Accrue a provision only after a binding sale
operation/line of business agreement

!140 acowtancy.com

Downloaded by sajedul Islam (sajedulnsu15@gmail.com)


lOMoARcPSD|3181314

Accrue a provision only after a detailed formal


Restructuring by closure of business
plan is adopted and announced publicly. A Board
locations or reorganisation
decision is not enough
In the case of a restructuring, it is important to provide only for costs necessarily entailed by the
restructuring.
18.6 ACCA SYLLABUS GUIDE OUTCOME 6:-
Define contingent assets and liabilities and describe their accounting treatment.
Contingent liabilities are:
(a) possible obligations that arise from past events and whose existence will be confirmed
only by the occurrence or nonoccurrence of one or more uncertain future events not
wholly within the control of the entity i.

(b)present obligations that arise from past events but are not recognised because:
they are not probable that an outflow of resources embodying economic
benefits will be required to settle the obligation; or
ii. the amount of the obligation cannot be measured with sufficient reliability

Contingent liabilities should not be recognized in financial statements but they should be
disclosed, unless the possibility of any outflow is remote.

The required disclosures are:


• A brief description of the nature of the contingent liability
• An estimate of its financial effect

!141 acowtancy.com

Downloaded by sajedul Islam (sajedulnsu15@gmail.com)


lOMoARcPSD|3181314

#
Contingent assets are possible assets that arise from past events and whose existence will be
confirmed only by the occurrence or nonoccurrence of one or more uncertain future events not
wholly within the control of the entity.
A contingent asset must not be recognized. Only when the realization of the related economic
benefits is virtually certain should recognition take place. At that point, the asset is no longer a
contingent asset!
Contingent assets must only be disclosed in the notes if they are probable. A brief description of
the contingent asset must be provided together with an estimate of its financial effect and details

Virtually Probable Possible Remote


certain
Liability Create a Contingent -
provision liability
(disclose)
Asset Create an asset Contingent - -
asset
(disclose) of

any uncertainties.

!142 acowtancy.com

Downloaded by sajedul Islam (sajedulnsu15@gmail.com)


lOMoARcPSD|3181314

Lecture Example 1
In the post balance sheet period, prior to authorising for issue the financial statements of Tentacle
for the year ended 31 March 2007, the following material information has arisen.
Tentacle is being sued by an employee who lost a limb in an accident while at work on 15 March
2007. The company is contesting the claim as the employee was not following the safety
procedures that he had been instructed to use. Accordingly the financial statements include a
note of a contingent liability of $500,000 for personal injury damages. In a recently decided case
where a similar injury was sustained, a settlement figure of $750,000 was awarded by the court.
Although the injury was similar, the circumstances of the accident in the decided case are
different from those of Tentacle’s case. Required:-
ACCA Paper 2.5 June 2007 Qs 5b (iii)
Further questions
Question 1
State and quantify how the item above should be treated when finalising the
Which of the following is not essential for recognition as a liability :
A. There
financial should beofaTentacle
statements present obligation
for the yeartoended
transfer economic
31 March benefit
2007.
B. The obligating event triggering the obligation should have already happened
C. It should be possible to make a reliable estimate of the amount
D. The obligation to transfer economic benefit should be certain
Question 2
Which of the following statements is correct :
A. When an asset is acquired a provision will need to be made for all repairs needed during its
use
B. There is no need to account for a liability if the amount is not known for certain
C. The amount of audit fees remaining unpaid may be referred to as a Provision
D. A liability with uncertainty regarding eighter its timing or amount may be called a Provision
Question 3
Which of the following statements is incorrect :
A. A liability is a present obligation, arising from past transaction, which probably has to be
paid
B. If the occurrence of the obligation is in doubt there is no need to account for the
liability
C. Damages awarded by court against the business may be ignored because i twill be appealed
D. A liability has to be accounted for at the best reliable estimate even if the amount is not
certain
Question 4
Which of the following requirements in IAS 37 are calculated to ensure that Provisions are not
created and used to eighter understate or overstate the performance and net assets of a business:
i) A provision cannot be made for anticipated future losses ii) A provision is defined
as a liability with uncertainity in timing or amount iii) A provision cannot be used for any

!143 acowtancy.com

Downloaded by sajedul Islam (sajedulnsu15@gmail.com)


lOMoARcPSD|3181314

purpose other than what it was intended for iv) The need for & amount of provision shall
be reviewed on every reporting date
A. All four
B. ii, iii and iv
C. i, ii and iii
D. i and ii only
Question 5
A manufacturer received a claim for $200,000 as damages because some products were not
delivered in time. The manufacturer admits the delay but disputes the amount of the claim
pointing out that alternative products cost no more than $28,000. How should the manufacturer
treat the claim when finalising the financial statements for the year?
A. Account for a liability of $28,000
B. Account for a liability of $200,000
C. Account for the best estimate as liability
D. Ignore the claim
Question 6
On 1 June 2014 AB Printers signed a contract with B plc for $24 million. In terms of this
contract B plc is required to supply 80,000 boxes of paper per year for three years from 1 July
2014. By 31 December, B plc has supplied 32,000 boxes and has sent an invoice for $8 million.
AB Printers has made no payment to B plc. How should AB Printers show the amount owed to B
plc as at 31 December 2014 ?
A. $3.2 million as a liability
B. $2.4 million as a liability
C. $3.2 million as a provision
D. $8 million as a liability
Question 7
P plc is facing a claim for $500,000 from a customer, in respect of a fire due to defective electric
circuit in a kitchen appliance sold by P. The legal opinion is that the claim will succeed ; but P
will be able to re-claim 80% of the amount from F plc who supplied the malfunctioned circuit-
breakers. How should P plc report this on its SFP ?
A. $100,000 as a provision
B. $500,000 as provision and $400,000 as asset
C. $500,000 as a liability and $400,000 as asset
D. $100,000 as a liability
Question 8
A contractor finds that, due to the negligence of his employees, he has to replace the tiles in a
shop. The replacement is expected to cost $5,000; but it is anticipated that the damaged tiles may
fetch around $1,000. At what amount should the contractor account for his obligation to replace
the tiles on his year-end SFP ?

!144 acowtancy.com

Downloaded by sajedul Islam (sajedulnsu15@gmail.com)


lOMoARcPSD|3181314

Which of the following statements is correct

A.A contingent liability should always be disclosed in a note


B.A contingent liability may be accounted for if its occurrence is certain
C. A contingent liability should be disclosed as a note, unless its occurrence is remote
D. A contingent liability may always be ignored

Which of the following should be disclosed as a note to financial statements as a

A. $5,000 as a liability and $1,000 as an asset


B. $4,000 as a liability
C. $4,000 as a provision
D. $5,000 as a liability
Question 9
Question 10
contingent liability ?
A. A customer’s claim which, according to legal opinion, has a 75% chance of success
B. A customer’s claim that is unlikely to succeed
C. Damages awarded against a company by court, for wrongful dismissal of an employee
D. A customer’s claim for injuries suffered on company premises with 5% chance of success
CHAPTER 19:
IAS 12 – TAXATION
19.1 ACCA SYLLABUS GUIDE OUTCOME 1:-
Account for current taxation in accordance with relevant accounting standards.
From an accounting point of view, tax is an informed guess made at the year end. By the time it
is actually paid the real figure may be more or less than what we put in our accounts.
A debit balance on the trial balance for tax means that there has been an under provision for tax
last year and thus needs adding to this year’s current tax.
A credit balance on the trial balance for tax means that there has been an over provision for tax
last year and thus needs reducing from this year’s current tax.
Therefore, each year, the statement of profit or loss will show either an over or under provision
from the previous year as well as the current tax guess for that year.

!145 acowtancy.com

Downloaded by sajedul Islam (sajedulnsu15@gmail.com)


lOMoARcPSD|3181314

Lecture Example 1
Flora, a limited liability company, shows an over provision of $3,400 on its tax liability account
at the end of the year ended 31 December 20X8 before accounting for that year’s tax charge.
It estimates tax on profits for the year to be $67,900.
What amounts should be shown in the financial statements for the year ended 31 December
20X8 in respect of tax?
Lecture Example 2
Sasha has estimated its income tax liability for the year ended 31 December 2009 at $180,000.
Extract from the trial balance as at 31 December 2009
Dr Cr
$ $
Income tax 3,000
Show the income tax expense for the year ended 31 December 2009 and the liability for
income taxes in the SFP at that date.
19.2 ACCA SYLLABUS GUIDE OUTCOME 2:-
Explain the effect of taxable temporary differences on accounting and taxable profits.
Deferred Tax Liability
A deferred tax liability is the amount of income tax payable in future periods in respect of
taxable temporary differences. In simple terms, deferred tax is tax that is payable in the future.
This is caused by differences between IFRS rules and Tax rules (tax base), also known as taxable
temporary differences.
Temporary Differences
Temporary differences are defined as being differences between the carrying amount of an asset
(or liability) within the SFP and its tax base, i.e. the amount at which the asset (or liability) is
valued for tax purposes by the relevant tax authorities.
A typical example behind deferred tax in Paper F7 is depreciable non-current assets.
Within financial statements, non-current assets with a limited economic life are subject to
depreciation. However, within tax computations, non-current assets are subject to capital
allowances (also known as tax depreciation) at rates set within the relevant tax legislation.
Where at the year end, the cumulative depreciation charged and the cumulative capital
allowances claimed are different, the carrying value of the asset (cost less accumulated
depreciation) will then be different to its tax base (cost less accumulated capital allowances) and
hence a taxable temporary difference arises.
19.3 ACCA SYLLABUS GUIDE OUTCOME 3:-
Compute and record deferred tax amounts in the financial statements.
Tax Base
Let’s presume in one country’s tax law royalties receivable are only taxed when they are
received.
IFRS

!146 acowtancy.com

Downloaded by sajedul Islam (sajedulnsu15@gmail.com)


lOMoARcPSD|3181314

IFRS, on the other hand, recognises them when they are receivable.
Now let’s say in year 1, there are $1,000 royalties receivable but not received until year 2.
The statement of profit or loss would show:
Royalties Receivable $1000
Tax 0 (They are taxed when received in yr 2)
This does not give a faithful representation as we have shown the income but not the related tax
expense.
Therefore, IFRS actually states that matching should occur so this tax needs to be brought into
year 1.
Dr Tax (I/S)
Cr Deferred Tax (SFP provision)
So, basically deferred tax is caused simply by timing differences between IFRS rules and tax
rules.
Therefore IFRS demands that matching should occur i.e.:
Difference Tax effect Deferred Tax Double entry
More Income in I/S More tax needed Liability Dr Tax (I/S)
Cr Def Tax
Liability (SFP)

Hopefully you can see then that the opposite also applies:
Difference Tax effect Deferred Tax Double entry
More expense in I/S Less tax needed Asset Dr Def tax asset Cr
Tax (I/S)
In fact, the following table all applies:
Difference Tax effect Deferred Tax
1 More Income More tax Liability
2 Less income Less tax Asset
3 More expense Less tax Asset
4 Less expense More tax Liability

!147 acowtancy.com

Downloaded by sajedul Islam (sajedulnsu15@gmail.com)


lOMoARcPSD|3181314

Difference Tax effect Deferred Tax

1 More Income More tax Liability

2 Less income Less tax Asset

3 More expense Less tax Asset

4 Less expense More tax Liability

Remember this “more income etc” is from the point of view of IFRS, i.e.: The accounts
are showing more income, as the taxman does not tax it until next year.

We will now look at each of these 4 cases in more detail.

Case 1

Issue
IFRS shows more income than the taxman has taken into account.
Example
Royalties receivable above.
Double entry required
Dr Tax (I/S)
Cr Deferred tax Liability (SFP)

!148 acowtancy.com

Downloaded by sajedul Islam (sajedulnsu15@gmail.com)


Difference Tax effect
lOMoARcPSD|3181314

Deferred Tax
1 More Income More tax Liability
2 Less income Less tax Asset
3 More expense Less tax Asset
4 Less expense More tax Liability

IFRS shows less income than the taxman has taken into account.

Taxman taxes some income which IFRS states that this should be deferred, such as
upfront receipts on a long term contract.

Double entry required


Case 2
Issue
Example
Dr Deferred Tax Asset (SFP)
Cr Tax (I/S)
This will have the effect of eliminating the tax charge for now, so matching the fact that IFRS is
not showing the income yet either.
Once the income is shown, then the tax will also be shown by:
Dr Tax (I/S)
Cr Deferred tax asset (SFP)
Case 3
Difference Tax effect Deferred Tax
1 More Income More tax Liability
2 Less income Less tax Asset
3 More expense Less tax Asset
4 Less expense More tax Liability
IFRS shows more expense than the taxman has taken into account.

Issue
Example
Double entry
required
IFRS depreciation is more than Tax depreciation (WDA or CA)

!149 acowtancy.com
Deferred Tax Asset (SFP) 100
Downloaded by sajedul Islam (sajedulnsu15@gmail.com)
100
lOMoARcPSD|3181314

Dr Cr Tax
(I/S)
Illustration
IFRS TAX

Asset Cost 1,000 1,000


Depn (400) (300)
NBV 600 700
Simply compare 700 – 600 = 100
Case 4
Difference Tax effect Deferred Tax
1 More Income More tax Liability
2 Less income Less tax Asset
3 More expense Less tax Asset
4 Less expense More tax Liability
IFRS shows less expense than the taxman has taken into account.
Issue
Example
Double entry
required
Dr Tax I/S
IFRS depreciation is less than Tax depreciation (WDA or CA)
Cr Deferred
Tax Liability
Illustration
IFRS TAX

Asset Cost 1,000 1,000


Depn (300) 100 (400)
NBV 700 100 600

Simply compare 700 – 600 = 100


NOTE
In actual fact, the standard refers to assets and liabilities rather than more income and more
expense etc.
Simply use the above tables and substitute the word asset for income and expense for liability.
Difference Tax effect Deferred Tax
1 More Income More tax Liability

!150 acowtancy.com

Downloaded by sajedul Islam (sajedulnsu15@gmail.com)


lOMoARcPSD|3181314

2 Less income Less tax Asset


3 More expense Less tax Asset
4 Less expense More tax Liability

Possible Examination examples of Case 1 & 4


• Accelerated capital allowances (accelerated tax depreciation) See above
• Interest revenue
Some interest revenue may be included in profit or loss on an accruals basis, but taxed
when received.
• Revaluations to fair value
Revaluations of NCA are a further example of a taxable temporary difference. When a
NCA is revalued to its current value within the financial statements, the revaluation
surplus is recorded in equity (in a revaluation reserve) and reported as other
comprehensive income. While the carrying value of the asset has increased, the tax base
of the asset remained the same and so a temporary difference arises.
Tax will become payable on the surplus when the asset is sold and so the temporary
difference is taxable. Since the revaluation surplus has been recognized within equity, to
comply with matching, the tax charge on the surplus is also charged to equity.
NOTE: Double entry here is
Dr Revaluation Reserve with the tax (as this is where the “income” went)
Cr Deferred tax liability (SFP)
Difference Tax effect Deferred Tax
1 More Income More tax Liability
2 Less income Less tax Asset
3 More expense Less tax Asset
4 Less expense More tax Liability

!151 acowtancy.com

Downloaded by sajedul Islam (sajedulnsu15@gmail.com)


lOMoARcPSD|3181314

Possible Examination examples of Case 2 & 3

Provisions may not be deductible for tax purposes until the expenditure is

Current losses can be carried forward to be offset against future taxable profits.
This will result in a deferred tax asset.

73

A non-current asset costing $2,000 was acquired at the start ofyear 1. It is being
depreciated straight line over four years.
The capital allowances granted on this asset are:

• Provisions
incurred.
• Losses

Lecture Example 3
$
Year 1 800
Year 2 600
Year 3 360
Year 4 240
Calculate the tax charged to the statement of profit or loss in each of the four years and the
tax liability at the end of each year. Assume the tax rate is 25%.

73
Article, “Deferred Tax”, S. Baker and T.Clendon, Student Accountant, August 2009
http://www.accaglobal.com/content/dam/acca/global/pdf/sa_aug09_baker_clendon.pdf
Lecture Example 4
Trial Balance (extract) as at 31 March 2012
Dr Cr
$ $
Income tax 120 Deferred tax
9,000
• The estimated income tax on the profits for the year to 31 March 2012 is $11,500
• During the year, $10,300 was paid in full and final settlement of income tax on the profits
for the year ended 31 March 2011. The statement of financial position at 31 March 2011
had included $10,180 in respect of this liability.

!152 acowtancy.com

Downloaded by sajedul Islam (sajedulnsu15@gmail.com)


lOMoARcPSD|3181314

• At 31 March 2012, the carrying amounts of the net assets of Delta exceeded their tax base
by $38,000.
• The rate of income tax in this jurisdiction is 25%.
Calculate the tax charged to the statement of profit or loss and the tax liability at 31 March
2012.
Lecture Example 5
Tadeon trial balance as at 30 September 2006:
Deferred tax balance 1 October 2005 $12,000
The directors have estimated the provision for income tax for the year ended 30 September 2006
at $38 million.
At 30 September 2006 there were $74 million of taxable temporary differences, of which $20
million related to the revaluation of the leasehold property. The income tax rate is 20%.
Calculate the tax charged to the statement of profit or loss and the tax liability at 30
September 2006.
(Paper 2.5 Dec 2006 Qs 2 Part)
CHAPTER 20:
IAS 10 – EVENTS AFTER THE REPORTING PERIOD
20.1 ACCA SYLLABUS GUIDE OUTCOME 1:-
Distinguish between and account for adjusting and non-adjusting events after the
reporting date.
Identify items requiring separate disclosure, including their accounting treatment and
required disclosures
“Events after the reporting period” are those events, both favourable and unfavourable, that occur
between the end of the reporting period and the date when the financial statements are authorised
for issue”.
Two types of events can be identified:
a. those that provide evidence of conditions that existed at the end of the reporting period
(adjusting events); and
b. those that are indicative of conditions that arose after the end of the reporting period
(non-adjusting events).
20.1.1 Examples of adjusting events given in IAS 10 are:
a. the resolution of a court case, as the result of which a provision has to be recognised
instead of the disclosure by note of a contingent liability;
b. evidence of impairment of assets e.g. property;
c. bankruptcy of a major customer;
d. sale of inventories at prices suggesting the need to reduce the figure in the SFP to the net
value actually realized (inventories sold at a loss);
e. discovery of fraud or errors that show the financial statements were incorrect.
20.1.2 Examples of non-adjusting events given in IAS 10 are:

!153 acowtancy.com

Downloaded by sajedul Islam (sajedulnsu15@gmail.com)


lOMoARcPSD|3181314

a. decline in market value of investments;


b. announcement of a plan to discontinue part of the enterprise;
c. major purchases and sales of assets;
d. destruction of a major asset by fire etc;
e. sale of a major subsidiary;
f. major dealings in the company's ordinary shares;
20.1.3 Further provisions covered by IAS 10:
a. Authorisation for issue of financial statements
An enterprise should disclose the date when the financial statements were authorised for issue
and who gave that authorisation. If the owners or others have the power to amend the financial
statements after issue, that fact should be disclosed.
b. Going concern
If the management decides after the end of the reporting period that it is necessary to liquidate
the enterprise, the financial statements should not be prepared on a going concern basis.
c. Dividends
If an entity declares dividends after the reporting period, the entity shall not recognise those
dividends as a liability at the end of the reporting period. That is a non-adjusting event.
Lecture Example 1
In the post balance sheet period, prior to authorising for issue the financial statements of Tentacle
for the year ended 31 March 2007, the following material information has arisen.
The notification of the bankruptcy of a customer. The balance of the trade receivable due from
the customer at 31 March 2007 was $23,000 and at the date of the notification it was $25,000.
No payment is expected from the bankruptcy proceedings.

Required:-
State and quantify how the item above should be treated when finalising the financial
statements of Tentacle for the year ended 31 March 2007.
ACCA Paper 2.5 June 2007 Qs 5b (i)
Lecture Example 2
Triangle, a public listed company, is in the process of preparing its draft financial statements for
the year to 31 March 2005. The following matters have been brought to your attention:
On 15 May 2005 the company’s auditors discovered a fraud in the material requisitions
department. A senior member of staff who took up employment with Triangle in August 2004
had been authorising payments for goods that had never been received. The payments were made
to a fictitious company that cannot be traced. The member of staff was immediately dismissed.
Calculations show that the total amount of the fraud to the date of its discovery was $240,000 of
which $210,000 related to the year to 31 March 2005. (Assume the fraud is material).
Required:-

!154 acowtancy.com

Downloaded by sajedul Islam (sajedulnsu15@gmail.com)


lOMoARcPSD|3181314

Explain how this item above should be treated in Triangle’s financial statements for the
year to 31 March 2005 in accordance with current international accounting standards.
Your answer should quantify the amounts where possible.
ACCA Paper 2.5 June 2005 Qs 5 (ii)
Lecture Example 3
At 30 September 2003 Bowtock had included in its draft balance sheet inventory of $250,000
valued at cost. Up to 5 November 2003, Bowtock had sold $100,000 of this inventory for
$150,000. On this date new government legislation (enacted after the year end) came into force
which meant that the unsold inventory could no longer be marketed and was worthless.
Bowtock is part way through the construction of a housing development. It has prepared its
financial statements to 30 September 2003 in accordance with IAS 11 ‘Construction Contracts’
and included a proportionate amount of the total estimated profit on this contract. The same
legislation referred to above (in force from 5 November 2003) now requires modifications to the
way the houses within this development have to be built. The cost of these modifications will be
$500,000 and will reduce the estimated total profit on the contract by that amount, although the
contract is still expected to be profitable.
Required:-
Assuming the amounts are material, state how the information above should be reflected in
the financial statements of Bowtock for the year ended 30 September 2003.
ACCA Paper 2.5 December 2003 Qs 5c (ii)
Further Questions65
Question 1
Which TWO of the following events which occur after the reporting date of a company but
before the financial statements are authorised for issue are classified as ADJUSTING events in
accordance with IAS 10 Events after the Reporting Period?
(i) A change in tax rate announced after the reporting date, but affecting the current tax
liability
(ii) The discovery of a fraud which had occurred during the year
(iii) The determination of the sale proceeds of an item of plant sold before the year end
(iv) The destruction of a factory by fire
A. (i) and (ii)
B. (i) and (iii)
C. (ii) and (iii)
D. (iii) and (iv)
Question 2
Isaac is a company which buys agricultural produce from wholesale suppliers for retail to the
general public. It is preparing its financial statements for the year ending 30
September 2014 and is considering its closing inventory.

65 Specimen Exam Applicable from December 2014


!155 acowtancy.com

Downloaded by sajedul Islam (sajedulnsu15@gmail.com)


lOMoARcPSD|3181314

In addition to IAS 2 Inventories, which of the following IFRSs may be relevant to determining
the figure to be included in its financial statements for closing inventories?
A. IAS 10 Events After the Reporting Period
B. IFRS 5 Revenue from contracts with customers
C. IAS 16 Property, Plant and Equipment
D. IAS 41 Agriculture
Question 3
The events below took place between the reporting date (30.06.14) and the date the financial
statements were authorised for issue(31.08.14). Are the following events adjusting or non-
adjusting events?
ii) Payment was received from a customer whose debt has already been written off

iii) In a re-organisation the company has abandoned one of its main production lines

iv)Received information of an accident that occurred in May 2014 and destroyed

i) The rate of exchange of the country of the main supplier has become very unfavourable
A. Adjusting event
B. Non-adjusting event
A. Adjusting event
B. Non-adjusting event
A. Adjusting event
B. Non-adjusting event
most of the inventory
A. Adjusting event B.
Non-adjusting event

!156 acowtancy.com

Downloaded by sajedul Islam (sajedulnsu15@gmail.com)


lOMoARcPSD|3181314

v) Received claim for compensation from an employee who was wrongly dismissed in April
2014
A. Adjusting event B.
Non-adjusting event
vi) Company made a right issue
A. Adjusting event
B. Non-adjusting event
vii) Inventory, reported at cost at reporting date, is found to be defective and cannot be sold
A. Adjusting event B.
Non-adjusting event
viii) The court awarded substantial damages to a customer whose claim was not expected to
succeed.
A. Adjusting event
B. Non-adjusting event

CHAPTER 21:
IFRS 5 – NON-CURRENT ASSETS HELD FOR SALE AND DISCONTINUED
OPERATIONS
21.1 ACCA SYLLABUS GUIDE OUTCOME 1:-
Discuss the importance of identifying and reporting the results of discontinued
operations.
An item of PPE becomes subject to the provisions of IFRS 5 (rather than IAS 16) if it is
classified as held for sale. This classification can either be made for a single asset (where the
planned disposal of an individual and fairly substantial asset takes place) or for a group of assets
(where the disposal of a business component takes place).
An analysis between continuing and discontinuing operations tends to improve the usefulness of
the financial statements. It is important to know which parts of it are continuing their operations
and those which have ceased or been sold or are about to be in the near future. Only the results
of continuing operations should be used in forecasting future results; profits or losses from
discontinuing operations will not be repeated.
!157 acowtancy.com

Downloaded by sajedul Islam (sajedulnsu15@gmail.com)


lOMoARcPSD|3181314

For example, if a group made a large profit from one of its subsidiaries that it has recently sold
(or will soon sell), this will have a material effect on any forecast of the group’s future profit.
This is because the profits from the subsidiary disposed of will no longer contribute to future
group profit. Also, the converse would be true where the disposal or closure of a loss-making
subsidiary could improve future profitability. 66
Information on discontinued operations can also help to assess management’s strategy. One
would expect loss-making activities to be sold or closed down, but selling a profitable activity
may indicate that a company has liquidity or debt problems.
21.2 ACCA SYLLABUS GUIDE OUTCOME 2:-
Define and account for non-current assets held for sale.
21.2.1 When is an asset held for sale?
An asset is classified as held for sale if its carrying amount will be recovered principally through
a sale transaction rather than through continued use.
1. management is committed to a plan to sell
2. the asset is available for immediate sale in its present condition
3. an active programme to locate a buyer is initiated
4. the sale is highly probable, within 12 months of classification as held for sale
5. the asset is being actively marketed for sale at a sales price reasonable in relation to its
fair value
21.2.2 Measurement
Immediately before the initial classification
The carrying amount of the asset will be measured in accordance with applicable IFRSs.
Generally, bring depreciation up to date (if cost model followed) or revalue (if revaluation policy
followed).
After classification as held for sale
When non-current assets or disposal groups are classified as held-for-sale, they are measured at
the lower of the carrying amount and fair value less costs to sell.
If fair value less costs to sell is below the current carrying value, then the asset is written down to
fair value less costs to sell and an impairment loss recognized. Any impairment loss that arises
by using the measurement principles in IFRS 5 must be recognised in profit or loss, even for
assets previously carried at revalued amounts.
Revalued assets will need to deduct costs to sell from their fair value and this will result in an
immediate charge to profit or loss.
Subsequent increases in fair value
A gain is recognised in the statement of profit or loss up to the amount of all previous
impairment losses. No depreciation
Non-current assets or disposal groups that are classified as held for sale shall not be depreciated
as its carrying value will be recovered principally through sale rather than continuing use.

66 June 2013 Qs 4a
!158 acowtancy.com

Downloaded by sajedul Islam (sajedulnsu15@gmail.com)


lOMoARcPSD|3181314

Presentation on the Statement of Financial Position


When an asset is classified as held for sale, IFRS 5 requires that it be moved from its existing
presentation on the SFP (non-current assets) to a new category (under current assets) of the SFP
– ‘non-current assets held for sale’.
If the criteria for classifying a non-current asset as held-for-sale occur after the end of the
reporting period, then the non-current asset should not be shown as held-for-sale but disclosure
of the fact in the notes should be made.
When a held for sale asset is sold
When the asset is sold, any difference between the new carrying value and the net selling price is
shown as a profit or loss on sale.
Change of plans
If criteria for an asset to be classified as held-for-sale are no longer met, then the asset or disposal
group ceases to be held-for-sale.
In this case, it should be valued at the lower of the carrying amount before the asset or
disposal group was classified as held-for-sale (as adjusted for any subsequent depreciation,
amortisation or re-valuation), and its recoverable amount at the date of the decision not to
sell. Any adjustment to the value should be shown in income from continuing operations for the
period.
Lecture Example 167
An asset has a carrying value of $600,000. It is classified as held for sale on 30 September 20X6.
At that date its fair value less costs to sell is estimated at $550,000. The asset was sold for
$555,000 on 30 November 20X6. The year end of the entity is 31 December 20X6.
1. How would the classification as held for sale, and subsequent disposal, be treated in the
20X6 financial statements?
2. How would the answer differ if the carrying value of the asset at 30 September 20X6 was
$500,000, with all other figures remaining the same?
Lecture Example 268
An asset being classified as held for sale is currently carried under the revaluation model at
$600,000. Its latest fair value is $700,000 and the estimated costs of selling the asset are $10,000.
Show how this transaction would be recorded in the financial statements.
21.2.3 Subsidiaries Held for Disposal
IFRS 5 applies to accounting for an investment in a subsidiary held only with a view to its
subsequent disposal in the near future.
Subsidiaries already consolidated now held for sale
The parent must continue to consolidate such a subsidiary until it is actually disposed of. It is not
excluded from consolidation and reported as an asset held for sale under IFRS 5.
67 Article, “Property, Plant and Equipment and Tangible Assets”, by P. Robins, Student Accountant, August 2007
http://www.accaglobal.com/content/dam/acca/global/pdf/sa_aug07_robins.pdf
68 Article, “Property, Plant and Equipment and Tangible Assets”, by P. Robins, Student Accountant, August 2007
http://www.accaglobal.com/content/dam/acca/global/pdf/sa_aug07_robins.pdf
!159 acowtancy.com

Downloaded by sajedul Islam (sajedulnsu15@gmail.com)


lOMoARcPSD|3181314

21.3 ACCA SYLLABUS GUIDE OUTCOME 3:-


Define and account for discontinued operations.
Classification
A discontinued operation is a component of an entity that either has been disposed of or is
classified as held for sale, and:
1. represents a separate major line of business or geographical area of operations,
2. is part of a single co-ordinated plan to dispose of a separate major line of business or
geographical area of operations, or
3. is a subsidiary acquired exclusively with a view to resale and the disposal involves loss of
control.
Statement of Profit or Loss and other Comprehensive Income Presentation
The profit after tax of the discontinued operation and the post-tax gain or loss recognised on the
measurement to fair value less cost to sell or fair value adjustments on the disposal of the assets
(or disposal group) should be presented as a single amount on the face of the statement of
profit and loss and other comprehensive income.
Detailed disclosure of revenue, expenses, pre-tax profit or loss, and related income taxes is
required either in the notes or on the face of the statement of profit or loss in a section distinct
from continuing operations.
Statement of Cash Flow Presentation
The net cash flows attributable to the operating, investing, and financing activities of a
discontinued operation shall be separately presented on the face of the statement of cash flow
or disclosed in the notes.
No Retroactive Classification
IFRS 5 prohibits the retroactive classification as a discontinued operation, when the discontinued
criteria are met after the end of the reporting period.

!160 acowtancy.com

Downloaded by sajedul Islam (sajedulnsu15@gmail.com)


lOMoARcPSD|3181314

Illustration
An entity plans to dispose of a group of net assets which form a disposal group. The net assets at 31
December 2006 are set out below.
Carrying value at 31
Dec 2006 $m
Goodwill 16
Property, plant and
28
equipment
Inventory 20
Financial assets (profit of
$4m recognised in 17
equity)
Financial liabilities (14)
67

The property, plant and equipment and inventory were stated at deemed cost on moving
to IFRS. Under IFRS, property, plant and equipment would be stated at $26m, and
inventory stated at $18m. The fair value less costs to sell of the disposal group is $47m.
Assume that the disposal group qualifies as held-for-sale.

Show how the disposal group would be accounted for in the financial statements for the
year ended 31 December 2006.

Answer

Fair value less


Carrying value Re- measured Impairment
costs to sell
$m $m $m
$m
Goodwill 16 16 (16) –
Property, plant
28 26 26
and equipment
Inventory 20 18 18
Financial
17 17 17
assets
211!

acowtancy.com

Downloaded by sajedul Islam (sajedulnsu15@gmail.com)


lOMoARcPSD|3181314

Financial
(14) (14) (14)
liabilities
67 63 (16) 47
IFRS 5 requires that immediately before the initial classification of the disposal group as held-
for-sale, the carrying amounts of the disposal group be measured in accordance with applicable
IFRS, and any profit or loss dealt with under that IFRS. The reduction in the carrying amount of
property, plant and equipment will be dealt with in accordance with IAS 16, and that of the
inventory in accordance with IAS 2.
After the re-measurement, the entity will recognise an impairment loss of $16m on
remeasurement to the lower of carrying amount and fair value less cost to sell. This loss is
allocated to goodwill in accordance with IAS 36 . Thus, goodwill will be reduced to 69zero. The
loss will be charged against profit or loss.
In the SFP, the major classes of assets and liabilities classified as held-for-sale should be
separately disclosed on the face of the SFP or in the notes. Thus, in this case, there would be
separate disclosure of the disposal group as follows.
$m
Assets
Non-current assets
Current assets
Non-current assets and current assetsclassified as
61
held-for-sale (note)

Equity and liabilities


Equity attributable to parent
Amounts recognised directly in equity relating to non-current
assets held-for-sale
Minority interest
Total equity

Non-current liabilities
Current liabilities
69 The impairment loss is allocated in the following order:
Liabilities directory associated with non-current
• 14 unit (group of units); and
first, reduce the carrying amount of any goodwill allocated to the cash-generating
assets classified as held-for-sale
• then, reduce the carrying amounts of the other assets of the unit (group of units) pro rata on the basis.
Total liabilities !162 acowtancy.com

Total equity and liabilitiesDownloaded by sajedul Islam (sajedulnsu15@gmail.com)


lOMoARcPSD|3181314

Lecture Example 3
Partway is in the process of preparing its financial statements for the year ended 31 October
2006. The company’s main activity is in the travel industry mainly selling package holidays
(flights and accommodation) to the general public through the Internet and retail travel agencies.
During the current year the number of holidays sold by travel agencies declined dramatically and
the directors decided at a board meeting on 15 October 2006 to cease marketing holidays through
its chain of travel agents and sell off the related high-street premises. Immediately after the
meeting the travel agencies’ staff and suppliers were notified of the situation and an
announcement was made in the press.
The directors wish to show the travel agencies’ results as a discontinued operation in the
financial statements to 31 October 2006. Due to the declining business of the travel agents, on 1
August 2006 (three months before the year end) Partway expanded its internet operations to offer
car hire facilities to purchasers of its internet holidays.
The following are Partway’s summarised Statement of Profit or Loss results – years ended:
31 Oct 2006 31 Oct 2005
Internet Travel Agencies Car Hire Total Total
$’000 $’000 $’000 $’000 $’000
Revenue 23,000 14,000 2,000 39,000 40,000
Cost of sales (18,000) (16,500) (1,500) (36,000) (32,000)
Gross profit/(loss) 5,000 (2,500) 500 3,000 8,000
Operating expenses (1,000) (1,500) (100) (2,600) (2,000)
Profit/(loss) before tax 4,000_ (4,000) 400 400_ 6,000_
The results for the travel agencies for the year ended 31 October 2005 were: revenue $18 million,
cost of sales $15 million and operating expenses of $1.5 million.
Required:-
1. Discuss whether the directors’ wish to show the travel agencies’ results as a
discontinued operation is justifiable.
2. Assuming the closure of the travel agencies is a discontinued operation, prepare the
(summarised) statement of profit or loss of Partway for the year ended 31 October
2006 together with its comparatives.
Note: Partway discloses the analysis of its discontinued operations on the face of its statement of
profit or loss.
(ACCA Paper 2.5 December 2006 Qs 5b)
CHAPTER 22:
PREPARING THE ACCOUNTS OF A SINGLE ENTITY: - HOW TO ANSWER THIS
QUESTION
This question will ask you to prepare a set of accounts (normally a Statement of Profit or Loss
and Other Comprehensive Income, Statement of Financial Position (SFP) and statement of
changes in equity (SOCIE)).

!163 acowtancy.com

Downloaded by sajedul Islam (sajedulnsu15@gmail.com)


lOMoARcPSD|3181314

The examiner normally gives you a trial balance and some adjustments to make, which
you then do and prepare the accounts.

Otherwise, he will give you a set of accounts already prepared and just give you a list of

The most common is the trial balance.

Take the trial balance figures and put them immediately into a statement of profit or loss
and other comprehensive income and SFP.

adjustments to make to these.


Step 1
Leave lots of space for adjustments.
Leave the SOCIE until later.
Step 2
Do the following common adjustments:
• Taxation
• Interest
• Dividends
• Simple depreciation
22.1 Current Tax
As discussed in the chapter “Taxation”, tax, from an accounting point of view, is an informed
guess made at the year end. By the time it is actually paid the real figure may be more or less
than what we put in our accounts.
Therefore, each year, the statement of profit or loss will show either an over or under provision
from the previous year as well as the current tax guess for that year.
In the trial balance given to you if there is an amount for taxation given – this will be the under or
over provision from last year.

!164 acowtancy.com

Downloaded by sajedul Islam (sajedulnsu15@gmail.com)


If it is a debit it is an under provision (interest expense last year was lower than it should
lOMoARcPSD|3181314

have been) and so needs adding to this year’s current tax.

If it is a credit it is an over provision (interest expense last year was higher than it should
have been) and so wants reducing from this year’s current tax.

Dr Cr

Tax 10

1) Current tax for the year is 100


Illustration
Trial Balance (extract) Notes to
the question:
Answer:
Step 1 TB into accounts
I/S Tax (+10
Step 2 Do adjustment
I/S Tax (+10 + 100)
SFP Tax payable 100
If the figure in the trial balance was a credit – then that means it was an over provision in the
previous year and so we need to take this away from this year’s figure in the statement of
comprehensive income.
The answer would be:
I/S Tax (-10 + 100) = 90 SFP
Tax Payable 100
22.2 Deferred Tax
The trial balance will show a deferred tax balance – this either goes to assets (if dr) or liabilities (if
cr) on the SFP.
All you need to do now is account for the movement between this opening balance and the closing
balance given to you as an adjustment.
Illustration
Trial Balance (extract)
Dr Cr
Notes to the question:
1) Deferred tax is to be 100
Answer:tax
Deferred 80
Step 1 TB into accounts
SFP Deferred tax Liability 80
Step 2 Do adjustment
!165 acowtancy.com

Downloaded by sajedul Islam (sajedulnsu15@gmail.com)


lOMoARcPSD|3181314

I/S Tax (+20)


SFP Deferred Tax Liability (80 + 20) 100
Deferred tax is calculated as:
Tax rate x Timing differences
Illustration
There are $1,000 timing differences caused by accelerated capital allowances. Tax rate 30%.
Solution
Any deferred tax caused by a timing difference on a revaluation – the deferred tax goes
to the SFP as normal but the other side is NOT to the statement of profit or loss. It
reduces the revaluation surplus instead.

Dr Cr

80
300

1) The revaluation caused a timing difference of 300 (Tax 30%).

Deferred Tax is to be 300


NOTE
Illustration
Trial Balance (extract)
Deferred tax
Revaluation reserve Notes to
the question:
Answer:
Step 1 TB into accounts
SFP Deferred tax Liability 80
Revaluation Reserve 300
Step 2 Do adjustment
SFP Deferred Tax Liability (80 + 10) 90
Revaluation reserve (300 – 10) 290
N.B.: Deferred tax = 300 x 30% = 90 (b/f 80) so movement is 10.

!166 acowtancy.com

Downloaded by sajedul Islam (sajedulnsu15@gmail.com)


lOMoARcPSD|3181314

Comprehensive example
1) The revaluation caused a timing difference of 300 (Tax 30%). Trial Balance
2) Tax for year is estimated to be 200. (extract)
Dr
Cr
Tax
10

Deferred Tax Liability (70 + 20) 90


Revaluation reserve (300 – 20) 280
Deferred tax 70 Revaluation reserve
300
Notes to the question:
Answer:
Step 1 TB into accounts
I/S Tax (+10
SFP Deferred tax Liability 70
Revaluation Reserve 300
Step 2 Do adjustment
I/S Tax (+10 + 200) 210
SFP Tax Payable 200

22.3 Interest Adjustment


Issue
Simply ensure the correct amount payable is in the accounts regardless of what has been paid.
An 8% 1,000 Loan – should show $80 interest in I/S
An 8% 1,000 Loan for 9 months = $80 x 9/12 in I/S
Illustration
Trial Balance (extract) Dr Cr
Interest 120
8% Loan 2,000

!167 acowtancy.com

Downloaded by sajedul Islam (sajedulnsu15@gmail.com)


lOMoARcPSD|3181314

Answer:
Step 1 TB into accounts
I/S Interest (120
SFP 8% Loan 2,000
Step 2 Do adjustment
I/S Interest (120 + 40) = 160
SFP Interest Accrual +40
8% Loan 2,000
Note Interest = 2,000 x 8% = 160
NOTE – Effective Interest Rates
If you are given a loan with an effective interest rate and a paid (coupon) interest rate – always use
the effective rate to calculate the interest expense.
The interest due this time, though, does not go to accruals but gets added onto the loan.
Illustration
Trial Balance (extract) Dr Cr
Interest 120 2% Loan
2,000
Note to Question
The loan has an effective interest rate of 10%.
Answer:
Step 1 TB into accounts
I/S Interest (120
SFP 2% Loan 2,000
Interest (120 + 80) = 200 (10% of $2000) Step 2 Do
SFP 2% Loan 2,000 + 80 = 2,080 adjustment
I/S
22.4 Dividend
Adjustment
Double Entry
Dr Retained
Earnings
Presentation

Cr Dividend Payable Sometimes in the


question, the

Share Capital Share Premium Retained Earnings


B/f X X X
In the Statement of Changes in Equity
Issue of shares X X
Profit after tax !168 acowtancy.com
X
Dividends (X)
C/f X x (sajedulnsu15@gmail.com)
Downloaded by sajedul Islam X
lOMoARcPSD|3181314

double entry is not needed as the examiner will tell you that is already accounted for correctly.
Therefore, you simply have to show it in the SOCIE.
Calculation
Number of shares x Dividend per share
Illustration
A dividend of 0.10 per share. This has been accounted for correctly.

Number of shares x Dividend per share


? x 0.10

The number of shares is calculated by taking the share capital figure and dividing it by
the nominal value – given in brackets after the share capital on the TB.

Share Capital Share Premium Retained Earnings


B/f X X X
Issue of shares X X
Profit after tax X
Dividends -200
C/f X X X
Trial Balance (extract) Dr Cr
Share Capital (@ $1) 2,000
Share Premium 500
Note to Question
Answer
2,000 / 1 x 0.1 = 200
This is shown in the SOCIE
Illustration
Trial Balance (extract) Dr Cr
Share Capital (@ $0.5) 2,000 Share
Premium 500

!169 acowtancy.com

Downloaded by sajedul Islam (sajedulnsu15@gmail.com)


lOMoARcPSD|3181314

Note to Question
A dividend of 0.10 per share. This has
been accounted for correctly.
Answer
2,000 / 0.5 x 0.1 = 400

!170 acowtancy.com

Downloaded by sajedul Islam (sajedulnsu15@gmail.com)


lOMoARcPSD|3181314

22.5 The Financial Statements


One of the statements introduced by IAS 1 (revised) is the statement of profit or loss and other
comprehensive income. This statement presents all items of income and expense recognized in
profit or loss together with all other items recognized in income and expense. Entities may
present all items together in a single statement or present two linked statements – one displaying
the items of income and expense recognised in the statement of profit or loss and the other
statement beginning with profit or loss and displaying all the items included in ‘other
comprehensive income’.
Therefore, whereas the statement of profit or loss includes all realised gains and losses (e.g. net
profit for the year), the statement of profit or loss and other comprehensive income would
include both the realised and unrealised gains and losses (e.g. revaluation surplus).
Proforma 1: One single statement
31 March 20X8

Revenue
Statement of Profit or Loss and Other Comprehensive Income for the year ended X
Cost of sales (X)
Gross profit X Other income X
Distribution costs (X) 20X8 (X) Administrative expenses
20X7
(X) (X) $’000 $’000
Finance costs X (X) (X)
Investment income (X) X X
Profit before tax X X X
Income tax expense X (X) (X)
Profit for the year X X
Other comprehensive income:
Gains on property revaluation X X
Total comprehensive income for the year X X
Proforma 2: Two separate statements
Statement of Profit or Loss for the year ended 31 March 20X8
20X8 20X7
$’000 $’000
Revenue X X

!171 acowtancy.com

Downloaded by sajedul Islam (sajedulnsu15@gmail.com)


lOMoARcPSD|3181314

Cost of sales (X) (X)


Gross profit X X
Other income X X
Distribution costs (X) (X)
(X) (X)
(X) (X)
X
X
(X)
X

Statement of comprehensive income for the year ended 31 March 20X8

20X8
$’000 $’000
X

X
Income tax relating to components of other
comprehensive income X
Total comprehensive income for the year X X
Administrative expenses
Finance costs
Investment income X
Profit before tax X
Income tax expense (X)
Profit for the year X
20X7
Profit for the year X
Other comprehensive income:
Gains on property revaluation X
X
Statement of financial position as at 31 March 20X8

$'000
ASSETS
Non-current assets
Property, plant and equipment X
Other intangible assets X
X Current
assets
Inventories X Trade receivables X

!172 acowtancy.com

Downloaded by sajedul Islam (sajedulnsu15@gmail.com)


lOMoARcPSD|3181314

Other current X assets


X
Cash and cash equivalents
X
X Total assets
EQUITY AND LIABILITIES
Equity
Share capital X
Share premium account X
Revaluation reserve X
Retained earnings X
X Non- current liabilities
Long term borrowings X
Long term provisions X
Current liabilities
Trade payables X
Short term borrowings
X
Current tax payable X
Short term provisions X
Total equity and liabilities X
Statement of changes in equity – Proforma
Share Share Revaluation Retained Total
Capital Premium Reserve Earnings
’000 ’000 ’000 ’000 ’000
Balance at 31 March 20X7 X X X X X
Changes in accounting X X policy

!173 acowtancy.com

Downloaded by sajedul Islam (sajedulnsu15@gmail.com)


lOMoARcPSD|3181314

Restated balance X X X X X

Issue of share capital X X X

Dividends (X ) (X )

Total comprehensive X X X
income

Balance at 31 March 20X8 X X X X X

22.6 ACCA SYLLABUS GUIDE OUTCOME 1:-


Indicate the circumstances where separate disclosure of material items of income and
expense is required

The term ‘exceptional items’ refers to material items of income and expense of such size, nature
or incidence that disclosure is necessary in order to explain the performance of the entity.
The accounting treatment is to:
a. Include the item in the standard line in the statement of profit or loss
b. Disclose the nature and amount in notes.

!174 acowtancy.com

Downloaded by sajedul Islam (sajedulnsu15@gmail.com)


lOMoARcPSD|3181314

In some cases it may be more appropriate to show the item separately on the face of

• include the item in the standard line in the statement of profit or loss
• disclose the nature and amount in notes.
• write down of inventories to net realisable value (NRV)
• write down of property, plant and equipment to recoverable amount

• gains/losses on disposal of non-current assets


• discontinued operations

the statement of profit or loss.


Examples include:
• restructuring
• litigation settlements
• reversals of provisions.
CHAPTER 23:
IAS 33 – EARNINGS PER SHARE
The objective of IAS 33 is to prescribe principles for determining and presenting earnings per
share (EPS) amounts to improve performance comparisons between different entities in the same
reporting period and between different reporting periods for the same entity.

!175 acowtancy.com

Downloaded by sajedul Islam (sajedulnsu15@gmail.com)


23.1 ACCA SYLLABUS GUIDE OUTCOME 1:-
lOMoARcPSD|3181314

Calculate the EPS in accordance with relevant accounting standards (dealing


with bonus issues, full market value issues and rights issues).

Profits After Tax - Preference dividends

Weighted average number of ordinary shares

The earnings numerator should be after deducting all expenses including taxes and
Basic earnings per share
Calculation preference
dividends.
The denominator (number of shares) is calculated by adjusting the shares in issue at the
beginning of the period by the number of shares bought back or issued during the period,
multiplied by a time-weighting factor.
Date Total Time Bonus Weighted
Example - Weighted average number of shares calculation
Shares Apportionment Factor Average
1/1 100 3/12 4/3 33
1/4 300 3/12 4/3 100
1/7 400 6/12 200
333
1/1/2009 100 shares
1/4/2009 Bonus IssueFull market price issue of 200 shares
1/7/2009 1 for 3 bonus issue
In a bonus issue of shares, ordinary shares are issued to existing shareholders for no
23.1.1
additional consideration. In these circumstances, the number of ordinary shares
outstandingbeforethe issue is adjusted for the proportionate change in the number of
shares outstanding as if the event had occurred at the beginning of the earliest period
reported.

1 for 2 – now got 3 used to have 23/2=


2 for 5 – now got 7 used to have 57/5=
3 for 4 – now got 7 used to have 47/4=

!176 acowtancy.com

Downloaded by sajedul Islam (sajedulnsu15@gmail.com)


lOMoARcPSD|3181314

Analysing the net profit margin enables you to determine how well the business has managed to
control its indirect costs during the period. In the exam, when interpreting operating profit
margin, it is advisable to link the result back to the gross profit margin.
For example, if gross profit margin deteriorated in the year then it would be expected that the net
profit margin would also fall. However, if this is not the case, or the fall is not so severe, it may
be due to good indirect cost control or perhaps there could be a oneoff profit on disposal
distorting the operating profit figure.
Lecture Example 1
Comparator assembles computer equipment from bought-in components and distributes them to
various wholesalers and retailers. It has recently subscribed to an inter-firm comparison service.
Members submit accounting ratios as specified by the operator of the service, and in return,
members receive the average figures for each of the specified ratios taken from all of the
companies in the same sector that subscribe to the service.
The specified ratios and the average figures for Comparator’s sector are shown below.
Ratios of companies reporting a full year’s results for periods ending between 1 July 2003 and
30 September 2003:
Return on capital employed 22.1%
Net assets turnover 1.8 times
Gross profit margin 30% Net profit
(before tax) margin 12.5%
Current ratio 1.6:1 Quick ratio
0.9:1
Inventory holding period 46 days Accounts
receivable collection period 45 days Accounts payable
payment period 55 days
Debt to equity 40%
Interest cover 7 times
Dividend yield 6%
Dividend cover 3 times
Comparator’s financial statements for the year to 30 September 2003 are set out below:
Statement of Profit or Loss $000
Sales revenue 2,305
Cost of sales (1,870)
______
Gross profit 435
Other operating expenses (215)
______
Operating profit 220
Interest payable (34)

!177 acowtancy.com

Downloaded by sajedul Islam (sajedulnsu15@gmail.com)


lOMoARcPSD|3181314

______
Profit before taxation 186
Income tax (90)
______
Profit after taxation 96
______
Extracts of changes in equity:
Retained earnings – 1 October 2002 179
Net profit for the period 96
Dividends paid (interim $60,000; final $30,000) (90)
______
Retained earnings – 30 September 2003 185
______
Statement of Financial Position $000 $000 Non-current
assets (note (i)) 540
Current Assets
Inventory 275
Accounts receivable 320
Bank nil 595
_____ _______
1,135 _______
Share Capital and Reserves
Ordinary shares (25 cents each) 150
Retained Earnings 185
_______
335
Non-current liabilities
8% loan notes 300
Current liabilities
Bank overdraft 65
Trade accounts payable 350
Taxation 85 500
_____ ______
1,135 ______

Notes
1) The details of the non-current assets are:
Cost Accum depn Carrying value

!178 acowtancy.com

Downloaded by sajedul Islam (sajedulnsu15@gmail.com)


lOMoARcPSD|3181314

$000 $000 $000


At 30 September 2003 3,600 3,060 540
2) The market price of Comparator’s shares throughout the year averaged $6.00 each.
Required:-
a. Calculate the profitability ratios for Comparator equivalent to those provided by the
inter-firm comparison service.
b. Assess the performance of Comparator based on the ratios calculated in
(1) above.
(December 2003 ACCA Paper 2.5 adjusted)
24.4.2 Liquidity Ratios
24.4.2.1 Current Ratio
___Current Assets___
Current Liabilities
The current ratio considers how well a business can cover the current liabilities with its current
assets. It is a common belief that the ideal for this ratio is between 1.5 and 2 : 1 so that a business
may comfortably cover its current liabilities should they fall due.
However this ideal should be considered in the context of the company : the nature of the assets
in question, the company’s ability to borrow further to meet liabilities and the stability of its cash
flows.
For example, a business in the service industry would have little or no inventory and therefore
could have a current ratio of less than 1. This does not necessarily mean that it has liquidity
problems so it is better to compare the result to previous years or industry averages.
24.4.2.2 Quick Ratio
Current Assets – Inventories
Current Liabilities
One of the problems with the current assets ratio is that the assets counted include inventories
which may or may not be quickly sellable (or which may only be sellable quickly at a lower
price).
The ideal ratio is thought to be 1:1, but as with the current ratio, this will vary depending on the
industry in which the business operates.
The quick ratio is also known as the acid test ratio. This name is used because it is the most
demanding of the commonly used tests of short term financial stability.
When assessing both the current and the quick ratios, remember that both of these ratios can be
too high. This would mean too much cash is being tied up in current assets as opposed to new
more profitable investments.
It is important to look at the information provided within the question to consider whether or not
the company has an overdraft at year-end. The overdraft is an additional factor indicating
potential liquidity problems and this form of finance is both expensive (higher rates of interest)
and risky (repayable on demand)

!179 acowtancy.com

Downloaded by sajedul Islam (sajedulnsu15@gmail.com)


lOMoARcPSD|3181314

Lecture Example 2
Following from Lecture Example 1:-
a. Calculate the liquidity ratios for Comparator equivalent to those provided by the
inter-firm comparison service.
b. Assess the liquidity position of Comparator based on the ratios calculated in (1) above.
24.4.3 Efficiency Ratios
24.4.3.1 Receivables Collection Period (in days)
Trade Receivables x 365
Credit Sales
This ratio calculates how long credit customers take to pay.
A short credit period for receivables will aid a business’ cash flow. However, some businesses
base their strategy on long credit periods to achieve higher sales in highly competitive markets.
If the receivables days are shorter compared to the prior period, it could indicate better credit
control or potential settlement discounts being offered to collect cash more quickly whereas an
increase in credit periods could indicate a deterioration in credit control or potential bad debts.
24.4.3.2 Payables Payment Period (in days)
__Trade Payables__ x 365
Credit Purchases 73
This ratio calculates how long the company takes to pay its suppliers.
An increase in payables days could indicate that a business is having cash flow difficulties and is
therefore delaying payments. It is important that a business pays within the agreed credit period
to avoid conflict with suppliers.
If the payables days are reducing, this indicates suppliers are being paid more quickly. This could
be due to credit terms being tightened or taking advantage of early settlement discounts being
offered.
24.4.3.3 Inventory Days
__Closing (or average) Inventory_ x 365
COS
This ratio calculates how long goods to be sold stay in stock.
Generally, the lower the number of days that inventory is held the better as holding inventory for
long periods of time constrains cash flow and increases the risk associated with holding the
inventory. The longer inventory is held the greater the risk that it could be subject to theft,
damage or obsolescence. However, a business should always ensure that there is sufficient
inventory to meet the demand of its customers.
24.4.3.4 Working Capital Cycle
A company only gets cash once an item has been in stock and then the debtor pays (Inventory days
+ receivables days).

73 Take cost of sales if credit purchases are not given


!180 acowtancy.com

Downloaded by sajedul Islam (sajedulnsu15@gmail.com)


lOMoARcPSD|3181314

Inventory (in days) + Receivables (in days) – Payables (in days)

This needs to be kept as small as possible for liquidity purposes.

a. Calculate the efficiency ratios for Comparator equivalent to those provided


by the inter-firm comparison service.

b. Assess the performance of Comparator based on the ratios calculated in

This total should then be reduced by the payable days (the company doesn’t need the cash until the
end of this).
So, the working capital cycle (in days) is:
Lecture Example 3
Following from Lecture Example 1:-
(1) above.
24.4.4 Gearing Ratios
24.4.4.1 Gearing
A company can raise money by loans (Debt) or issuing shares (Equity).
Gearing can be calculated either:
_____Debt ______74 Debt
+ Equity75
OR
___Debt___
Equity
The gearing ratio is of particular importance to a business as it indicates how risky a business is
perceived to be based on its level of borrowing.
High gearing means high debt (in relation to equity). As borrowing increases so does the risk as
the business is now liable to not only repay the debt but meet any interest commitments under it.
If interest rates increase, then the company could be in trouble unless they have high enough
profits to cover this. In addition, to raise further debt finance could potentially be more difficult
and more expensive.
24.4.4.2 Interest Cover

74 Debt = Loans + Preference Shares

75 Equity = Ordinary share capital + Reserves + Non-controlling interest


!181 acowtancy.com

Downloaded by sajedul Islam (sajedulnsu15@gmail.com)


lOMoARcPSD|3181314

If a company has a high level of gearing it does not necessarily mean that it will face difficulties as
a result of this.
For example, if the business has a high level of security in the form of tangible noncurrent assets
and can comfortably cover its interest payments, a high level of gearing should not give an
investor cause for concern.
The interest cover is calculated:
Profit before Interest and Tax (PBIT)
Interest payable
A ratio of at least 3 is deemed to be satisfactory.
The interest coverage ratio is a measurement of the number of times a company could make its
interest payments with its earnings.
It is the equivalent of a person taking the combined interest expense from their mortgage, credit
cards etc, and calculating the number of times they can pay it with their annual income.
PBIT has its short fallings; companies do pay taxes, therefore it is misleading to act as if they
didn’t. A wise and conservative investor would simply take the company’s earnings before
interest and divide it by the interest expense. This would provide a more accurate picture of
safety.
Lecture Example 4
Following from Lecture Example 1:-
a. Calculate the gearing ratios for Comparator equivalent to those provided by the inter-
firm comparison service.
b. Assess the performance of Comparator based on the ratios calculated in (1) above.
24.4.5 Investor Ratios
24.4.5.1 Dividend per share
Ordinary dividend
Number of ordinary shares in issue ranking for dividend
The ratio tells equity holders the amount of dividend distributed per share. It will be influenced by
the profitability of the company, its capital structure and dividend policy.
Some companies at some stages of their development choose to distribute a large percentage of
post tax profits. Others – especially those experiencing fast growth – will want to plough as
much as possible back into their business.
24.4.5.2 Dividend cover
It is expressed as:
Earnings after tax and preference dividends = Number of times
Ordinary dividend
This shows how many times over the profits could have paid the dividend. For example, if the
dividend cover is 3, this means that the firm’s profit attributable to shareholders was three times
the amount of dividend paid out.

!182 acowtancy.com

Downloaded by sajedul Islam (sajedulnsu15@gmail.com)


lOMoARcPSD|3181314

Dividend cover is a measure of the ability of a company to maintain the level of dividend paid
out. The higher the cover, the better the ability to maintain dividends if profits drop. This needs
to be looked at in the context of how stable a company’s earnings are: a low level of dividend
cover might be acceptable in a company with very stable profits, but the same level of cover at a
company with volatile profits would indicate that dividends are at risk.
Because buyers of high yield shares tend to want a stable income, dividend cover is an important
ratio for income investors. Dividend cover is the inverse of the dividend payout ratio.
24.4.5.3 Dividend yield
This is the percentage rate of return by investing in shares at current market price. It is expressed as:
Dividend per share x 100
Market price per share
This shows how much a company pays out in dividends each year relative to its share price. It is the
return on investment for a share.
It is also a way to measure how much cash flow you are getting for each dollar invested in an equity
position – in other words, how much “bang for your buck” you are getting.
Investors who require a minimum stream of cash flow from their investment portfolio can secure
this cash flow by investing in stocks paying relatively high, stable dividend yields.
If two companies both pay annual dividends of $1 per share, but ABC company’s stock is trading
at $20 while XYZ company’s stock is trading at $40, then ABC has a dividend yield of 5% while
XYZ is only yielding 2.5%. Thus, assuming all other factors are equivalent, an investor looking
to supplement his or her income would likely prefer ABC’s stock over that of XYZ.
Lecture Example 5
Following from Lecture Example 1:-
a. Calculate the investor ratios for Comparator equivalent to those provided by the inter-
firm comparison service.
b. Assess the performance of Comparator based on the ratios calculated in (1) above.
24.4.6 How to answer interpretation questions
Step 1 – Look at the movement in cash
If up – How has this happened?
(a) Increased sales? Excellent news. Is this maintainable?
(b) Increased margins? Excellent again but is this sustainable long term?
(c) Better Liquidity? Good receivable, stock and payable management is good news, be careful
we are not missing out on suppliers early settlement discounts though.
(d) Got a loan? How much interest is being charged? Can this interest be paid? Is it below the
ROCE? What is the effect on gearing? What has the proceeds from the loan been spent on?
Assets or covering losses?
(e) Issued shares? What is the effect on dividend cover and yield? Again what has the money
been spent on?

!183 acowtancy.com

Downloaded by sajedul Islam (sajedulnsu15@gmail.com)


lOMoARcPSD|3181314

Also what is the ROCE? Should the extra cash be better invested rather than remaining in the bank
account.
The opposite applies if cash is down. Note though that if a loan has been repaid ensure that the
loan is charging interest higher than the current ROCE. If not the money is better spent investing
in new projects if possible.
NOTE: See how in all of these ratios will help!
Step 2
How profitable is the company?
(a) Look at ROCE compared to previous year. What has caused the increase/decrease?
(b) Look at all the margin ratios – does this help us understand where we are performing well
or badly?
(c) Has an increase in profitability resulted in an increase in cash? If not – then the company
may be overtrading. Overtrading arises when a company expands its sales revenue fairly
rapidly without securing additional long-term capital adequate for its needs. This means
that it is focussing too heavily on profits and not enough on liquidity. This is dangerous and
the symptoms are: - 1. Inventory increasing, possibly more than proportionately to revenue
2. Receivables increasing, possibly more than proportionately to revenue
3. Cash and liquid assets declining at a fast rate
4. Trade payables increasing rapidly
Step 3
How is gearing?
1) If high – is there a problem with interest cover?
2) If low – encourage the company to take out a loan if there are profitable projects.
NOTE: Do all of this but additionally ALWAYS, always, always use the scenario to help explain
the answers – this is where the marks are. Therefore, do not simply list all the possibilities of
why a ratio may have changed; link the reason to the scenario that you have been provided with.
24.6 ACCA SYLLABUS GUIDE OUTCOME 6:-
Discuss the effect that changes in accounting policies or the use of different accounting
policies between entities can have on the ability to interpret performance.
It can be useful to compare ratios for an individual company with those of other firms in the
same industry. However, comparing the financial statements of similar businesses can be
misleading because:
• the businesses may use different accounting policies
• ratios may not be calculated according to the same formula (for example, there are several
possible definitions of gearing and ROCE)
• large organisations can achieve economies of scale (e.g. by negotiating extended credit
periods, or discounts for bulk buying with suppliers) while these measures may not be
available to smaller businesses

!184 acowtancy.com

Downloaded by sajedul Islam (sajedulnsu15@gmail.com)


lOMoARcPSD|3181314

• companies within the same industry can serve completely different markets and there may
be differences in sales mix and product range. These can affect profitability and activity
ratios such as profit margin and expenses to sales.
24.7 ACCA SYLLABUS GUIDE OUTCOME 7:-
Indicate other information, including non-financial information that may be of relevance to
the assessment of an entity’s performance.
Additional financial information may be required in analysing the performance of a company such
as:
• Budgeted figures
• Other management information
• Industry averages
• Figures for a similar business
• Figures for the business over a period of time.
You may also require other non-financial information such as:
• Market share
• Key employee information
• Sales mix information
• Product range information
• The size of the order book
• The long-term plans of management

!185 acowtancy.com

Downloaded by sajedul Islam (sajedulnsu15@gmail.com)


lOMoARcPSD|3181314

24.8 ACCA SYLLABUS GUIDE OUTCOME 8:-


Discuss the different approaches that may be required when assessing the performance
of specialised, not-for-profit and public sector organisations
The main goal of specialised, not-for-profit and public sector organisations is to achieve value
for money. This is in contrast with the aim of a profit-making organisation whose aim is to
achieve a profit or return on capital.
Value for money is achieved by a combination of the three E’s: -
1. Effectiveness – success in achieving its objectives/providing its service
2. Efficiency – how well its resources are used
3. Economy – keep cost of inputs low
As profit and return are not so meaningful, many ratios will have little importance in these
organisations, for e.g. ROCE, gearing, investor ratios in general.
However, such organisations must also keep control of income and costs, therefore other ratios
will still be important such as working capital ratios.
As the main aim of these organisations is to achieve value for money, other nonfinancial ratios
take on added significance: -
1. Measures of effectiveness such as the time taken to treat out-patients are treated in a
hospital
2. Measures of efficiency such as the pupil-to-teacher ratio in a school
3. Measures of economy such as the teaching time of cheaper classroom assistants in a
school as opposed to more expensive qualified teachers
Further Questions
Question 1 85
Quartile is in the jewellery retail business which can be assumed to be highly seasonal. For the
year ended 30 September 2014, Quartile assessed its operating performance by comparing
selected accounting ratios with those of its business sector average as provided by an agency.
You may assume that the business sector used by the agency is an accurate representation of
Quartile’s business.
85
Which of the following circumstances may invalidate the comparison of Quartile’s ratios with
those of the sector average?
(i) In the current year, Quartile has experienced significant rising costs for its
purchases
(ii) The sector average figures are complied from companies whose year end is
between 1 July 2014 and 30 September 2014
(iii) Quartile does not revalue its properties, but is aware that other entities in this
sector do

Specimen Exam Applicable from December 2014

!186 acowtancy.com

Downloaded by sajedul Islam (sajedulnsu15@gmail.com)


lOMoARcPSD|3181314

(iv) During the year, Quartile discovered an error relating to the inventory count at 30
September 2013. This error was correctly accounted for in the financial statements for

The following information has been taken or calculated from Fowler’s financial
statements for the year ended 30 September 2014.

Fowler’s cash cycle at 30 September 2014 is 70 days.


Its inventory turnover is six times.
Year-end trade payables are $230,000.
Purchases on credit for the year were $2 million.
Cost of sales for the year was $1 ·8 million.
the current year ended 30 September 2014
A. All four
B. (i), (ii) and (iii)
C. (ii) and (iii) only
D. (ii), (iii) and (iv)
Question 2 86
What is Fowler’s trade receivables collection period as at 30 September 2014?
All calculations should be made to the nearest full day. The trading year is 365 days.
A. 106 days
B. 89 days
C. 56 days
D. 51 days
86
Question 3 87
Trent uses the formula:
(trade receivables at its year end/revenue for the year) x 365 to calculate
how long on average (in days) its customers take to pay.
Which of the following would NOT affect the correctness of the above calculation of the average
number of days a customer takes to pay?
Specimen Exam Applicable from December 2014

!187 acowtancy.com

Downloaded by sajedul Islam (sajedulnsu15@gmail.com)


lOMoARcPSD|3181314

A. Trent experiences considerable seasonal trading


B. Trent makes a number of cash sales through retail outlets
C. Reported revenue does not include a 15% sales tax whereas the receivables do
include the tax
D. Trent factors with recourse the receivable of its largest customer
Question 4
Extracts from the financial statements of Miller Co are shown below:
31 May 2012
$000
Revenue 475
Cost of sales (342) Gross profit
133
Expenses (59)
Finance cost (26)
Profit before tax 48
What is the interest cover ratio for the year ended 31 May 2012?
A. 2.85
B. 1.85
C. 5.12
D. 0.35
87

Specimen Exam Applicable from December 2014

!188 acowtancy.com

Downloaded by sajedul Islam (sajedulnsu15@gmail.com)


lOMoARcPSD|3181314

CHAPTER 25:
STATEMENTS OF CASH FLOWS
25.1 ACCA SYLLABUS GUIDE OUTCOME 1:-
Prepare a statement of cash flows for a single entity (not a group) in accordance with
relevant accounting standards using the direct and the indirect method.
IAS 7, Statements of Cash Flows, splits cash flows into the following headings:

2. Cash flows from investing activities


3. Cash flows from financing activities

Cash flows

outflows inflows

Cash Cash equivalents

Cash on hand Short term highly liquid


investments e.g. current
investments
1. Cash
flows from operating activities

!189 acowtancy.com

Downloaded by sajedul Islam (sajedulnsu15@gmail.com)


lOMoARcPSD|3181314

25.1.1 Cash flows from operating activities


These represent cash flows derived from operating or trading activities. There are two methods
which can be used to find the net cash from operating activities:- direct and indirect method.
The indirect method begins with profit before tax from the statement of profit or loss. This is
adjusted for the interest expense and investment income and also for non-cash items, e.g.
depreciation, amortisation, profit/loss on disposal and impairments. It is also adjusted for

Statement of profit or loss $80

SFP

X2 X1
$ $
Interest Payable 100 140

Opening Payable 140


Charge in statement of profit or loss 80

Cash paid (120)


increases and decreases in working capital.
Finance Costs – Illustration
Closing Payable 100
Finance costs go to the operating activities section of the statement of cash flow.
Taxation – Illustration
Statement of Profit or Loss $80
SFP
X2 X1
$ $
Tax Payable 100 140 Deferred Tax
50 80
Opening Payable 140 + 80 220 Charge in
Statement of profit or loss 80
300
Cash paid (150)
Closing Payable 100 + 50 150

!190 acowtancy.com

Downloaded by sajedul Islam (sajedulnsu15@gmail.com)


lOMoARcPSD|3181314

Tax goes to the operating activities section of the statement of cash flow.
25.1.2 Cash flows from investing activities
These are related to the acquisition or disposal of any non-current assets or investments together
with returns received in cash from investments, i.e. dividends and interest.
Investment Property – Illustration
Statement of Profit or Loss $80
SFP
X2 X1
Investment Property 200 140
(N.B: There were no purchases of investment property during the year)
Opening 140 Statement of profit
or loss 80
200
Cash received (20)
Closing Payable 200
Investment property income goes to the investing activities section of the statement of cash
flows.
Property, plant and equipment
We deal with this slightly differently:
Step 1: Write down the PPE figures per the accounts
Step 2: Work out the cash element of each item (if any)

!191 acowtancy.com

Downloaded by sajedul Islam (sajedulnsu15@gmail.com)


lOMoARcPSD|3181314

X2 X1

PPE 200 140

Disposal = asset sold for $100 making $20 profit

The key here is to try and find the balancing figure which will be additions in the year.
Illustration
SFP
Notes:
Depreciation in year = $50
Revaluation = $100
Note: we are dealing with NBVs
Step 1: Write down the PPE figures per the accounts
Opening 140
Depreciation (50)
Revaluation 100
Disposal (80) NBV = 100-20)
Additions 90 Closing
200
The balancing figure is $90 and this is additions.
Step 2: Work out the cash element of each item (if any)
Cash

Opening 140 -
Depreciation (50) -
Revaluation 100 -
Disposal (80) 100
Additions 90 (90)
Closing 200 -

!192 acowtancy.com

Downloaded by sajedul Islam (sajedulnsu15@gmail.com)


lOMoARcPSD|3181314

Both the additions and the cash proceeds go the investing activities section of the statement of
cash flows.
25.1.3 Cash flows from financing activities
Financing cash flows comprise receipts from or repayments to external providers of finance in
respect of principal amounts of finance. For e.g.:
(i) Cash proceeds from issuing shares
(ii) Cash proceeds from issuing debentures, loans, notes, bonds, mortgages
and other short or long term borrowings
(iii) Cash repayments of amounts borrowed (iv) Dividends paid to
shareholders
Shares – Illustration
SFP X2 X1
Share Capital 200 140 Share Premium
150 80
Opening 140 + 80 220 Cash
received 130 Closing
Payable 200 + 150 350
Share income goes to the financing activities section of the statement of cash flows. N.B.: If
there’s been a bonus issue:
1) Out of share premium – ignore it
2) Out of Retained earnings – reduce the cash by the amount
Loans – Illustration
SFP X2 X1
10% Loan 100 140
Opening 140
Cash paid (40) Closing
Payable 100
Loan repayments go to the financing activities section of the statement of cash flows.

!193 acowtancy.com

Downloaded by sajedul Islam (sajedulnsu15@gmail.com)


lOMoARcPSD|3181314

25.1.4 Statement of Cash Flows – Indirect Method


Statement of cash flows for the year ended 31 December 20X7 (INDIRECT
METHOD)
$000 $000
Cash flows from operating activities
Profit before taxation 3,390
Adjustment for:
Depreciation 450
Investment income (500)
Interest expense 400
3,740
Increase in trade and other receivables (500)
Decrease in inventories 1,050
Decrease in trade payables (1,740)
Cash generated from operations 2,550
Interest paid (270)
Income taxes paid (900)
Net cash from operating activities 1,380
Cash flows from investing activities
Purchase of property, plant and equipment (900)
Proceeds from sale of equipment 20
Interest received 200
Dividends received 200 Net cash used in
investing activities (480)
Cash flows from financing activities
Proceeds from issue of share capital 250
Proceeds from long-term borrowings 250
Dividends paid* (1,290)
Net cash used in financing activities (790)
Net increase in cash and cash equivalents 110
Cash and cash equivalents at beginning of period 120
Cash and cash equivalents at end of period 230 * This
could also be shown as an operating cash flow.
Lecture Example 1
The following draft financial statements relate to Tabba, a private company.
Statements of Financial Positions as at
30 September 2005 30 September 2004
$’000 $’000 $’000 $’000
Tangible non-current assets (note (ii)) 10,600 15,800

!194 acowtancy.com

Downloaded by sajedul Islam (sajedulnsu15@gmail.com)


lOMoARcPSD|3181314

Current assets
Inventories 2,550 1,850 Trade
receivables 3,100 2,600
Insurance claim (note (iii)) 1,500 1,200
Cash and bank 850 8,000 nil 5,560
Total Assets 18,600 21,450
Equity and liabilities
Share capital ($1 each) 6,000 6,000
Reserves:
Revaluation (note (ii)) nil 1,600
Retained earnings 2,550 2,550 850 2,450
8,550 8,450
Non-current liabilities
Finance lease obligations (note (ii)) 2,000 1,700
6% loan notes 800 nil
10% loan notes nil 4,000
Deferred tax 200 500
Government grants (note (ii)) 1,400 4,400 900 7,100
Current liabilities
Bank overdraft nil 550 Trade payables 4,050 2,950
Government grants (note (ii)) 600 400 Finance lease
obligations 900 800
Current tax payable 100 5,650 1,200 5,900
Total equity and liabilities 18,600 21,450
The following information is relevant:
(i) Statement of Profit or Loss extract for the year ended 30 September 2005:
$’000
Operating profit before interest and tax 270
Interest expense (260)
Interest receivable 40
Profit before tax 50
Net income tax credit 50
Profit for the period 100
Note: the interest expense includes finance lease interest.
(ii) The details of the tangible non-current assets are:
Cost Accumulated Depreciation Carrying value
$’000 $’000 $’000
At 30 Sept 2004 20,200 4,400 15,800
At 30 Sept 2005 16,000 5,400 10,600

!195 acowtancy.com

Downloaded by sajedul Islam (sajedulnsu15@gmail.com)


lOMoARcPSD|3181314

During the year Tabba sold its factory for its fair value $12 million and agreed to rent
it back, under an operating lease for a period of five years at $1 million per annum, at
the date of sale it had a carrying value of $7.4 million based on a previous revaluation
of $8.6 million less depreciation of $1.2 million since the revaluation. The profit on
the sale of the factory has been included in operating profit. The surplus on the
revaluation reserve related entirely to the factory. No other disposals of non-current
assets were made during the year.
Plant acquired under finance leases during the year was $1.5 million. Other purchases
of plant during the year qualified for government grants of $950,000.
Amortisation of government grants has been credited to cost of sales.
(iii) The insurance claim relates to flood damage to the company’s inventories which
occurred in September 2004. The original estimate has been revised during the year
after negotiations with the insurance company. The claim is expected to be settled in
the near future.
Required:
Prepare a statement of cash flow using the indirect method for Tabba in accordance
with IAS 7 Statements of Cash Flow for the year ended 30 September 2005.
(ACCA Paper 2.5 December 2005 Qs 4)
25.1.5 Statement of Cash Flows – Direct Method
In the direct method, the cash records of the business are analysed for the period, picking out all
payments and receipts relating to operating activities. These are summarised to give the net
figure for the cash flow statement. Not many businesses adopt this approach as it can be quite
time consuming. However, this is the preferred method under IAS 7.
$000 $000
Cash flows from operating activities
Cash receipts from customers 30,150

!196 acowtancy.com

Downloaded by sajedul Islam (sajedulnsu15@gmail.com)


lOMoARcPSD|3181314

(27,600)
2,550
(270)
(900)

$000liability
The following information is available about the transactions of Mermot, a limited
company, for the year ended 31 December 20X1.
Depreciation 880
Cash paid for expenses 2,270
Increase in inventories 370
Cash paid to employees 2,820
Decrease in receivables 280 Cash

Cash paid to suppliers 4,940


Decrease in payables 390
Cash received from customers 12,800
Net profit before taxation 2,370
payments to suppliers and employees
Cash generated from operations
Interest paid
Income taxes paid
Net cash from operating activities 1,380
Lecture Example 2
Mermot has no interest payable or investment income.
Required: Compute Mermot’s net cash flow from operating activities for the company’s cash
flow statement for the year ended 31 December 2001 using:
a. Direct method
b. Indirect method
25.2 ACCA SYLLABUS GUIDE OUTCOME 2:-
Compare the usefulness of cash flow information with that of a statement of profit or loss
or a statement of profit or loss and other comprehensive income.

!197 acowtancy.com

Downloaded by sajedul Islam (sajedulnsu15@gmail.com)


lOMoARcPSD|3181314

A business may appear profitable on its statement of profit or loss, however if its cash outflow
exceeds its cash inflow over a prolonged period then it will not survive.
Readers of a company's financial statements might also be misled by a reported profit figure.
1. Shareholders might believe that if a company makes a profit after tax, then this is the
amount which it could afford to pay as a dividend.
2. Employees might believe that if a company makes profits, it can afford to pay higher
wages next year.
3. Survival of a business entity depends not so much on profits as on its ability to pay its
debts when they fall due.
Indeed, a business must generate sufficient cash from its operations to reward the various
stakeholders e.g., shareholders and lenders. An expanding company might have negative
operating cash flow as it builds up the level of its inventories and receivables in line with the
increased turnover. However, an increase in working capital without an increase in turnover
might indicate operational inefficiencies and will lead to liquidity problems.
Benefits and Drawbacks of the Statement of Cash Flows
One of the most useful financial statements produced by a business is the statement of cash flow
because it provides a clear and understandable picture of cash movements over the financial
year. A statement of cash flow provides useful additional information that is not provided by the
statement of profit or loss. For example, it identifies whether cash has increased or decreased
from one year to the next and also where the cash has come from.
Statements of cash flow are a useful addition to the financial statements of a company because
accounting profit is not the only indicator of performance. They concentrate on the sources and
uses of cash and are a useful indicator of a company's liquidity and solvency. Also, users of
accounts can readily understand cash flows, as opposed to statements of profit or loss and
statements of financial position which are subject to manipulation by the use of different
accounting policies.
However, the main weakness of a statement of cash flow is that it is a historic statement.
Therefore, it does not indicate whether the business will be able to meet its debts in the future. A
more helpful statement would be a forecast statement of cash flow.
25.3 ACCA SYLLABUS GUIDE OUTCOME 3:-
Interpret a statement of cash flows (together with other financial information) to assess
the performance and financial position of an entity.
Statements of cash flow should help an external user to evaluate the effect of financial
management decisions taken during the year. However, the user will then need to decide whether
these cash movements are normal in relation to past movements and comparable industry
information, or abnormal and require further analysis.
Over time a business needs to generate positive cash flows from its operating activities,
otherwise it is unlikely to survive. Indeed, a business must generate sufficient cash from its
operations to reward the various stakeholders e.g., shareholders and lenders. An expanding

!198 acowtancy.com

Downloaded by sajedul Islam (sajedulnsu15@gmail.com)


lOMoARcPSD|3181314

company might have negative operating cash flow as it builds up the level of its stock and
debtors in line with the increased turnover. However, an increase in working capital without an
increase in turnover might indicate operational inefficiencies and will lead to liquidity problems.
The user of a statement of cash flow can also identify the impact of a business’s performance in
managing working capital. Changes in working capital on operating cash flow can be analysed
together with working capital ratios such as the inventory holding period, the receivables
collection period and the payables payment period. Another useful ratio to monitor a business’s
investment in working capital over time (and against other businesses) is that of working
capital/sales. This ratio shows how much capital is required to finance operations in addition to
the capital required to finance fixed assets.
Further Questions
Question 1
In the year ended 31 May 2012, Galleon Co purchased non-current assets with a cost of
$140,000, financing them partly with a new loan of $120,000. Galleon Co also disposed of non-
current assets with a carrying value of $50,000 making a loss of $3,000. Cash of $18,000 was
received from the disposal of investments during the year.
What should be Galleon Co’s net cash flow from investing activities according to IAS 7
Statement of cash flows?

The following extract is from the financial statements of Pompeii, a limited liability

2010 2009
$000 $000

120 80
60 40
85 68
–––– ––––
265 188

A. $45,000
B. $75,000
C. $69,000 D. $48,000 Question 2

!199 acowtancy.com

Downloaded by sajedul Islam (sajedulnsu15@gmail.com)


lOMoARcPSD|3181314

company at 31 October:
Equity and liabilities
Share capital
Share premium
Retained earnings
Non current liabilities
Bank loan 100 150
–––– ––––
365 338
–––– ––––
What is the cash flow from financing activities to be disclosed in the statement of cash flows for
the year ended 31 October 2010?
A. $60,000 inflow
B. $10,000 inflow
C. $110,000 inflow
D. $27,000 inflow
Question 3
Carter, a limited liability company, has non-current assets with a carrying value of $2,500,000 on
1 December 2007. During the year ended 30 November 2008, the following occurred:
1. Depreciation of $75,000 was charged to the income statement
2. Land and buildings with a carrying value of $1,200,000 were revalued to $1,700,000
3. An asset with a carrying value of $120,000 was disposed of for $150,000
The carrying value of non-current assets at 30 November 2008 was $4,200,000.

!200 acowtancy.com

Downloaded by sajedul Islam (sajedulnsu15@gmail.com)


lOMoARcPSD|3181314

What amount should be shown for the purchase of non-current assets in the statement
of cash flows for the year ended 30 November 2008?

Which of the following involves a movement of cash?


A.Depreciation of fixed assets
B.Creation of a provision for pensions

A. $1,395,000
B. $1,895,000
C. $1,425,000
D. $195,000
Question 4
C. A rights issue
D. A bonus issue
Question 5
Which one of the following events will reduce the cash balances of a business?
A. Purchase of fixed assets on interest free credit
B. Dividend proposed pending shareholder approval
C. Purchase of stock on credit
D. Creditors paid amounts owed
Question 6
A company with healthy profits is facing a cash shortage. Which of the following events could
account for this?
A. Delaying payments to creditors
B. The recent acquisition of equipment
C. An increase in dividends proposed by the directors
D. The shortening of the credit period granted to debtors
Question 7

!201 acowtancy.com

Downloaded by sajedul Islam (sajedulnsu15@gmail.com)


lOMoARcPSD|3181314

A.On profit – Decrease; On cash – Increase


B.On profit – Increase; On cash – Increase
C. On profit – None; On cash – Increase
D. On profit – Increase; On cash – Decrease

In IAS 7 Statement of Cash Flows where would you find a bank current account debit

A.In investing activities


B.In operating activities
C. In financing activities
D. In cash and cash equivalents

Question 8
balance?

!202 acowtancy.com

Downloaded by sajedul Islam (sajedulnsu15@gmail.com)

You might also like